Zanimljiva matematika

68

Transcript of Zanimljiva matematika

Page 1: Zanimljiva matematika
Page 2: Zanimljiva matematika

Nakon osam godina ponovo sa Vama!

γ Šta je to GAMA?

GAMA je časopis za kreativne mlade matematičare, fizičare i programere, ali i za one koji su dosad mislili da to nijesu.

γ KomejenamijenjenčasopisGAMA?

Svim učenicima gimnazija i ostalih srednjih škola u Crnoj Gori, ali će u njemu ponešto moći da pročitaju i mlađi, a mnoge stvari će, svakako, zanimati i starije!

γ Ko je zaslužan za drugi broj časopisaGAMA?

U prvom redu, kreativni učenici Gimnazije „Slobodan Škerović“ iz Podgorice, a zatim grupa profesora entuzijasta iz iste škole kao i profesori sa Prirodno-matematičkog fakulteta, iz Zavoda za školstvo i Ispitnog centra.

γ ŠtasuambiciječasopisaGAMA?

Prije svega da ga prihvatite kao najbolji časopis za popularisanje matematike, fizike i informatike. U tom cilju trudićemo se da sa čitaocima, kako učenicima tako i profesorima i nastavnicima, uspostavimo što prisniju saradnju, čime ćemo iz broja u broj biti još bolji. To zaista nije teško jer matematika i fizika su tako lijepe, korisne i neizbježne, toliko prepune divnih ideja i neočekivanih rješenja „nerješivih“ problema da je dovoljno biti samo aktivni posmatrač, učesnik i objektivni reporter, naravno uz veliku informatičku podršku.

Dakle, PIŠITE NAM! KRITIKUJTE NAS ILI POHVALITE! PITAJTE NAS! PREDLAŽITENAM!

REDAKCIJA

Napomena: Zbog finansijskih problema, koji nažalost prate jedan ovakav časopis, nijesmo mogli da štampamo časopis u prošloj školskoj godini. Zato skrećemo pažnju čitaocima da su odjeljenja učenika, autora članaka, važeća za školsku 2007/08. godinu.

Ne znati nije sramota.

Sramota je ne htjeti znati.

Sokrat

Page 3: Zanimljiva matematika

1

Sadržaj:

Poznati matematičariPitagora 2

Jedan problem – više rješenjaPitagorina teorema 4

Anegdote 9Feljton

Istorija matematike – I dio 10Gama istražuje

Piramide u Gizi/Grobnice ili kosmičko savršenstvo 12Saznajte nešto više

Nejednakost između aritmetičke i geometrijske sredine 16Određivanja minimalne i maksimalne vrijednosti u zadacima iz fizike primjenom nejednakosti aritmetičke i geometrijske sredine 20

Paradoksi i sofizmi 24Zanimljiva matematika

BrojeviMirakuli 26

Šibice 27Zanimljiva matematika 28

Zanimljiva fizikaPitanja i odgovori „10+10“ 29

Logičke igreSudoku 30

Zanimljiva matematikaMatematičko-logičke zagonetke i problemi 32Ajnštajnov zadatak 33

Citati 34Zanimljiva fizika 35U razgovoru sa...

Intervju – Bećo Merulić 36Odabrani zadaci

Matematika 37Takmičenja

Državno takmičenje 2008.Matematika 38Fizika 46Informatika 57

U korak sa ...Kako učiti informatiku 62

Odgovori i rješenja 63

GAMAčasopis za mlade matematičare, fizičare i programere, broj 2, februar 2009. godineIzdavač: Gimnazija „Slobodan Škerović”, PodgoricaZa izdavača: Radiša ŠćekićGlavni i odgovorni urednik: Mirjana Pješčić, prof.Urednik učenik: Rastko Pajković IIIiRedakcija:Božo Baković, prof. (urednik za informatiku)Radovan Ognjanović, prof. (urednik za fiziku)Boljević Gordana, učenik IVhBožidar Bukilić, učenik IIIlĆorović Lilanda, učenik II10Džaković Jelena, učenik II10Lakićević Milan, učenik IIIlMaraš Irena, učenik IVjPopović Ivana, učenik IIIlPopović Jovana, učenik I12Radević Mihailo, učenik II10Radoman Milena, učenik II2Šestović Bojana, učenik IVhTapušković Matija, učenik II2

Lektor: Mirjana Perović, prof.Priprema na računaru i dizajn korice:Rastko Pajković, učenik IIIiŠtampa: M Print - Podgorica

Page 4: Zanimljiva matematika

2

Rođen je na ostrvu Samosu u VI v.p.n.e. (oko 580. god.). Imao je 18 godina kada je učestvovao na Olimpijskim igrama, gdje je odnio sve nagrade

u pesničenju. Poslije pobjede odlučio je da putuje: Jonija, Sirija, Liban samo su neke od destinacija, ali su za njegovu nauku najznačajniji susreti sa mnogim učiteljima i misliocima onog vremena, među kojima je bio i Tales iz Mileta, kao i boravci u Vavilonu i Egiptu. U Vavilonu je u zarobljeništvu izučavao tamo šnju matematiku, dok je u Egiptu upoznao egipatsku geometriju i, na kraju, postao egipatski sveštenik, upućen u njihove tajne obrede. Kao pedesetogodišnjak seli se u Kroton gdje osniva religiozno-asketsko bratstvo (pitagorejce), tajno društvo posvećeno proučavanju brojeva.

Moglo bi se reći da je Pitagora gotovo mitska ličnost jer čak nema sačuvanih prepisa njegovih originalnih tekstova. Moguće da su i njegovi sljedbenici učitelju osnivaču pripisivali niz kasnijih sopstvenih matematičkih postignuća i filosofskih uvida. Kao kakav kaluđerski ili viteški red, bratstvo je imalo svoja pravila i njegovalo strog način života. Da bi izbjegli da njihove tajne saznaju stranci koji nijesu pripadali zajednici, većinom su svoja znanja prenosili usmeno. Napisano ostaje, usmeno odlijeće. Da njihove riječi ne bi odletjele, usavršili su brojne vježbe za razvijanje pamćenja. Stoga su postojali akusmatici1, kojima su prenošeni rezultati, ali ne i dokazivanja neophodna da bi se do njih dospjelo i matematičari2, kojima su prenošeni rezultati i dokazi. Matematičare je podučavao sam Pitagora. Ženama je takođe bilo dozvoljeno da pripadaju redu. Među njima najatraktivnija je bila mlada i lijepa Teano, kojom se

1 akuzmatikoi(grč.) - „oni koji žive u svojim kućama”, posjećivali su zajednicu samo tokom dana.

2 mathematikoi(grč.) - živjeli su stalno u zajednici, bez ličnih interesa

Pitagora oženio. Pitagorejci su vjerovali u reinkarnaciju i propovijedali strogo vegeterijanstvo.

Pitagora se bavio muzikom i matematikom. Kao filosof, smatrao je da je suština svih stvari broj i da se sve u vasioni može svesti na nepromjenljivu harmoniju brojčanih odnosa. Pripisuje mu se učenje o monadama (jedinice koje su isto što i besmrtne ljudske duše), Pitagorina teorema, otkriće prve muzičke skale i figure nazvane „tetraktys” (slika 1) koja implicira dekadni sistem brojeva, zatim dokaz teoreme da je zbir unutrašnjih uglova u trouglu 180 stepeni, itd.

Kako je za pitagorejce broj iskon svega, smatrali su da su jedini dopustivi brojevi prirodni a veličine one koje se mogu opisati

kao njihovi odnosi (tj. kao racionalni broj). Ipak, zanimljivo je da je jedno od njihovih najvećih otkrića upravo suprotno tom uvjerenju. Naime, radi se o nesamjerljivosti stranice i dijagonale kvadrata (slika 2). Znamo da su dvije veličine, a i b, samjerljive ako im je odnos racionalan broj. Danas, pak, znamo da je odnos dijagonale d i stranice a kvadrata jednak kvadratnom korijenu iz 2, koji je iracionalan

broj, pa je ta činjenica očigledno u suprotnosti s osnovnim vjerovanjem pitagorejaca da je sve opisivo pomoću prirodnih brojeva.

Oni su to, zapravo, i sami otkrili te je to uzdrmalo temelje njihovog kulta.

Matematičarima je uvedena zabrana iznošenja rezultata van članova unutrašnjeg kruga. Postoji anegdota da je Hipas prekršio tu zabranu i za kaznu bio prisiljen da izvrši samoubistvo skokom u more sa neke visoke litice.

Postoje različite pretpostavke o Pitagorinoj smrti. Neki čak tvrde da je umro od sreće kad je pronašao neku svoju teoremu. Zanimljivo, ali vjerovatno netačno. Jedna od njih kaže da je Pitagora, bježeći sa svojim učenicima

Pitagora

„Čovjek koji je svuda vidio brojeve”

Pitagora je prvi izrekao: „Ja sam filosof”

nazivajući tako sebe onim koji teži mudrosti.

Poznati matematičari

Page 5: Zanimljiva matematika

3

iz Krotona, čiji su se stanovnici bili digli protiv njega, naišao na polje pasulja preko koga je trebalo umaći progoniteljima. Pošto su, prema vjerovanju, u pasulju prebivale duše pokojnika, Pitagora ne htjede da pregazi mahunarke. „Radije ću umrijeti nego da pregazim jadni pasulj”, rekao je i sjeo kraj polja. Stanovnici Krotona su ga sustigli i masakrirali i njega i njegove učenike. Druga teorija, pak, kaže da je u miru proživio ostatak života...

1

1 2

Slika 1. figura „tetraktys” - implicira dekadni sistem

brojeva

Slika 2. dijagonala kvadrata stranice 1

Evo samo nekih od brojnih Pitagorinih otkrića:

Čuvena Pitagorina teorema čiji je prvi pisani dokaz -dao Pitagora, iako se danas zna da su je prije njega koristili Egipćani, Vavilonci, Indusi, Kinezi i druge stare civilizacije. Ova teorema izazivala je veliko interesovanje kroz istoriju matematike. Legenda kaže da je, pošto je otkrio čuve nu teoremu, Pitagora priredio hekatombu - svečanost žrtvovanja stotinu volova, tada zastupljenu u Grčkoj. Od tada, kažu, volovi ne vole matematiku...

Pitagora je otkrio da je muzički interval odnos dva broja, -pa je na primjer interval oktave 1/2, kvinte 2/3, a kvarte 3/4. Brojčani odnosi tako postaju kadri da predstavljaju muzičke harmonije. Za pitagorejce, cijelo nebo je sazdano od muzičkih ljestvica i brojeva, pa je muzika harmonija sfera.

Pitagorejci su u matematiku uveli pojam „savršenog -broja”, broja koji predstavlja zbir svih svojih pravih djelilaca. Prvi savršen broj je 6, zbir trojke, dvojke i jedinice a šestica je ujedno djeljiva svim ovim brojevima. Naredni savršen broj je 28.

6 3 6 2 6 1 6 +

28 14 28 7 28 4 28 2 28 1 28 +

Pitagora je prvi podijelio brojeve na kategorije neparnih i -parnih, ali je otišao i korak dalje, personifikujući neparne kao „muške” a parne kao „ženske”.

Pitagorejci su među brojevima „pronašli” brojeve -prijatelje – dva broja koja imaju svojstvo da je jedan jednak zbiru svih pravih djelilaca drugog i obrnuto. Primjer: 220 i 284. Pravi djelioci broja 220 su: 1, 2, 4, 5, 10, 11, 20, 22, 44, 55 i 110, čiji zbir daje 284. Pravi djelioci broja 284 su: 1, 2, 4, 71 i 142, čiji zbir daje 220. Anegdota kaže, da je, kada su Pitagoru pitali kako bi opisao pravog prijatelja, slavni matematičar i filosof odgovorio: „Prijatelj je kao drugo ja, kao broj 220 broju 284”

284 142 284 71 284 4 284 2 284 1 220 +

220 110 220 55 220 44 220 22 220 20 220 11 220 10 220 5 220 4 220 2 220 1 284 +

Smatra se da ime discipline - matematika svoje porijeklo -duguje pitagorejcima. Naime, uži krug njegovih sljedbenika, koji su sebe nazivali „mathematikoi”, ime je očigledno dobio iz ritualnih i religijskih razloga. Pitagora im je, impresioniran ritualima egipatskih sveštenika, dao ime po mathemi - nazivu za jednu vrstu zemlje koju su egipatski sveštenici koristili u ritualima. Riječ matematika, nastala na opisani način, po Pitagorinom mišljenju, trebalo je da označava „ono što se uči”.

Bojana Šestović,Gordana Boljević

IV-h

c2 = a

2 + b2

ab

c

Poznati matematičari

Page 6: Zanimljiva matematika

4

AB

C

U geometriji je veoma značajna teorema koja nosi ime Pitagore, a odnosi se na pravougli trougao: zbir površina kvadrata konstruisanih nad katetama jednak je površini kvadrata konstruisanog nad hipotenuzom. Drugim riječima, ako su a i b dužine kateta i c dužina hipotenuze onda je a² + b² = c²

Prema onome što se zna, do danas nije poznato kako je i ko prvi dokazao ovu teoremu. Pretpostavlja se da je najprije pronađen dokaz za jednakokrako pravougli trougao ∆ABC, jer za ovakav trougao, povlačenjem dijagonala A1C, B1C, AB2 i BA2 neposredno proizlazi da se kvadrat ABB2A2 nad hipotenuzom sastoji iz četiri međusobno podudarna jednakokraka trougla koja su podudarna sa jednakokrakim pravouglim trouglovima: ∆A1AC , ∆A1C2C, ∆BB1C i ∆B1C1C od kojih su sastavljeni kvadrati nad katetama, kao i sa datim trouglom ∆ABC, jer imaju jednake hipotenuze i uglove na njoj od 45º.

A B

C

C2 C1

B1

B2A2

A1

Danas je poznato oko stotinu dokaza ove teoreme za proizvoljan pravougli trougao, ali se uočava da se dokazi mogu podijeliti u četiri grupe:

1. dokazi koji koriste razlaganje površina (mozaički dokazi).2. dokazi koji koriste izjednačavanje površina,3. dokazi pomoću računanja,4. dokazi koji koriste sličnost geometrijskih figura.

Dokaz 1: papir i makaze

Teoremu možemo dokazati sa samo dva reza makazama. Ukoliko kvadrate ABCD i EFGH, izrezane od papira, postavimo kao na slici dolje lijevo i spojimo tačke D i G sa B’ (A–B’–F i B’F = AB), nastaće dva podudarna pravougla trougla. Pri tom će manji kvadrat biti kvadrat nad manjom katetom, a veći nad većom. Sa dva reza makazama po B’D i B’G dobijaju se djelovi od kojih se može nasložiti kvadrat uz hipotenuzu bilo kojeg od dva podudarna trougla (slika dolje desno). Površina trećeg kvadrata jednaka je zbiru dva data manja.

Dokazi Pitagorine teoreme„Kvadrat nad hipotenuzom, to zna svako dete, jednak je zbiru kvadrata nad obe katete”

Branislav Nušić

A B E

CD

H G

FB’

Jedan problem―više rješenja

Page 7: Zanimljiva matematika

5

Dokaz 2

Jedan od dokaza potiče od čuvenog indijskog matematičara Bhaskare koji je rođen 114. godine. Bhaskara je sastavio četiri podudarna trougla ∆ABC, ∆EAH, ∆DEF i ∆BDG. Hipotenuza tih trouglova je jednaka stranici c, dok duže katete b tih trouglova obrazuju unutrašnji kvadrat HCGF dužine stranica b – a. Gledajući sliku, zaključuje:

b a b a c−( ) + ⋅ ⋅ =2 242

, odakle je a² + b² = c²

A B

C

DE

F

G

Haa

aa

c

c

c

c

b

b

b

b

b–a

Dokaz 3: Garfildov dokaz

Dokaz potiče od Džejmsa Abrahama Garfilda (1831-1881), koji je postao američki predsjednik 1881. godine. Konstruišimo duž BD normalno na AB, tako da je AB = BD = c i spustimo iz tačke D normalu DE na pravu koja sadrži stranu CB (raspored tačaka na pravoj je C – B – E).

∆ACB i ∆BED su pravougli trouglovi čija je stranica AB = BD = c i ∠CBA = ∠EDB (uglovi sa normalnim kracima), odakle slijedi da su oni podudarni i da je ED = CB = a i AC = BE = b. Četvorougao ACED je trapez.

c

c

a

a

b

b

DE

B

AC

PACED = 2P∆ACB + P∆ABD

čime je dokaz završen.

a b a b ab c a ab bab c

a ab b a

+( ) +( )= + ⇔

+ +( )= + ⇔

⇔ + + =2

22 2

2

2 22 2

2 2 2 2

2 2

bb c a b c+ ⇔ + =

2 2 2 2 ,

Dokaz 4

Posmatrajmo pravougli trougao ∆ABC sa pravim uglom kod tjemena C Konstruišimo kvadrat nad hipotenuzom c, tj. FBGA i kvadrate nad katetama a i b, odnosno ACDE i CBKQ Iz jednakosti stranica AC = AE, FA = AB i jednakosti uglova ∠EAB = ∠CAF slijedi podudarnost trouglova ∆EAB i ∆CAF, tj. jednakost njihovih površina, odnosno b² = x ∙ c (1).Analogno uočavamo da su i trouglovi ∆ABK i ∆GBC podudarni (AB = BG, CB = BK i ∠ABK = ∠GBC) odakle slijedi: a² = y ∙ c (2).Iz (1) i (2) slijedi: a² + b² = (x + y) ∙ c = c², što je i trebalo dokazati.

Jedan problem―više rješenja

c

c

a

a

b

b

BC

E≡A

DF

E

D

Q

K

G

B

F

A

C

N

xc

x y

yc

a

a

b

b

c c

c

a2

b2

A BD

C

c⁄2

c⁄2c⁄2

b a

α⁄2

γ⁄2 γ⁄2

β⁄2

r=x

x x=rB1 A1

C

A B

r

Oy

y z

z

r

A1 A B1

C

Bc

ab

b

Page 8: Zanimljiva matematika

6

Dokaz 5: vektorski

Pitagorinu teoremu možemo dokazati i koristeći se svojstvima skalarnog proizvoda vektora.

Kako je vektor

c jednak zbiru vektora

a i

b, to :

c c a b a b

c a a a b b b a a b

⋅ = +( ) ⋅ +( )= ⋅ + ⋅ + ⋅ = + ⋅

2

2 2 92

cos 0

0

2 2 2

° + = +b a b

,

odakle slijedi c² = a² + b², što je trebalo dokazati.

Dokaz 6: korišćenje Apolonijeve teoreme

Poznatu relaciju koja vlada među stranicama pravouglih trouglova možemo dokazati i korišćenjem Apolonijeve teoreme:

„Ako je D tačka stranice AB trougla ∆ABC takva da je AD : BD = m : n, tada je

n ∙ AC ² + m ∙ BC ² = n ∙ AD ² + m ∙ BD ² + (m + n) ∙ CD²”

Kod pravouglog trougla ∆ABC, sa pravim uglom kod tjemena C, očigledno je m = n = 1, pa prethodna relacija dobija oblik:

AC² + BC² = AD² + BD² + 2 ∙ CD²,

odakle slijedi:

a b c c c2 22 2 2

2 22

2+ =

+

+

, tj. a² + b² = c², čime je teorema dokazana.

Dokaz 7

Posmatrajmo pravougli trougao ∆ABC sa uglovima α, β, γ koji odgovaraju stranicama dužine a, b, c i neka je r poluprečnik kružnice upisane u taj trougao. Neka su A1, B1 i C1 dodirne tačke kružnice sa stranicama BC, CA i AB. Obilježimo sa P površinu, sa s poluobim, a sa x, y i z respektivno dužine parova tangentnih duži CB1 i CA1, AB1 i AC1, BA1 i BC1 datog trougla

Važi relacija: tg tg tg tg tg tgα β α γ β γ2 2 2 2 2

+

+

221 0 0 0

= + + = > > >, , , , . α β γ π α β γ

Dokaz:

tg rytg r

ztg r

xα β γ2 2 2

=

=

=, , i pa polazeći od lijeve strane jednakosti dobijamo:

tg tg tg tg tg tgα β α γ β γ2 2 2 2 2

+

+

22

2

= ⋅ + ⋅ + ⋅ =

+ +( )⋅ ⋅

=ryrz

ryrx

rzrx

r x y zx y z

= ⋅⋅ ⋅

= ⋅⋅ ⋅ ⋅

=⋅( )

⋅ −( ) ⋅ −( ) ⋅ −( )= =r s

x y zr ss x y z

r ss s c s a s b

PP

2 2 2 2 2

2 1.

Vratimo se dokazu Pitagorine teoreme.

Kako je x = r i γ

2450= , dobijamo:

c

c

a

a

b

b

BC

E≡A

DF

E

D

Q

K

G

B

F

A

C

N

xc yc

a

a

b

b

c c

c

a2

b2

A BD

C

c⁄

c⁄c⁄

b a

α⁄2

γ⁄2 γ⁄2

β⁄2

r=x

x x=rB1 A1

C

A B

r

Oy

y z

z

r

A1 A B1

C

Bc

ab

b

Jedan problem―više rješenja

Page 9: Zanimljiva matematika

7

r r y zr y z

r r y zy z

Py z

P y z2

1 1 1⋅ + +( )⋅ ⋅

= ⇒⋅ + +( )

⋅= ⇒

⋅= ⇒ = ⋅ .

Kako je P površina pravouglog trougla, to je 2P = ab, pa je:

a b y z a b s a s b

a b b c a a c b

a

⋅ = ⋅ ⋅ ⇒ ⋅ = ⋅ −( ) ⋅ −( ) ⇒

⇒ ⋅ = ⋅ + − ⋅ + − ⇒

2 2

22 2

⋅⋅ = ⋅ − + − ⇒

⇒ = +

b a b b c a

c a b

2 2 2

2 2 22 2 2

Dokaz 8: Hofmanov dokaz

Ovaj dokaz (Majnic, 1821. god.) koristi važno svojstvo kruga:

Teorema: Ako je P tačka van kruga k, u ravni tog kruga, tada je proizvod odsječaka koje krug k određuje na bilo kojoj sječici povučenoj iz tačke P jednak kvadratu odgovarajuće tangentne duži.

Dokaz: Neka su B i C presjeci sječice, povučene iz tačke P, sa krugom k. Tada imamo da je ∆CPA~∆BPA (∠CPA=∠BPA, ∠CAP=∠ABP; ugao između tetive i tangente ∠CAP u jednoj krajnjoj tački tetive jednak je periferijskom uglu ∠ABP nad tetivom). Iz sličnosti trouglova slijedi da je AP : PB = PC : AP ⇒ AP² = PC ∙ PB, što je trebalo dokazati.

PC

A

B

k

Koristeći ovo svojstvo, dokazaćemo Pitagorinu teoremu. Kada se oko tjemena A pravouglog trougla ∆ABC, kao centra, opiše krug sa poluprečnikom AC = b i hipotenuza AB produži do presjeka A1 sa ovim krugom, tada prema navedenoj teoremi važi:

BC BA BB a c b c b

a c bc a b

21 1

2

2 2 2

2 2 2

= ⋅ ⇒ = +( ) ⋅ −( ) ⇒

⇒ = − ⇒⇒ = +

α⁄

γ⁄ γ⁄

β⁄

r=x

x x=rB1

C1

A1

C

A B

r

Oy

y z

z

r

A1 A B1

C

Bc

ab

b

Jedan problem―više rješenja

Page 10: Zanimljiva matematika

8

Dokaz 9: Simpsonov dokaz

Na kraju, dozvolićemo sebi da odstupimo od matematičke strogoće zapisa dokaza teoreme i dati jednu interpretaciju dokaza po Simpsonu (Elements de geometrie, Pariz, 1766.), koji se smatra mogućim dokazom samog Pitagore. Dokaz je na svoj način izložio Leonard Mlodinov u svom djelu „Euklidov prozor”, koji ovdje navodimo u originalu:

„Da bi se stvari pojednostavile, nazovimo nekako stranice trougla. Hipotanuza već ima naziv, premda poduži, pa ćemo ga zadržati, ali ćemo staviti veliko početno slovo, Hipotenuza, da bismo je razlikovali od matematičkog naziva hipotenuza. Neka se dve katete pravouglog trougla zovu Aleksej i Nikolaj. Igrom slučaja, upravo su ovo imena mojih sinova. U času dok ovo pišem, Aleksej je viši od Nikolaja, pa neka veća kateta bude nazvana Aleksej, a manja Nikolaj (dokaz ostaje na snazi i ako su katete jednake). Počećemo tako što ćemo nacrtati kvadrat čija je osnovica zbir Alekseja i Nikolaja. Stavimo potom po jednu tačku na svaku stranu kvadrata, na ono mesto gdje se susreću dužina Alekseja i Nikolaja, a potom spojimo te tačke. Dva načina koja nas zanimaju prikazana su na slici. Na prvom crtežu imamo kvadrat čija je osnovica Hipotenuza i četiri trougla „viška”. Na drugom crtežu imamo dva kvadrata čije su osnovice Aleksej i Nikolaj i dva pravougaonika „viška” koja se mogu dijagonalno prepoloviti, čime se dobijaju četiri trougla istovetna onima koja predstavljaju „višak” na prvom crtežu.

Preostaje samo računanje. Dva izdeljena kvadrata imaju istovetne površine, tako da nam, kada uklonimo po četiri ista trougla „viška” sa svakog od njih, preostaju dve površine koje moraju biti istovjetne. Jedna od njih je kvadrat čija je osnovca Hipotenuza, dok su u drugom slučaju posredi dva kvadrata čije su osnovice Aleksej, odnosno Nikolaj. Time je teorema dokazana!”

Literatura:1. Đoko G. Marković, Geometrijski poliformizam, Podgorica, 2006.2. Borisav Simić, I to je matematika, Zavod za udžbenike i nastavna sredstva, Beograd, 1992.3. Leonard Mlodinov, Euklidov prozor, Laguna, 2005.4. http://alas.matf.bg.yu/~zlucic/view_doc.php?id=391

Bojana ŠestovićGordana Boljević

IV- h

Mladi, siromašni matematičar objašnjava jednom francuskom plemiću dokaz Pitagorine teoreme. Objašnjava strpljivo i polako, ali svaki put plemić odgovara: „Ne razumijem.” Nakon više uzaludnih pokušaja mladi instruktor izgubi živce: „Monsenjor, kunem vam se svojom čašću da je Pitagorina teorema istinita!” U taj tren plemić ustaje, ljubazno se nakloni, i s izrazom čuđenja kaže: „Trebali ste mi to odmah reći. Ne bi mi nikad palo na pamet da posumnjam u vašu čast...”

Jedan problem―više rješenja

Aleksej

Aleksej

Aleksej

Aleksej2

Aleksej

Aleksej

Nikolaj

Nikolaj

Nikolaj

Nikolaj2

Nikolaj

Nikolaj

Hipotenuza

Hipotenuza

Aleksej

Aleksej

Aleksej

Aleksej

Nikolaj

Nikolaj

Nikolaj

Nikolaj

Hipotenuza

Hipotenuza

Page 11: Zanimljiva matematika

9

Anegdoteo poznatim matematičarima

EuklidNa kraju prvog predavanja koje je Euklid održao jednoj grupi studenata početnika, jedan od studenata upitao ga je: „A šta će nam u životu matematika?” Euklid nije odgovorio ništa. Nakon pola sata po svome robu poslao mu je jedan zlatnik i otpustio ga iz škole.

TalesMlinDiogen Laertije piše: „Hijeronim s Roda priča da je Tales, želeći da pokaže kako je lako obogatiti se, predviđajući da će biti dobar rod maslina, uzeo u zakup sve mlinove maslina i tako nagomilao čitavo bogatstvo.” Istu su anegdotu prenijeli i Ciceron i Aristotel, koji u „Politici” piše ovako:

„Jer kad su mu zbog njegova siromaštva prigovorili kako je filosofija beskorisna, on je, kažu, saznavši posmatrjući zvijezde da će te godine biti dobar urod maslina, već zimi s ono malo novca što je imao veoma povoljno zakupio sve prese za ulje u Miletu i na Hiju, jer niko nije nudio više novca. Kad je zatim došlo pravo vrijeme, i nenadano su se i istovremeno tražile mnoge prese, iznajmljivao ih je po koliko je on htio, pa je, zaradivši mnogo novca, pokazao kako je filosofima lako obogatiti se kad to hoće, ali to nije ono čemu oni teže. Govori se kako je na taj način Tales dokazao svoju mudrost.”

GausGaus se kao dijete isticao svojom oštroumnošću. Postoji anegdota da je jednom prilikom iznenadio svoga učitelja koji je učenicima zadao da saberu sve prirodne brojeve do 100. Samo što je učitelj sjeo, devetogodišnji Gaus se javio da je uradio zadatak.– Kako? – pitao je učitelj.– Vrlo lako. Grupisao sam brojeve u 50 parova : u prvom su brojevi 1 i 100, u drugom 2 i 99, u trećem 3 i 98 itd. do pedesetog para u kome su brojevi 50 i 51. Zbir brojeva u svakom paru je 101, pa zbir brojeva od 1 do 100 iznosi 101∙50=5050

DirihlePriča se da njemački matematičar Dirihle nije bio mnogo razgovorljiv. Kada mu se rodio sin,on je svojoj tašti poslao telegram koji je vjerovatno najkraći u istoriji telegrafije. Glasio je: 2+1=3

Bojana Šestović

Gordana Boljević

IV-h

A

B

C

DE

Page 12: Zanimljiva matematika

10

Mesopotamija1, područje između i oko Eufrata i Tigra na Arabijskom poluostrvu, danas središnji dio Iraka, bila je kolijevka nekoliko najstarijih kultura. Govoreći o matematici stare Mesopotamije, podrazumijevamo ostavštinu Sumera, Vavilonaca, Asiraca, Akađana, Kaldejaca i drugih naroda koji su u različitim razdobljima, počev od trećeg milenijuma prije naše ere (sumerski preiod) pa sve do perioda od 3. v. pr. n.e. (period grčke vladavine), boravili na djelovima tog područja. Izraz „vavilonski” često se koristi kao sinonim za „mesopotamski”.Izvori informacija koji se odnose na nivo mesopotamske matematike vrlo su obimni. Mnogo stotina tablica pisanih klinastim pismom2 bave se problemima koje bismo danas zvali algebarskim, ili se bave geometrijskim odnosima.

Kako su računali?U Mesopotamiji je razvijen brojevni sistem sa osnovom 60 Još uvijek se ne može sa sigurnošću reći zašto baš osnova 60, ali je evidentno da ima nekoliko prednosti. Prvo: kod brojnih sistema sa većom osnovom , zapis broja je kraći. Druga, važnija prednost, je što broj 60 ima čak deset pravih djelioca – 2,3,4,5,6,10,12,15,20 i 30, zbog čega mnogo više brojeva ima konačnu reprezentaciju nego u našem dekadnom sistemu. Treća prednost je stara podjela godine na 360 dana (12 mjeseci po 30 dana). Tom broju dodavalo se još 5 „suvišnih” dana svake godine. Vavilonci nijesu imali pogodne metode za vršenje raznih, pa i najprostijih aritmetičkih operacija. Iz tog razloga u vavilonskoj matematici, „caruju” razne vrste tablica. Tako imamo, na primjer:

Tablice recipročnih brojeva 1m

i njihovih (tačnih ili

približnih) vrijednosti u sistemu sa osnovom 60. Pomoću njih su Vavilonci, između ostalog, dijelili brojeve i to tako

što su ih svodili na množenje prema formuli ab ab

=

1 .

Arheolozi su u posjedu tablica sa recipročnim vrijednostima brojeva sve do nekoliko milijardi; Tablice množenja;

1 Mesopotamija – grčki doslovno znači: „(zemlja) između (dvije) rijeke”

2 Sumeri su urezivanjem tragova na vlažnim glinenim tablicama, koje su se potom sušile na suncu, stvorili prvo pismo. Kako ti zarezi podsjećaju na crtež klina, to je pismo dobilo ime – klinasto pismo.

Tablice kvadrata i kubova, koje za broj n kazuju koliko je n² i n³. Pronađene su tablice još iz 2000. godine prije n.e. koje sadrže kvadrate brojeva od 1 do 59 (to su sve cifre u sistemu sa osnovom 60).Tablice kvadratnih korijena pa, štaviše, i kubnih korijena izraženih približno pomoću šezdesetina. Na primjer, 2 su zapisivali kao „1 24 51 10” . Naravno u pitanju su odgovarajuće oznake na klinastom pismu za odgovarajuće cifre u sistemu sa osnovom 60. Znači, broj je

1 2460

5160

10602 3+ + + , što je približno 1,414213... i razlikuje

se od 2 tek na sedmom decimalnom mjestu Smatra se da su poznavali čuveni a i danas standardni metod za izračunavanje kvadratnog korijena „cifru po cifru”.

Tablice eksponencijalnih brojeva, tj. za izvjesne vrijednosti osnove c, a za razne izložioce n koji se uzimaju redom, daju se vrijedosti stepena cn. Na primjer, 2n za n = 1, 2, 3, 4, itd. jednako je 2, 4, 8, 16, itd., a pri tome nalazimo i osnove koje nijesu cijeli brojevi kao, npr. 4,5.Dakle, još jedna opšta karakteristika stare mesopotamske matematike jeste izrada tablica, kojom su se Vavilonci služili kao što se mi danas služimo npr. logaritamskim tablicama

Mesopotamija

Feljton

ISTORIJA MATEMATIKE - I DIO

Da li znate zašto se krug dijeli na 360°?Negdje oko 2400. pr. n.e., drevni Sumeri su primijetili vidljivu kružnu putanju Sunčevog godišnjeg puta preko neba i znali su da je potrebno oko 360 dana da obavi svoj put. Prema tome, njima je bilo razumno da podijele kružni put na 360 stepeni da bi mogli da prate dnevno kretanje Sunca. To je vodilo ka današnjem krugu od 360 stepeni. Kako stepen dijelimo na 60 minuta, a minute na 60 sekundi, to je i danas prisutan brojni sistem sa osnovom 60 koji su koristili stari Vavilonci.

Pokušavalo se već dosta davno da se ova starovavi-lonska podjela kruga zamijeni dekadnom, pri čemu se krug dijelio na 400 jednakih djelova, gradi, svaki grad na 10 decigradi, decigrad na 10 centrigradi, itd. Međutim, praktičnije je ostati pri podjeli kruga na 360° jednostavno zato što broj 360 ima 22 činioca koji obezbjeđuju lakše dijeljenje kruga.

Page 13: Zanimljiva matematika

11

Vrhunac drevne sumersko-vavilonske aritmetike je, bez sumnje, rješa-vanje algebarskih jednačina prvog, drugog pa čak i trećeg stepena, kao i jednačina sa jednom i sa dvije nepoznate. S obzirom da nijesu ra-spo lagali simbolima za nepoznate i poznate brojeve, kao ni znacima računskih radnji, njihova algebra je bila retorička tj. sve se opisivalo riječima. Jedno od glavnih hendikepa vavilonske numeričke matematike bilo je dugotrajno odsustvo nule, sve do 3. vijeka pr. n.e. kada je uveden simbol za upražnjeno mjesto u zapisu broja ali je i danas njegova funkcija ostala nejasna

GeometrijaGeometrija Mesopotamije vjerovatno je već oko 2000. godine prije n.e. raspolagala pravilima za izračunavanje površine pravougaonika, pravouglog i jednakokrakog (a možda i opšteg) trougla kao i za obim pravouglog paralelopipeda i nekih posebnih pravih prizmi. Obim kruga i obim kružnog valjka računalo se, u starije vrijeme, s aproksimaciojom 3 za broj π, a kasnije su Vavilonci upotrebljavali i mnogo bolju aproksimaciju π = 3 1/8 = 3,125 (greška samo oko 0,5%).Za Pitagorinu teoremu su znali i to u njenom opštem obliku, ali najvjerovatnije ne i dokaz. Glinena tablica koja datira iz perioda između 1900. i 1600. god. pr. n.e. daje tabelu Pitagorinih trojki cijelih brojeva koji zadovoljavaju formulu: a² + b² = c². S pravom se pretpostavlja da je služila za rješavanje pravouglog trougla. Ova tablica se u istoriji smatra prvim dokumentom iz teorije brojeva. U Britanskom muzeju se čuva tablica koja sadrži zadatak koji se rješava upotrebom Pitagorine teoreme. Tekst glasi:„4 je dužina a 5 je dijagonala. Kolika je širina? Nju ne znamo (ona je nepoznata). 4 puta 4 je 16. 5 puta 5 je 25. Ako oduzmeš 16 od 25, ostaće 9. Koliko treba da uzmeš da bi ga pomnoživši dobio 9? 3 puta 3 je 9. 3 je širina.”Važno je napomenuti da se ni u jednom vavilonskom dokumentu ne pominje dokaz kao matematički pojam, niti se može naslutiti da su ikada osjetili potrebu da nešto dokažu. Matematiku su, uglavnom, shvatali kao praktičnu vještinu.

AstronomijaGeneracije i generacije vavilonskih astronoma, uglavnom iz redova sveštenika, osmatrale su nebo i bilježile rezultate svojih osmatranja. Stotine, a prema nekim izvorima, i hiljade godina osmatranja stvorile su ogromnu bazu podataka. Uporedo sa time, samo uređenje posmatranja dalo je veliki doprinos matematici. Vavilonski astronomi razvili su sferni koordinatni sistem, praktično isti kakav je u upotrebi u savremenoj astronomiji. Osnovni krug nebeske sfere činila je prividna putanja Sunca tokom godine. Kako je položaj svakog nebeskog tijela određen „geografskom” širinom i dužinom u odnosu na taj krug, koordinate tijela tokom vremena su bilježene u čitave biblioteke glinenih tablica. Iako je bilježenje čisto aritmetički posao, same tablice se mogu shvatiti, ako se položaji zapisuju dovoljno dugo, kao prve tablice sinusoida. Razvoj trigonometrije i odgovarajućih matematičkih vještina povezanih sa njom, u kasnijem helenističkom periodu, skoro je u potpunosti pokrenut zahvaljujući vavilonskim astronomskim osmatranjima. Danas, kada na Grke gledamo kao na svoje intelektualne roditelje, treba napomenuti da je niz značajnih grčkih intelektualaca, među kojima i sam Tales, svoja osnovna matematička znanja donosio sa putovanja u Vavilon. Dakle, uticaj Vavilonjana sa svojim korpusom matematičkih i astronomskih znanja i danas se prepoznaje u našem intelektualnom naslijeđu.

Milena Radoman, II-2Literatura:1. http://www.matf.bg.ac.yu/~zlucic/opstamat.pdf;2. http://ahyco.ffri.hr/seminari2007/povijestmatematike.htm;3 Pregled istorije i filozofije matematike, Milan Božić, Zavod za udžbenike i nastavna sredstva, Beograd, 2002.

Feljton

Page 14: Zanimljiva matematika

12

Matematička dimenzija piramide

Nemojte da vas ovaj naslov navede da odustanete prije nego pročitate tekst. Nećemo samo navesti dimenzije, uporedićemo ih sa odnosima u matematici, a onda ćete proviriti u tajne ove čudesne građevine. Bićete iznenađeni, ali spremni za nastavak...

Nauka ni danas ne može da odgovori na pitanje kako su te drevne civilizacije znale za geometrijske odnose kasnije nazvane π, φ, Pitagorina teorema. Sve te odnose vidimo skladno ugrađene u dimenzijama piramide. Ono što zadivljuje su mjere koje pokazuju nevjerovatnu preciznost.

Iako (pretpostavimo) nijesu poznavali Pitagorin trougao, ∆upravo takav trugao čini polovinu površi koji se dobija ako se piramida presiječe vertikalnom ravni koja prolazi kroz apoteme naspramnih strana. Stranice dobijenog trougla stoje u razmjeri 3 : 4 : 5!

Odnos obima osnove i dvostruke visine odgovara odnosu ∆obima kruga i njegovog prečnika, odnosno broju π.

Apotema i polovina osnove su u odnosu zlatnog presjeka, tj. njihov odnos je jednak broju φ. ∆

Ugao između bočne strane i osnove približno je jednak broju sedmica u godini, odnosno nagibu fosfatne ∆kiseline u DNK – 52º.

Stranice Velike piramide u perfektnoj su liniji sa stranama svijeta. Preciznost je zadivljujuća ∆– 99,99%. Piramida predstavlja najpreciznije orijentisanu građevinu na svijetu, preciznije

orijentisanu i od Meridijanske zgrade u sklopu Griničnog opservatorijuma.

Interesantno je da dužina strane kvadratne osnove daje 365,342 egipatskih lakata. Broj ∆je identičan broju dana tropske sunčane godine!

Dimenzije piramide i planeta Zemlja

Možda ste negdje čuli ili pročitali o dimenzijama naše palnete skladno ugrađenim u Veliku piramidu. Ako nijeste, imate priliku da saznate koliko su graditelji ovog drevnog čuda bili svjesni dimenzija planete Zemlje:

Trebalo je 22 vijeka da se izračuna udaljenost između Zemlje i Sunca (149 400 000 km). To bismo, približno, znali ∆mnogo ranije da se neko sjetio da sa 1 000 000 000 pomnoži visinu Kepsove piramide u Gizi.

Zapremina piramide pomnožena sa vrijednošću gustine blokova od kojih je izgrađena daje srednju vrijednost gustine ∆Zemljine kugle!

Piramida u Gizigrobnica ili kosmičko savršenstvo

Znate li da nabrojite svjetska čuda starog svijeta? Jedno među njima je posebno.To je jedino očuvano drevno svjetsko čudo. Da, pogodili ste.

To je Keopsova piramida.

Istorija i nastanakUčili smo da su piramide stvorene oko 2560. g.p.n.e. Pretpostavlja se da su ih gradili stari Egipćani i da su služile kao grobnice egipatskim faraonima. Da li ste kada posumnjali u to? Sigurno jeste, baš kao i veliki broj učenih ljudi koji smatraju da ne možemo sigurno reći ni ko ih je sagradio ni kad su sagrađene. O njihovom nastanku stvorene su legende – od one da ih je izgradila civilizacija koja je postojala prije naše, pa do toga da su ih gradili vanzemaljci. Kad govorimo o piramidama, ništa nije nemoguće. Zato se nećete iznenaditi kad kažemo da su to najsavršenije građevine na svijetu, ali i to da je onaj ko ih je gradio obratio pažnju na matematiku u svakom dijelu piramide. Odlično je uklopio geometrijske odnose. Zato ona traje do danas, i trajaće još dugo...

5 4

3

Gama istražuje

Page 15: Zanimljiva matematika

13

Odnos zapremina piramide i naše planete je tačno 1:10 ∆ 15!

Egipatski lakat bio je podijeljen na 25 palaca, a svaki palac ima 25.4265 milimetara. Pomnožimo li to sa 100 milijardi, ∆dobijamo približno tačnu vrijednost putanje koju u svom okretanju oko Sunca prođe Zemlja u jednom danu!

Obim osnove Velike piramide i njena visina stoje prema ekvatorijalnom obimu Zemlje i njenim polarnim radijusom ∆u razmjeri 1:43 200. Prema tome, Velika piramida bi mogla da bude precizno umanjen model sjeverne Zemljine hemisfere! I ne samo to: „specijalni broj” iz razmjere izveden je iz jednog od ključnih planetarnih kretanja Zemlje, tj. stope njene aksijalne precesije1

Mjerenja novijeg vremena pokazala su da je Keopsova piramida smještena tačno u centar kopnene mase Zemlje, na ∆meridijan koji presijeca najviše kopna i najmanje mora. Uzevši u obzir ukupnu površinu svih kontinenata i površinu koju zauzima piramida, te stoga i potencijalno mogućih 3 milijarde mjesta na kojima je mogla biti izgrađena, šanse za tu „slučajnost” su 1: 3 000 000 000!

Kraljeva odaja i sarkofag u matematičkim odnosima

Da ipak pretpostavimo da su piramide građene kao grobnice i zavirimo u kraljevu odaju. Tamo ćemo naići na nevjerovatne geometrijske odnose

Kraljeva odaja formira savršen pravougaonik koji izražava i pokazuje primjer „zlatnog presjeka”. Interesantno je i da je smještena tačno na sredini vertikalnog presjeka piramide, u sloj čija je površina jednaka polovini površine osno ve piramide, gdje je dijagonala jednaka dužini osnove i gdje je širina fasade jednaka polovini dijagonale osnove. Visina kraljeve odaje je tačno polovina dijagonale njenog poda! Nakon svega ovoga neće vas ni iznenaditi činjenica da masa

1 precesija - kretanje koje izvodi rotirajuće tijelo zbog djelovanja sile koja nastoji da promijeni smjer osovine rotiranja tijela u prostoru. Zbog precesijskog kretanja Zemlje, njena osovina opisuje jednom u 26 000 godina potpuni omotač kupe, pri čemu nebeski pol opiše kružnicu među zvijezdama. Zemljina precesija nastaje djelovanjem Sunca i Mjeseca na Zemlju, koja nije pravilna kugla, već geoid.

SJEVERNAAMERIKA

JUŽNAAMERIKA

AFRIKA

AUSTRALIJA

A Z I J AEVROPA

Velike piramide

Velike piramide

geografska dužina

geografska širina

Gama istražuje

Geodetska lokacija Velike piramide u Gizi na 30 stepeni sjeverne geografske širine (jedna trećina puta između ekvatora i Sjevernog pola) i u centru svjetske kopnene mase pogodne za život.

Page 16: Zanimljiva matematika

14

Keopsovog sarkofaga iznosi tačno 1015 puta manje od mase Zemlje! Dodajmo i to da je kraljeva odaja smještena na trećini visine piramide, a prisjetimo se da je Keopsova piramida smještena skoro tačno na trećini puta između Sjevern-nog pola i Ekvatora.

Piramida i astronomija

Pitate se kako je moguća veza ove vrste? Piramide ne prestaju da iznenađuju:

Djelo Roberta Bovala „The Orion Mystery” detaljno obrazlaže da raspored tri piramide u Gizi odslikava poziciju tri glavne zvijezde Orionovog sazvježđa.

Pravo iznenađenje otkriveno Bovalovim astronomskim proračunom bilo je sljedeće: Uprkos činjenici da neki aspekti Velike piramide astronomski imaju veze s dobom piramida, spomenici u Gizi su, kao cjelina, raspoređeni tako da daju sliku neba (čiji se izgled tokom vijekova mijenja), ali ne onakvu kakva je bila 2500. godine stare ere, u razdoblju četvrte dinastije, već onakvu kakva je bila, samo tada, negdje oko 10450. godine stare ere! Da li nam to govori da su piramide skoro 8 000 godina starije nego što mislimo i da je 10450. g.p.n.e. postojala veoma napredna civilizacija za koju današnja istoriografija ne zna? Šta kažete na ovo: oko 10450 g.p.n.e. počela je „era Lava”, a svi znamo kako izgleda Sfinga!

Gama istražuje

stijena

kraljičina dvorana

prolaz nadolje

220 lakata380 stopa

ulaz

lažna grobnica

440 lakata670 stopa

280

laka

ta49

2 st

ope

prolaznagore

ventilacija

kraljeva dvoranaventi-lacija

velika dvorana

apotema: 356 lakata

613.78 stopa

½ osnove: 220 lakata

380 stopa

Zlatni presjek Keopsove piramide sveden u razmjere:pola osnove: visina = dijatesaron-kvarta

u laktovima: 220 : 280 = 0,786u stopama: 380 : 482 = 0,788

pola osnove: apotema = dijapenta-kvintau laktovima: 220 : 356 = 0,618

u stopama: 380 : 613,78 = 0,618pola osnove: cijela osnova = dijapazon-oktava

u laktovima: 220 : 440u stopama: 380 : 760

= 0,5

= φ

= 0,75

Page 17: Zanimljiva matematika

15

Keopsova piramida i molekul DNKZnamo šta je DNK: molekul koji predstavlja život svakog od nas. Piramida se može dovesti u vezu i sa ovim „izvorom” života:

Prosječni ugao pod kojim se vezuju azotaste baze bio bi ∆51°45’ , a kod Keopsove piramide ugao nagiba bočnih strana je 51°51’!

Odnos dužine veze citozin-guanin (1,08 nanometara) ∆prema dužini timin-adenin (1,11 nanometara), jednak je 0,973, a bočna ivica Keopsove piramide odnosi se prema dužini strane osnove kao 0,946!

Odnos prečnika molekula DNK (2 nanometara) prema ∆dužini koraka spirale (3,4 nanometara) bio bi 0,588. Ako apotemu bočne strane Keopsove piramide podijelimo dijagonalom osnove dobijamo 0,568.

Ugao uspona spirale DNK iznosi 26°, a ugao nagiba ∆glavne galerije na Keopsovoj piramidi - takođe 26°!

U korak spirale DNK (34 angstrema) smješteno je 10 ∆nukleotidnih parova, čije je međusobno rastojanje 3,4 angstrema ili 1/10 koraka. Keopsova piramida ni tu ne zastaje! Njena visina je oko 148m, a ulaz u piramidu nalazi se na visini od 14m - približno 1/10.

Ovo je samo djelić tajne neprevaziđenih građevina, naizgled jednostavnih, ali u stvari neistraženih i punih misterija. Ukazali smo na veze piramide s našim životom, našom planetom. Zato možmo reći da piramida sadrži proporcije Sunčevog sistema kao makrokosmosa i brojeve čovjeka kao mikrokosmosa. Piramida je riznica proporcija, brojki, tajni. Čovjek se plaši vremena a, kako kaže stara arapska izreka, „Vrijeme se plaši piramida”. Ne znam da li iko može biti ravnodušan i ne zapitati se šta su, u stvari, piramide, ko ih je stvorio, kako i zašto je taj neko tako dobro poznavao geometriju i odnose u njoj koji su otkriveni hiljadama godina poslije njihovog nastanka...

Milan LakićevićIII-l

Literatura:1. Dragan Vićanović, Tajne piramida, PRJ Zemun, 1994.2. Grejem Henkok, Tragovi bogova, Mono & Manana, Moć knjige, 2004.3. Grejem Henkok, Robert Boval, Čuvari čovječanstva, Mono & Manana, Moć knjige, Beograd 2005.4. Đerđ Doci, Moć proporcija, Stylos, Novi Sad, 2005.

M

i

l

a

n

Lakićević

Hieroglife napisalaVesna Kovačević

profesor

Gama istražuje

Page 18: Zanimljiva matematika

16

Nejednakost između aritmetičke i geometrijske sredine (u daljem tekstu AG nejednakost) jedna je od najpoznatijih algebarskih nejednakosti. S obzirom na to da srednjoškolci, uglavnom, nijesu u prilici da se na redovnoj nastavi matematike upoznaju sa ovom nejednakošću, evo prilike da se kroz časopis upoznate sa ovim veoma praktičnim „alatom”.

Definicija aritmetičke i geometrijske sredineNeka su a1, a2, ... an proizvoljni pozitivni realni brojevi i n∈N

Aritmetička sredina brojeva a1, a2, ... an definisana je izrazom a a an

n1 2+ + +... .

Geometrijska sredina brojeva a1, a2, ..., an definisana je izrazom a a ann1 2... .

NejednakostVezu između aritmetičke i geometrijske sredine daje sljedeća teorema:

Neka su a1, a2, ..., an proizvoljni pozitivni realni brojevi. Tada važi da je:

a a a

na a a nn

nn1 2

1 2+ + + ≥ ∈... ... , . Ν

Jednakost se postiže ako i samo ako je a1= a2= = an

Zavirimo li u istoriju, saznaćemo da su za AG nejednakost znali još i pitagorejci, ali se njen prvi dokaz (za n = 2) pripisuje Euklidu.

Dokazi AG nejednakostiDanas se u literaturi mogu naći desetine različitih dokaza ove nejednakosti. Opšti slučaj se dokazuje pomoću matematičke indukcije, a mi ćemo ovdje razmotriti tri dokaza teoreme za n = 2, i to: algebarski, jedan geometrijski i jedan analitički

dokaz, tj. razmatraćemo dokaz nejednakosti a b ab+ ≥2

, koja važi za svaka dva pozitivna realna broja a i b. Jednakost se dostiže ako i samo ako je a = b

Algebarski dokaz:

a b a ab b a b ab a b ab−( ) ≥ ⇔ − + ≥ ⇔ + ≥ ⇔ + ≥2

0 2 0 22

,

pri čemu su a i b pozitivni realni brojevi. Jednakost se dostiže ako i samo ako je a = b

Geometrijski dokaz:Veći kvadrat ima stranicu dužine a + b i njegova je površina očigledno veća od površine četiri pravougaonika sa stranicama a i b. Dakle, imamo:

a b ab a b ab a b ab+( ) ≥ ⇒ + ≥ ⇒ + ≥2 4 22

, što je trebalo dokazati.

Nejednakost između aritmetičke i geometrijske sredine

b a

a

a

b

b

a

(b–a)²

b

Saznajmo nešto više

Page 19: Zanimljiva matematika

17

Jednakost se postiže ako i samo ako je površina velikog kvadrata jednaka površini četiri pravougaonika, odnosno ako i samo ako kvadrat u sredini figure iščezava, a to se događa ako i samo ako je b – a = 0, tj. a = b

Analitički dokaz:

Funkcija f x ex( ) = je konveksna, što geometrijski znači da je grafik funkcije između dvije

tačke na grafiku uvijek ispod tetive koja spaja te dvije tačke. Na grafiku eksponencijalne funkcije izaberimo dvije tačke sa koordinatama x ex1

1, ( ) i x ex22, ( ) i uvedimo oznake

a ex= 1 i b ex= 2 .

Prava kroz izabrane tačke ima jednačinu:

y b b ax x

x x− = −−

−( )2 1

2

pa tačka na pravoj sa apsicom x x1 2

2+ ima ordinatu a b+

2.

S druge strane tačka sa istom apscisom, ali na grafiku eksponencijalne funkcije ima ordinatu

e e e a b abx x

x x1 2

1 2212

12

+

= ⋅( ) = ⋅( ) =

Kako se tačka na grafiku funkcije nalazi ispod tačke na tetivi, imamo da je a b ab+ ≥2

, tj.

slijedi AG nejednakost

Primjene AG nejednakosti

AG nejednakost je snažno sredstvo za dokazivanje raznih složenijih nejednakosti kao i zadataka drugog tipa pa se u literaturi mogu naći brojni zadaci koji se svode na njenu primjenu. Evo nekoliko primjera.

1. Dokazati da je za svako a > 1 i b > 1; ab

ba

2 2

1 18

−+

−≥

Rješenje:

Na osnovu AG nejednakosti imamo da je:

ab

ba

ab

ba

aba b

aa

bb

2 2 2 2

1 12

1 12

1 12

1 1−+

−≥

−⋅

−=

− ⋅ −=

−⋅

− (1)

Dokažimo još da je xx −

≥1

2 , za x > 1

x x x x x xx

−( ) ≥ ≥ − ≥ −( )−

≥2 0 4 4 4 11

22 2 2; ; ;

Ako u (1) primijenimo dokazanu nejednakost, dobijamo: ab

ba

2 2

1 12 2 2 8

−+

−≥ ⋅ ⋅ = , čime je dokaz završen.

2. Neka je n ∈ N Dokazati da je 1 12

1 1+ + + + > ⋅ +... .n

n n nn

Rješenje:

1 12

1 1 1 12

1 1 1 2 32

1+ + + + = +( ) + +

+ + +

= + + + +... ... ...

nn

nnnn

AGn n

nn nn n> ⋅ ⋅ ⋅ ⋅ + = ⋅ +2 3

21 1...

2a b+

ab

a

b

1 2

2x x+

xy e=

1x 2x

Saznajmo nešto više

Page 20: Zanimljiva matematika

18

3. U jednačini x pxp

p R22

12

0 0+ − = ∈, \ , važi x x14

24 2 2+ ≥ + gdje su x1 i x2 rješenja jednačine Dokazati

Rješenje:

Na osnovu Vietovih pravila imamo da je x x p1 2+ = − i x xp1 2 2

12

= −

x x x x x x x x x x x x pp1

424

12

22 2

12

22

1 22

1 2

2

12

22 22 2 2 1+ = +( ) − = +( ) −( ) − = + 22

2

4

44

44

44

2 14

2 12

2 12

2 1 2 2 1

− ⋅ =

= + + = + +

≥ + ⋅ =

p

pp

pp

AGp

p22 2+

4. Naći najmanju vrijednost izraza sxyz

yzx

xzy

= + + ako su x, y i z pozitivni brojevi čija je suma kvadrata jednaka 1.

Rješenje:

s x yz

y zx

x zy

x y z

x yz

x zy

22 2

2

2 2

2

2 2

22 2 2

2 2

2

2 2

2

2

12

= + + + + +( ) =

= +

++ +

+ +

+ ≥

≥ ⋅

12

12

22 2

2

2 2

2

2 2

2

2 2

2

2 2

2

x zy

y zx

y zx

x yz

AG

x yz

xx zy

x zy

y zx

y zx

x yz

x y z2 2

2

2 2

2

2 2

2

2 2

2

2 2

22 2 22 2 3+ ⋅ + ⋅ + = + + + =

Kako smo dobili da je s2 3≥ , slijedi da je s ≥ 3 tj. najmanja vrijednost datog izraza je 3.

Jednakost se dostiže za x y z= = = 33.

5. Riješiti sistem jednačina 2 2 8 22 2x y x y x y+ ++ = + =, .

Rješenje:

Na osnovu AG nejednakosti imamo 2 2 2 22 2 2 2x y x y x x y y+ + + + ++ ≥ pa iz prve jednakosti sistema dobijamo

x x y y2 2 4+ + + ≤ (1)

Kako je 22

x y x y+( ) ≥ +( ) , iz druge jednakosti sistema slijedi

x y+ ≥ 2 (2)

Iz 2 2 2 2x y x y+( ) ≥ +( ) i (2) slijedi

x y2 2 2+ ≥ (3)

Iz (1), (2) i (3) slijedi x y+ = 2 i x y2 2 2+ = , odakle rješenjem sistema jedne linearne i jedne kvadratne jednačine

dobijamo x = y = 1.

6. Ako za stranice trougla ∆ABC važi a b c abc P+ +( ) =16 2 , gdje je P njegova površina, dokazati da je trougao

jednakostraničan.

Rješenje:Posmatrajmo Heronov obrazac, P s s a s b s c= −( ) −( ) −( ), gdje je s poluobim trougla.

Neka je x s a y s b z s c x y z= − = − = − >( ), , , , , . 0

Saznajmo nešto više

Page 21: Zanimljiva matematika

19

Tada je s x y z P x y z xyz= + + = + +( ), ,

pa na osnovu uslova zadatka važi: 2 16x y z y z z x x y x y z xyz+ +( ) +( ) +( ) +( ) = + +( ) , tj

x y y z x z xyz+( ) +( ) +( ) = 8 (1)

Na osnovu AG nejednakosti važi:

x y xy y z yz z x zx+ ≥ + ≥ + ≥2 2 2, , (2)

odakle slijedi da je x y y z z x xyz+( ) +( ) +( ) ≥ 8 . Na osnovu (1) mora biti x y xy y z yz+ = + =2 2, i

z x zx+ = 2 . U (2) jednakosti važe za x = y, y = z i x = z Dakle x = y = z ⇒ s − a = s − b = s − c ⇒ a = b = c.

7. Niz (xn) zadovoljava relacije 0 1 1 141< < −( ) ≥+x x xn n n, za svako n Dokazati da je lim .

n nx→∞= 12

Rješenje:

Iz 1 1 2 1= −( ) + ≥ −( )x xAG

x xn n n n i 1 14

2 1 11 1−( ) ≥ ⇒ −( ) ≥+ +x x x xn n n n slijedi da je x xn n< +1 pa je niz (xn)

monotono rastući. Kako je niz i ograničen, to on konvergira. Neka je lim .n nx x→∞

= Tada je x x x1 14

12

02

−( ) ≥ ⇔ −

≤ ,

pa je x xn n= =→∞

12

12

, lim . tj.

A sada ostavljam čitaocu da primjenom AG nejednakosti riješi zadatke koji slijede. Rješenja zadataka biće objavljena u sljedećem broju časopisa

Mirjana Pješčić

Literatura:

1. http://e.math.hr/agnejednakost/index.html

2. Dr R. Šćepanović, dr M. Anđić, Zbirka riješenih zadataka iz matematike za takmičenja učenika srednjih škola, Laboratorija za matematiku i računarstvo, Podgorica, 2005.

3. V. Dragović, P. Mladenović, S. Ognjanović, Pripremni zadaci za matematička takmičenja za učenike srednjih škola, Društvo matematičara, Beograd, 1999.

4. M. Obradović, D. Georgijević, Matematiskop 4, Odabrani zadaci za drugi razred srednjih škola, Matematiskop, Beograd, 1995.

Zadaci:

1. Brojevi a, b, c i d su pozitivni i njihov proizvod je jednak 1. Dokazati da je:

a b c d ab ac ad bc bd cd2 2 2 2 10+ + + + + + + + + ≥ .

2. Dokazati da je log log log log , .4 5 6 75 6 7 8 4 4+ + + >

3. Neka su a i b dužine kateta, a c dužina hipotenuze pravouglog trougla i α i β veličine oštrih

uglova tog trougla. Dokazati nejednakost cos .222

2α β− ≥ abc

4. Dokazati da ako su α i β oštri uglovi i tg β = 3tg α, tada je β ≤ 30º + α.

5. Dužina najveće stranice jednakokrakog trapeza je 13, a obim 28. Naći stranice trapeza ako je njegova površina 27.

Saznajmo nešto više

Page 22: Zanimljiva matematika

20

Pojava infinitezimalnog računa predstavlja prelomni trenutak u istoriji čovjekovih stremljenja ka upoznavanju novog: njegovim stvaranjem ljudi su dobili moćan aparat za analizu najraznovrsnijih procesa, za dublje objašnjavanje fizičkih pojava i formiranje naučnog pogleda na svijet. Pri tome, primjena infinitezimalnog računa u kojoj se on učenicima i studentima koji „osjećaju” matematiku najjače predstavlja svojom jednostavnošću, ljepotom i logičnošću, svakako je određivanje maksimuma i minimuma.

Mogućnosti određivanja maksimuma i minimuma u srednjoj školi

Određivanje maksimuma i minimuma često je veoma važno u nastavi prirodno-matematičke grupe predmeta u srednjoj školi. Međutim, elementi infinitezimalnog računa, i prema prošlim i prema reformisanom nastavnom planu i programu opšte gimnazije, izučavaju se tek u četvrtom razredu.U ovom radu izlaže se jedan od elementarnih načina određivanja maksimuma i minimuma bez primjene infinitezimalnog računa. On, naravno, ima znatno užu primjenljivost u odnosu na infinitezimalni račun.

Nejednakost aritmetičke i geometrijske sredine

Nejednakost aritmetičke i geometrijske sredine N pozitivnih brojeva x1, x2,..., xn ima oblik:

x x xn

x x xnn

n1 21 2

+ + + ≥ ⋅ ⋅ ⋅ ⋅... , (*)

pri čemu znak jednakosti važi ako i samo ako je x1 = x2 = = xn Dokaz ove nejednakosti može da se nađe u gotovo svim standardnim udžbenicima matematičke analize. Kao ilustraciju njene direktne primjene u fizici uzimamo sljedeći primjer:

Primjer 1. Odrediti minimalnu vrijednost odnosa ekvivalentnog otpora N redno vezanih otpornika različitih otpora i ekvivalentnog otpora njihove paralelne veze.

Rješenje:Ekvivalentni otpor redno vezanih otpornika čiji su otpori r1, r2, ..., rn je R1 = r1 + r2 + ... + rn,

a ekvivalentni otpor njihove paralelne veze R2 može da se odredi iz jednačine 1 1 1 1

2 1 2R r r rn= + + +... .

Traženi odnos je

γ = = + + +( ) + + +

RR

r r rr r rn

n

1

21 2

1 2

1 1 1... ... .

Pošto iz (*) imamo da je

r r r nr r rn

n

n1 21 2

1 1 1+ + + ≥ + + + ... i

1 1 1 1 1 1

1 2 1 2r r rn

r r rn n

n+ + + ≥ ⋅ ⋅ ⋅ ⋅... ,

onda zaključujemo da je γ ≥ n 2 Prema tome, minimalna vrijednost traženog odnosa je γmin= n 2

Određivanja minimalne i maksimalne vrijednosti u zadacima iz fizike primjenom nejednakosti

aritmetičke i geometrijske sredine

Saznajmo nešto više

Page 23: Zanimljiva matematika

21

Posljedice nejednakosti i njena primjena

Iz nejednakosti (*) imamo:1. Najmanja vrijednost zbira n pozitivnih brojeva x1, x2, ..., xn je

x x x n x x xn MIN nn

1 2 1 2+ + +( ) = ⋅ ⋅ ⋅... ... (1)

za x1 = x2 = = xn 2. Najveća vrijednost proizvoda n pozitivnih brojeva x1, x2, ..., xn je

x x x x x xnn MAX

nn

1 21 2⋅ ⋅ ⋅ ⋅( ) = + + +

... (2)

za x1 = x2 = = xn Ako je n = 2, onda jednakosti (1) i (2), uzimajući da je x1 = A i x2 = B, glase:

A B A BMIN

+( ) = ⋅2 (3)

A B A BMAX

⋅( ) = +

2

2

(4)

za A = B Jednakosti (3) i (4) lako se dokazuju. Pođimo od tvrđenja

A B−( ) ≥2

0 , (5)

pri čemu znak jednakosti važi u slučaju kada je A = B. Iz (5), kvadriranjem, slijedi

A A B B− ⋅ + ≥2 0 , odnosno:

A B A B+ ≥ ⋅2 (6)Daljim transformacijama se dobija:

A B A B⋅ ≤ +

2

2

(7)

Nejednakosti (6) i (7) čine dokaz završenim.Dakle, maksimum ili minimum možemo odrediti pomoću neke od jednakosti: (1), (2), (3) ili (4).

Primjer 2. Otvoreni cilindrični sud visine H, do vrha je napunjen vodom. Na kojoj visini h od dna suda treba napraviti otvor da bi domet mlaza d bio maksimalan? (Površinu otvora smatrati zanemarljivo malom u odnosu na površinu poprečnog presjeka suda.)

Rješenje:Prema Toričelijevoj teoremi mlaz na izlazu ima brzinu

v g H h= −( )2

Ova brzina ima horizontalni pravac, tako da se dalje kretanje mlaza može posmatrati na sljedeći način:kako duž horizontalnog pravca ne djeluje nikakva -

sila, mlaz u tom pravcu prelazi put d = vt ;

kako mlaz duž vertikalnog pravca slobodno pada, to je pređeni put za isto vrijeme - h gt=2

2

Eliminisanjem vremena t iz ovih jednačina dobijamo da je d h H h= −( )2

Maksimalni domet, u oznaci dMAX , mlaz dostiže za odgovarajuću maksimalnu vrijednost izraza h(H – h).

Prema (4) imamo da je h H h HMAX

⋅ −( ) =

2

2

, za h H=2

Slijedi: maksimalni domet mlaza je dMAX= H, za h H=2

v

h

d

H

g

Saznajmo nešto više

Page 24: Zanimljiva matematika

22

Primjer 3. Na horizontalnoj podlozi nalaze se tri lopte masa m1, m2 i m3, respektivno. Centri lopti nalaze se na jednoj pravoj. Prvoj lopti saopštena je brzina v prema drugoj lopti. Poslije toga dolazi do apsolutno elastičnih centralnih sudara lopti. Kolika treba da bude masa druge lopte (m2) da bi, poslije sudara sa trećom loptom, brzina treće lopte bila maksimalna?

Rješenje:Iz zakona održanja impulsa i zakona održanja energije za centralni apsolutno elastični sudar prve i druge lopte:

m1v = m1v1 + m2v2 i m v m v m v1

21 1

21 2

2

2 2 2= +

dobija se da je brzina druge lopte poslije sudara

v m vm m2

1

1 2

2=+ ,

pri čemu je v1 brzina prve lopte poslije sudara sa drugom loptom.Za centralni apsolutno elastični sudar druge i treće lopte zakon održanja impulsa i zakon održanja energije možemo zapisati u obliku:

m2v2 = m2u2 + m3v3 i m v m u m v2 2

22 2

23 3

2

2 2 2= + ,

gdje su v2 i v3 brzine druge i treće lopte poslije njihovog sudara. Iz ovih jednačina je v m vm m3

2 2

2 3

2=+

Uvrštavanjem izraza za brzinu v2 imamo da je

v v m mm m m m

v f m m m31 2

1 2 2 31 2 34 4= ⋅

+ += ⋅

( )( )( , , )

Uvodeći oznake mm m

A1

1 2+= i m

m mB2

2 3+= ,

imamo da je f m m m A B( , , )1 2 3 = ⋅

Prema jednačini (4), brzina v3 će imati maksimalnu vrijednost za A = B, tj.m m m2 1 3= ⋅

To znači da će lopta imati najveću brzinu ako je vrijednost mase druge lopte geometrijska sredina vrijednosti masa prve i treće lopte.

Minimalna vrijednost funkcije y x px qx

( ) = + i primjeri primjene

Posmatrajmo funkciju

y x px qx

x( ) , .= + > 0

Uvođenjem oznaka px = A i qx

B= kraće imamo da je y=A+B

Na osnovu jednačine (3) je

y AB pqMIN = =2 2 za A = B, tj. x qp0 =

Razmotrimo sada dva primjera primjene ovog zaključka.

0 x0

ymin

y

x

Saznajmo nešto više

Page 25: Zanimljiva matematika

23

Primjer 4. Odrediti minimalnu vrijednost početne brzine tačkastog tijela da bi bez promjene smjera kretanja prešlo put s0 = 100m. Tačka se kreće ravnomjerno usporeno stalnim ubrzanjem a = 2 m/s2

Rješenje:

Položaj tačke određen je jednačinom x v t at= −0

2

2, odakle je v a t x

t0 2= +

Na osnovu prethodno izloženog određivanja minimuma funkcije y x px qx

( ) = + možemo uzeti da je

A at=2

i B xt

=

Tada je

v AB as m sMIN0 02 2 20= = = /

Primjer 5. Na raspolaganju imamo 24 akumulatora elektromotorne sile E = 2V i unutrašnjeg otpora r = 0,3Ω. Akumulatore vežemo redno (paralelno) u nekoliko jednakih segmenata, a zatim te segmente povežemo paralelno (redno). Kolika se maksimalna jačina struje može dobiti u spoljašnjem kolu ako na tako vezane akumulatore priključimo otpornik otpora R = 0,2Ω?

Rješenje:Neka je N ukupan broj akumulatora, a n broj akumulatora u jednom segmentu Tada je s = N/n broj segmenata u kolu. Posmatrajmo prvi slučaj: akumulatori su vezani redno u s segmenata, a segmenti su povezani paralelno. Tada je jačina struje

I nE

R rns

ERn

rnN

1 =+

=+

Jačina struje u ovom slučaju (I1) ima maksimalnu vrijednost ako je imenilac u gornjem razlomku minimalan.

Prema tome, problem se svodi na određivanje minimalne vrijednosti funkcije y n rNn R

n( ) = +

Na osnovu jednačine (3) imamo da je

y rNRMIN = 2

Prema tome

I ErRN

E NrRMAX1

22

= = = 20A.

U drugom slučaju, kada su akumulatori paralelno vezani u s redno vezanih segmenata, imamo da je

I sE

R s rn

NnE

R rNn

NE

Rn rNn

2

2

=+

=+

=+

Sličnim postupkom dobijamo da je I IMAX MAX1 2= = 20A

Prema tome, u ovom slučaju nije moguće dobiti jačinu struje veću od 20 A.

Umjesto zaključkaOvdje je naveden jedan od nekoliko načina određivanja maksimuma i minimuma bez korišćenja infinitezimalnog računa. Ovaj način je ilustrovan odgovarajućim primjerima iz srednjoškolske fizike.Primjena razmatranog načina/metode omogućava rješavanje navedenih i njima sličnih zadataka bez korišćenja matematičkog aparata tzv. više matematike. Zbog toga je korisno da ga učenik upozna, a naročito oni koji se pripremaju za državno ili međunarodno takmičenje iz fizike.

Radovan Ognjanović

Saznajmo nešto više

Page 26: Zanimljiva matematika

24

Ova rubrika će vas u svakom broju upoznavati sa paradoksima koji su dovodili do manjih ili većih kriza u matematici, kao i sa sofizmima koji imaju za cilj da nauče učenike da izbjegavaju lažne zaključke i nepravilnu upotrebu riječi.

Uređuje Mihailo Radević II10

PARADOKSI

Najšire shvaćeno, paradoks (ili antinomija) je rasuđivanje koje vodi u protivrječnost, iako izgleda da su polazne postavke tačne, a pravila rasuđivanja ispravna.

Kroz istoriju, paradoksi su znali da dovedu do razvoja određenih matematičkih disciplina. Tako je, na primjer, matematičar Bertrand Rasel1, početkom XX vijeka, u naivnoj teoriji skupova otkrio paradoks koji je doveo do naglog razvoja matematičke logike, što je svakako uticalo kako na modernu matematiku, tako i na logiku kao filosofsku disciplinu.

Raselov paradoks

U Kantorovoj naivnoj teoriji skupova, pod skupom se podrazumijeva kolekcija objekata koji imaju zajedničk svojstvo. Dugo se smatralo da je svaki skup definisan na ovaj način ispravno definisan skup. Tada je Rasel postavio pitanje koje je poništilo ovo uvjerenje. Pitanje je glasilo: „Da li skup svih skupova koji ne sadrže sebe, sadrži sebe?”.

Skup kao svoje elemente može da sadrži druge skupove. Na primjer, skup A = 1 2 3 1 2 5 6 7, , , , , , , sadrži tri skupa. Dalje, skup može da sadrži i sam sebe. Evo jedne takve definicije: „Skup svih skupova koji imaju više od dva elementa.” Pošto takvih skupova ima više od dva, i on sam će sadržati više od dva elementa, pa će biti dio samog sebe

Definišimo skup R2 kao „skup svih skupova koji ne sadrže sebe”. Postavimo pitanje, da li R sadrži sam sebe? Pretpo-stavimo da R ne sadrži sebe. Pošto je R skup svih skupova koji ne sadrže sebe, trebalo bi ipak da sadrži sam sebe, što je kontradikcija sa početnom pretpostavkom. To znači da moramo pretpostaviti suprotno – da R ipak sadrži sebe. Međutim, ni to nije moguće jer tada, po definiciji, R ne bi sadržao sebe.

1 Bertrand Arthur Wiliam Russel (1872 - 1970)2 običaj je u savremenoj matematici da se u čast Rasela taj „skup”

označava početnim slovom njegovog imena

Paradoksi i sofizmi

Matematika je uspravno zdanje koje bi se, za razliku od neke visoke zgrade, srušilo ako bi makar samo jedna matematička cigla bila rđava. Dopustite li da se u sistemu pojavi makar i najsitnija greška, više se ni u šta ne možete pouzdati. Prema jednoj logičkoj teoremi, ako u neki logički sistem uđe neka pogrešna teorema, onda biste pomoću nje mogli dokazati da je 1 jednako 2.

◊◊◊◊◊◊ ◊◊◊◊◊◊ ◊◊◊◊◊◊

Anegdota:

Prilikom jednog razgovora, neki skeptik pokušao je da nadmudri logičara Bertranda Rasela tako što bi osporio već pomenutu logičku teoremu.

„U redu”, kazao je skeptik, „ako dopustim da je jedan jednako dva, dokažite mi onda da ste vi papa.”

Priča se da je Rasel samo časak razmišljao, a onda uzvratio ovako:

„Papa i ja smo dva, prema tome – papa i ja smo jedan.”

Iz knjige „Euklidov prozor” Leonarda Mlodinova

Paradoksi i sofizmi

Page 27: Zanimljiva matematika

25

Radi boljeg shvatanja Raselovog paradoksa, formulišimo sljedeće paradokse koji su po strukturi isti kao i Raselov:

Paradoks brice

U puku je bio vojnik kojemu su naredili da brije sve one i samo one vojnike iz svog voda koji se ne briju sami. Postavlja se pitanje kako će se on odnositi prema samom sebi. Ako se bude brijao, treba ga uvrstiti u one koji se briju sami, a te vojnike on ne smije brijati. Ako se on sam ne bude brijao, treba ga priključiti vojnicima koji se sami ne briju, a tada je po naredbi dužan da se obrije.

Paradoks lažljivca - Epimenidesov paradoks (potiče još iz antičkog vremena)

Posmatrajmo rečenicu „Ja lažem.” Ako je ona istinita, onda ja lažem, pa je ta rečenica lažna. Ako je ona lažna, onda ja ne lažem, pa je ona istinita.

U principu, paradoksi se mogu javiti iz dva razloga: kao posljedice neodgovarajućih pretpostavki određene matematičke teorije i usljed nedovoljne razvijenosti naše intuicije. Raselov paradoks odnosi se na prvi slučaj i prevaziđen je uvođenjem nove aksiomatizacije teorije skupova1

SOFIZMI

Sofizam je namjerno nemoguć dokaz, lažan „dokaz” nekog tvrđenja. Pri tome se greška vrlo vješto i iskusno maskira u nekom dijelu dokazivanja.

Riješiti sofizam znači pronaći grešku u rasuđivanju na osnovu koje je zasnovan sofizam. Sofizmi imaju za cilj da pomažu kritičkom mišljenju jer ne treba se samo držati određene logičke sheme, određenih misaonih procesa, već i kritički osmisliti svaki korak rasuđivanja usvojenim principima matematičkog mišljenja i računske prakse.

Primjer:

Posmatrajmo niz jednakosti:

A BA BA A B A BA B AB BA B A B B A B

=== ⋅ =− = −−( ) +( ) = −(

;;

;

2 2

2

2 2 2

(jer je );

))+ =+ = ==

=

;;

;.

A B BB B B A BB B

(jer je );22 1

Gdje je greška?Odgovor je na strani sa rešenjima.

Literatura: 1. <http://www.im.ns.ac.yu/personal/madaraszr/Rozi-licno/skupovi.pdf> 2. Mićo Miletić, Paradoksi i sofizmi u matematici, Arhimedes, Beograd, 2004. 3. Milan Božić, Pregled istorije i filozofije matematike, Zavod za udžbenike i nastavna sredstva, Beograd, 2002. 4. N.J.Vilenkin, Priče o skupovima, Školska knjiga, Zagreb, 1975.

1 uveden je ZF sistem aksioma teorije skupova (nazvana po početnim slovima prezimena matematičara Zermela (Ernst Friedrich Ferdinand Zermelo 1871-1953) i Frenkela (Adolf Abraham Halevi Fraenkel, 1891-1965) koji su ga uveli.

Paradoksi i sofizmi

Page 28: Zanimljiva matematika

26

Pri računskim operacijama sa brojevima mogu se dobiti rezultati koji izazivaju i čuđenje i divljenje, poznati kao mirakuli1. Mirakuli koji slijede odnose se na račun sa brojem 9. Pogledajte:

1 ⋅ 9 + 2 = 11 12 ⋅ 9 + 3 = 111

123 ⋅ 9 + 4 = 1111 1234 ⋅ 9 + 5 = 11111

12345 ⋅ 9 + 6 = 111111 123456 ⋅ 9 + 7 = 1111111

1234567 ⋅ 9 + 8 = 11111111 12345678 ⋅ 9 + 9 = 111111111

123456789 ⋅ 9 + 10 = 1111111111

12345679 ⋅ (1 ⋅ 9) = 11111111112345679 ⋅ (2 ⋅ 9) = 22222222212345679 ⋅ (3 ⋅ 9) = 33333333312345679 ⋅ (4 ⋅ 9) = 44444444412345679 ⋅ (5 ⋅ 9) = 55555555512345679 ⋅ (6 ⋅ 9) = 66666666612345679 ⋅ (7 ⋅ 9) = 77777777712345679 ⋅ (8 ⋅ 9) = 88888888812345679 ⋅ (9 ⋅ 9) = 999999999

9 ⋅ 9 + 7 = 88 98 ⋅ 9 + 6 = 888 987 ⋅ 9 + 5 = 8888 9876 ⋅ 9 + 4 = 88888 98765 ⋅ 9 + 3 = 888888 987654 ⋅ 9 + 2 = 8888888 9876543 ⋅ 9 + 1 = 88888888 98765432 ⋅ 9 + 0 = 888888888

Teško je reći koji je mirakul impresivniji. Vjerovatno se pitate u čemu je „tajna”. Kako u matematici ništa nije slučajno, „tajna” je u sljedećem:

Prvi mirakul

Posmatrajmo izraz:10 2 10 3 10 10 1 1 1 2 91 2 3n n n n n n− − −+ ⋅ + ⋅ + +( ) ⋅ −( ) + +( ) ∈ ... , , ,..., .

Lako se provjerava da za n∈ 1 2 9, ,..., dobijamo redom prvi, drugi, ..., deveti red na lijevoj strani. Kada se izvrši množenje i slični sabirci saberu, dobija se

10 10 10 10 1 111 11 2

1

n n n

n

+ + + + + =− −

+

... ...

što predstavlja izraz na desnoj strani.

Drugi mirakul

Prva jednakost se direktno provjerava, a ostale slijede iz nje, s obzirom na to da je12345679 9 111111111⋅ ⋅ = ⋅ =( )k k kkkkkkkkk za k = 2 3 9, ,..., .

Treći mirakul

Posmatrajmo izraz:9 10 8 10 7 10 10 10 1 8 1 21 2 3⋅ + ⋅ + ⋅ + + −( ) ⋅ − + − ∈− − −n n n n n n... ( ) ( ) ( ), , ,. ...,8

Slično kao u prethodnom primjeru prvi, drugi,..., osmi red na lijevoj strani dobijaju se redom za n = 1, 2, 3, ..., 8. Nakon izvršenih operacija množenja i sabiranja, dobijamo

9 10 10 10 2 9 10 10 10 2 9 10 111 2 1 2⋅ − − − − = ⋅ − + + +( ) = ⋅ −− − − −n n n n n n n... ... 11 12 888 81 1

... ... ,n n− +

=

što predstavlja izraz na desnoj strani.

1 od engleske riječi miracle što znači čudo

MirakuliIz referata „Magije brojeva” Lilande Ćorović i Jelene Džaković

I10 (školska 2006/07)

Zanimljiva matematika - Brojevi

Page 29: Zanimljiva matematika

27

1. Od 29 štapića, iste dužine, sastavljena je netačna jednakost. Premjestite samo jedan štapić tako da se dobije tačna jednakost.

2. Ova jednačina rimskih brojeva sasvim je tačna, pa ne morate da pomijerate ni jednu šibicu. Kako to?

3. Petnaest šibica raspoređeno je tako da formira pet kvadrata. Oduzimanjem tri šibice treba dobiti tri kvadrata.

4. Datih devet šibica čine tri jednakostranična trougla. Premjestiti samo dvije šibice tako da se dobiju dva jenakostranična trougla.

PripremileIvana Popović, IIIlJovana Popović, I12

Zanimljiva matematika

Šibice

Page 30: Zanimljiva matematika

28

Ljepljivi smajlijiMilan, Božidar i Ivana bacaju ljepljive smajlije na tablu. Svaki smajli koji bace osvaja 10, 13 ili 17 poena. Tabela pokazuje koliko je kojih smajlija bačeno na tablu, ali ne znamo koliko je kojih bacila svaka osoba. Znate ovo: Ivana ima 56 poena. Broj Božidarovih poena je dvocifren broj sa dvije iste cifre. Božidar ima tri poena više od Milana. Koliko je kojih smajlija svaka osoba bacila na tablu?

10 poena 13 poena 17 poena

Krivudavi zatvorKako ćete lako ustanoviti da li se mrav nalazi unutar ili izvan krivudavog zatvora a da ne pratite trag do izlaza ?

Elegantne elipseNacrtati četiri prave linije koje će podijeliti ravan na takav način da svaka elipsa bude smještena u odvojenu oblast.

Tri kvadrataU kvadratima se nalaze brojevi od 1 do 12. Zbir brojeva prvog kvadrata je 24, drugog 34 a trećeg 20. Dopunite brojeve koji nedostaju.

1 ? 8 ? 6 ?

? 11 ? 5 ? 3

Tajanstveni nizKoja cifra nedostaje u nizu: 6 2 5 5 4 5 6 3?(Autor ovog zadatka je izjavio: „Ne poznajem nikoga ko je mogao da riješi ovu zagonetku!”) Možete li vi?

Nađite uljezaKoji se od ovih brojeva ovdje ne uklapa?3 8 15 24 35 48 54 63

Šta predstavlja slika?

U pitanju je fraktalni1 iks-oks Patrika Grima i Pola Sen Denija Na slici, matematičari Grim i Sen Deni nude analitičku prezentaciju svih mogućih partija igre iks-oks. Svako polje iks-oksa podijeljeno je na manje table da bi se prikazali različiti mogući izbori.

Iz jednog poteza U jednom dokazu Pitagorine teoreme koristi se sledeća slika Da li možete da je nacrtate jednim potezom olovke, ne podižući njen vrh od papira ne idući nijednom od duži dva puta?

DatumU Nekom mjesecu tri utorka su imala paran datum. Koji je datum bio poslednjeg petka u tom mjesecu?

1 Fraktali - slike nastale iteracijom, tj. upornim uzastopnim ponavljanjem nekog računskog ili geometrijskog postupka.

Zanimljiva matematika

Zanimljiva matematika

IZBOR REDAKCIJE

Page 31: Zanimljiva matematika

29

1. Mogu li se privlačiti dva istoimeno naelektrisana tijela?

2. Jedan kraj daske dužine 10 metara je na cilindru, a drugi kraj drži čovjek, kao na slici. On je gura naprijed a cilindar se kotrlja bez proklizavanja po horizontalnoj podlozi. Osim toga, nema ni klizanja između daske i cilindra. Daska je sve vrijeme horizontalna. Koliko rastojanje treba da pređe čovjek dok drugi kraj daske ne dođe do cilindra?

Napomena:

Pošto se cilindar kotrlja bez klizanja po podlozi, onda se duž kojom cilindar dodiruje dasku kreće se

dvaput većom brzinom od brzine kretanja ose cilindra. Zbog toga, kada čovjek pređe 10 metara, onda će osa cilindra biti ispred njega na rastojanju od 5 metara. Kada pređe još 5 metara, osa cilindra će mu biti na rastojanju od 2,5 metara itd. Hoće li čovjek uopšte i doći do cilindra?

Matematičari rješenje ovog zadatka obično svode na izračunavanje granične vrijednosti zbira beskonačno mnogo razlomaka čiji je brojilac dužina daske, a imenilac redom brojevi: 1, 2, 4, 8, 16, 32, ...

3. Talasna dužina crvene svjetlosti u vodi jednaka je talasnoj dužini zelene svjetlosti u vazduhu. Kakvu boju vidi čovjek pod vodom kada je voda osvijetljena crvenom svjetlošću?

4. Lopta je izbačena vertikalno naviše. Koje je vrijeme duže: penjanja ili spuštanja?

5. Kondukter u vozu hodao je od prvog do zadnjeg vagona i vratio se nazad, do prvog vagona. Intenzitet brzine voza je u toku ukupnog kretanja konduktera veći ili jednak intenzitetu brzine konduktera. Da li je kondukter prešao duži put u odnosu na zemlju od putnika koji mirno sjedi?

6. Dva jednaka izdubljena sferna ogledala postavljena su jedno pored drugog tako da im se glavne optičke ose i žiže poklapaju. Tačkasti izvor svjetlosti postavljen je na zajedničkoj optičkoj osi na rastojanju d od jednog ogledala. Gdje se dobija lik poslije odbijanja zraka od oba ogledala?

7. Kolikom silom treba vući uže pri dizanju tereta težine Q = 100N pomoću nepokretnog kotura, a kolikom ako se koristi pokretni kotur? Pretpostavlja se da su težine koturova dovoljno male da se mogu zanemariti.

8. Kada nam je Sunce bliže: ljeti ili zimi?

9. Kako možete u toku jedne sekunde da smanjite svoju težinu?

10. Jedan kraj žice je učvršćen za plafon, a za drugi kraj je privezano tijelo. Pustimo tijelo da slobodno pada. Pod dejstvom tijela žica će se izdužiti. Poznato je da se potencijalna energija tijela smanji za E, a u isto vrijeme, potencijalna energija žice se poveća samo za E /2. Da li je ovo u saglasnosti sa zakonom održanja energije?

Pitanja i odgovori „10+10”Priprema Radovan Ognjanović

Od ovog broja objavljujemo pitanja i odgo-vore koji sadrže različite kvalitativne zadatke i proble me, „paradokse”, manje ili više poznate po datke, činjenice, pojmove i zakone fizike. Akcenat će biti na kvalitativnoj strani posmatrane pojave.

Fizika i pitanja kojima se bave fizičari nijesu ograničeni zidovima kabineta i laboratorija, već su neprestano prisutni u svijetu u kojem postojimo.

Ovdje se ne teži saopštavanju novih znanja već osvježavanju i oživljavanju osnovnih znanja iz fizike, koje čitalac već ima. Za pitanja i odgovore iz „neobične” fizike običnih pojava nije bitno koliko će pitanja čitalac uspješno odgovoriti, već koliko će vremena razmišljati o njima. Naravno, umješna primjena ovih pitanja i odgovora u nastavi fizike pobuđuju interesovanje za fiziku i podržava aktivnije usvajanje znanja.

U svakom broju će biti po 10 novih pitanja i odgovora. Nadamo se da će vam pitanja iz zanimljive fizike ujedno biti zanimljiva pitanja iz fizike.

Zanimljiva fizika

Page 32: Zanimljiva matematika

30

Sigurno ste, ako ne probali da rješavate, onda bar čuli za sudoku. Riječ je o igri koja brzo i uspješno osvaja svijet zahvaljujući svojoj zaraznoj draži logičkih zagonetki koje vas mogu držati okupiranim satima. Smatra se da ju je izmislio čuveni švajcarski matematičar Leonard Ojler (1707-1783). Međutim, tokom osamdesetih ova igra postala je svojevrsna opsesija Japanaca koji je nazivaju sudoku (su = broj, doku = po jedan). Sudoku je veoma popularan zbog relativno jednostavnih pravila. Tok misli da bi se zagonetka riješila do kraja može postati veoma komplikovan, ali u tome je čar sudokua. Težina zagonetke zavisi od toga koliko je lako logički utvrditi brojeve koji nedostaju

Prvo ćemo preći preko osnovnih metoda rješavanja sudokua a za one koji nakon toga budu zainteresovani za ovu igru spremili smo par problema. Strpljenje i vježba su dva ključna momenta u dugoj i uspješnoj karijeri rješavanja sudokua.

Metode rješavanja

Za rješavanje tipičnog sudokua nije potrebno apsolutno nikakvo znanje iz matematike, osim sposobnosti da brojite od 1 do 9. Izazov je u tome da se brojevi stave na njihovo tačno mjesto u rešetki.

1 2 3 4 5 6 7 8 9

2

3

4 1 6 5

5 9 3 7

6 2 8 4

7

boks8

9

Standardna sudoku rešetka sadrži 81 kvadratić i podijeljena je u devet horizontalnih redova, devet vertikalnih kolona i devet bokseva ili „regiona”. Svaki od devet bokseva

se sastoji od devet kvadratića. U svaki kvadratić treba upisati po jednu cifru od 1 do 9, tako da svaki red, kolona i svaki boks sadrže devet različitih cifara. Neki kvadratići već sadrže brojeve, unaprijed zadate, koji jednoznačno određuju položaj svih ostalih brojeva u rešetki.

Postoje dvije osnovne metode koje pomažu pri izradi sudokua: ’slajsing-end-dajsing’ i brojanje. Obje metode zasnivaju se na metodi eliminacije.

Slajsing-End-Dajsing

Ovom metodom eliminišu se sve kolone i redovi u jednom boksu u kojima se već nalazi broj čija se pozicija utvrđuje (broj 3 na primjeru sa slike desno).

3

3

3

5

3

Prvo pregledate sve redove (slajsing), a onda provjerite kolone (dajsing). Važno je provjeriti sve cifre, i pregledati brojeve po redoslijedu njihove učestalosti.

U datom primjeru analizira se pozicija broja 3 u donjem lijevom boksu. Ako pogledamo redove vidjećemo da se broj 3 ne može nalaziti u poslednjem, devetom redu. Takođe ne može biti ni u prvoj ni drugoj koloni što ostavlja samo dva moguća polja u kojima bi se broj 3 mogao nalaziti. Pošto se u jednom od ta dva polja nalazi broj 5, pozicija broja 3 jednoznačno je određena u donjem lijevom boksu - možete ga sa sigurnošću upisati.

Sudoku

Logičke igre

Page 33: Zanimljiva matematika

31

Brojanje

Druga tehnika, brojanje, podrazumjeva pregled boksa, redova i kolona da bi se otkrili brojevi koji nedostaju. Vrijednost pojedinačnog kvadratića ponekad se može otkriti i brojanjem u nazad, tj. pregledom njenog boksa, reda i kolone za brojeve koji bi potencijalno mogli da odgovaraju i onda vidjeti koji je broj preostao.

1

2

6

4 5 7

3

8

9

U ovom primjeru u osjenčenom kvadratiću može biti samo broj 4, jer su sve druge mogućnosti iskorišćene - u njegovom redu nalaze se brojevi 5 i 7, u koloni brojevi 1, 2, 8 i 9 a u njegovom boksu 3 i 6.

Brojevi kandidati

Kada zagonetka postane teža, biće potrebno zabilježiti brojeve kandidate (brojeve na koje ste suzili pretragu za određeni prazni kvadratić, u primjeru sa desne strane nalaze se u svjetlijim kvadratićima). Poslije unošenja definitivnih brojeva, lako možete otkriti i zapisati brojeve kandidate za svaki kvadratić.

| | 1 9 7 4

6 5 − − − − − 1

| | 7 6 1

4 1 9 2

| 1 8 6

| 3 7 1

| 3 5 1

| 9 5

1 8 7

Ako uvidite da ste sveli jedan broj u isti red (ili kolonu) i boks, možete primijeniti metod eliminacije u fazi slajsing-end-dajsing i brojanje da biste još više suzili pretragu.

U datom primjeru broj 1 se u lijevom srednjem boksu može nalaziti na jednom od dva mjesta (do ovog zaključka lako se dolazi metodom slajsing-end-dajsing) što znači da se nigdje drugo, u istoj koloni, ne može nalaziti broj 1. To nam omogućava da sa sigurnošću upišemo broj 1 u osijenčeni kvadratić prvog boksa.

Druge, napredne metode eliminacije brojeva kandidata da bi se došlo do pravog rješenja objasnićemo u sljedećem broju, a vama ostavljamo da se, sada kada ste opremljeni osnovnim znanjem, zabavite rješavanjem zagonetki koje smo vam pripremili:

Lakša

8 9 1 2 7 6

1 3 5

5 6 4 8

3 2 1

4 5 8 6

7 2 4

9 2 4 5

6 8 9

4 8 9 7 1 3

Teža

9 7 4

6 5

7 6 1

4 9 2

8 6

3 7 1

3 5 1

9 5

1 8 7

Savjet:

- Koristite običnu olovku,- Nikada ne pogađajte! Uvijek pravite poteze bazirane na logičkoj dedukciji.

Bojana Šestović, Gordana Boljević,

IV-h

Logičke igre

Page 34: Zanimljiva matematika

32

Matematičko-logičke zagonetke i problemi

1 Gdje je euro?

Za ručak u kafani tri prijatelja platila su kelneru po 5 eura. Blagajnik za kasom rekao je kelneru da ručak ukupno vrijedi 10 eura i dao je kelneru 5 eura nazad. Kelner je svakom od trojice gostiju vratio po jedan i za sebe zadržao 2 eura. Dakle, svakog gosta ručak je koštao 4 eura, što iznosi 12 eura, i ako se još dodaju 2 eura koja je kelner zadržao za sebe, to ukupno iznosi 14 eura Gdje se izgubio jedan euro?

2 Kako dobiti auto?

Tri brata treba da podijele 35 automobila iste klase koje im je otac ostavio u nasljeđe, i to tako da najstariji brat dobije 1/2, srednji 1/3, a najmlađi 1/9 svih automobila. Budući da takvu podjelu nijesu mogli da izvrše jer 35 nije djeljiv sa 2, 3 i 9, braća su potražila pomoć od svog prijatelja matematičara. On je u podjelu uključio i svoj automobil, tako da je sada na raspolaganju imao 36 automobila koje je podijelio prema testamentu. Prvi je dobio 18, drugi 12, a treći 4 automobila. To je ukupno 34, pa je matematičar uzeo svoj automobil, a braća, zadovoljna što su podjelom dobili više nego što im po testamentu pripada, pokloniše mu i drugi automobil. Da li je, ipak, sve u redu ?3 Ko laže?

Dvoje djece, dječak i djevojčica, međusobno razgovaraju:

„Ja sam dječak”, kaže dijete sa crnom kosom

„Ja sam djevojčica”, kaže dijete sa riđom kosom.

Ako se zna da bar jedno od njih laže, ko je ko?

5 Tri gimnastičara

Tri gimnastičara imaju na majicama takmičarke brojeve 1, 3 i 6. Kakav položaj oni treba da zauzmu da dobijeni trocifren broj bude djeljiv sa 7 ?

4 Sijalica i tri prekidača

Nalazite se ispred vrata sobe u kojoj se nalazi samo jedna sijalica Isred Vas su tri prekidača. Samo jedan prekidač pali svijetlo. Vi ne vidite šta se dešava u sobi, pošto su vrata zatvorena. Imate pravo samo jednom da uđete u sobu, pritom u sobi možete da radite šta hoćete i ne možete izlaziti napolje i vraćati se opet. Kako ćete odrediti koji prekidač pali sijalicu?

Zanimljiva matematika

IZBOR REDAKCIJE

Page 35: Zanimljiva matematika

33

Zadatak je postavio Albert Ajnštajn, tvrdeći da 98% ljudi nije u stanju da ga riješi. Da li ste među onih 2%?

U jednom naselju nalazi se pet susjednih kuća u pet različitih boja. Vlasnik svake od njih druge je nacionalnosti. Svaki od tih ljudi pije drugačije piće, puši cigarete različite marke i ima drugačijeg kućnog ljubimca.

Koji od njih drži zlatnu ribicu?

Uputstva:Englez živi u crvenoj kući. -Šveđanin ima psa. -Danac pije čaj. -Zelena kuća je desno od bijele i jedna su do druge. -Vlasnik zelene kuće pije kafu. -Čovjek koji puši cigarete „Pall-Mall” drži ptice. -Vlasnik žute kuće puši cigarete „Dunhil”. -Čovjek koji stanuje u kući u sredini pije mlijeko. -Norvežanin živi u prvoj kući. -Čovjek koji puši cigarete marke „Blend”, stanuje pored čovjekakoji ima mačke. -Čovjek koji drži konje stanuje pored čovjeka koji puši cigarete „Dunhil”. -Onaj koji puši cigarete „Blue Master” pije pivo. -Njemac puši cigarete „Prince”. -Norvežanin stanuje do plave kuće. -Susjed čovjeka koji puši cigarete „Blend” pije samo vodu... -

...rješenje u sljedećem broju.

Ajnštajnov zadatak

Zanimljiva matematika

Page 36: Zanimljiva matematika

34

„Matematiku treba učiti zato što ona um u

harmoniju dovodi"

„Vi koji učite, učite matematiku.Ne gradite bez temelja"

„Matematika je ključ za cjelokupno ljudsko znanje”

Leonard Ojler

Mihail Vasiljevič Lomonosov

Galileo Galilej

Albert Ajnštajn

Leonardo Da Vinči

Rene Dekart

Citat i

„Postoji još jedan razlog zbog koga matematiku treba

posebno cijeniti. Upravo ona daje prirodnim naukama stepen pouzdanosti koji se bez nje

ne bi mogao dostići"

„Priroda je ogromna knjiga u kojoj je

napisana nauka. Ona je stalno otvorena pred našim očima,ali čovjek je ne može razumeti ukoliko prethodno ne nauči jezik i slova kojima

je napisana. A napisana je ona jezikom matematike..."

„Nikada nećemo postati matematičari, čak i ako

znamo napamet sve tuđe dokaze, ako naš um nije sposoban da samostalno rješava bilo

kakve probleme"

Page 37: Zanimljiva matematika

35

Žurka poznatih fizičara Poznati svjetski fizičari odlučili su da pođu na zajednički ručak. Posmatrajući goste, vratar, koji je bio student fizike, zapazio je sljedeće:

Ajnštajn je smatrao da je to bilo relativno dobro vrijeme za ručak.

Svi su gravitirali prema Njutnu, ali se on neprekidno kretao u krug konstantnom brzinom i nije pokazivao reakciju

Paskal je bio pod suviše velikim pritiskom da bi mogao da uživa.

Om je proveo najveći dio vremena pružajući otpor Amperovom mišljenju o toku događaja.

Volt je mislio da prisutno društvo ima veliki potencijal

Hajzenberg je mogao i da bude i da ne bude tamo

De Brojli je stajao u uglu i talasao se

Sve je magnetski privlačila Teslina ličnost.

Ispostavilo se da je Vat snažan govornik.

Herc je odlazio do stola u bifeu nekoliko puta u minuti

Faradej je bio veliki kapacitet za hranu.

Pripremila:Irena Maraš,

IV-j

Zanimljiva fizika

Nikola Tesla i njegov prvi susret sa elektricitetomMnogi kažu da se sa nekim stvarima iz djetinjstva susrijećemo cio život. Izgleda da se to desilo i Nikoli Tesli. Kako je izgledao njegov prvi susret sa elektricitetom, najbolje je opisao on sam:„Bio je suton. Pomilovao sam svog mačka i tada se dogodilo čudo od koga sam zanijemio. Moja ruka je izazvala pljusak praskavih varnica, a iznad mačkovih leđa se stvorilo polje svjetlosti.”Teslin otac Milutin Tesla, smatrao je da je to ona ista pojava koja se stvara pri udaru groma, elektricitet.Na kraju svega, svakako je najzanimljiviji zaključak povodom ovog događaja. Nikola Tesla je tada postavio pitanje na koje će samo on znati da odgovori:„A ja sam očaran razmišljao: - Nije li priroda jedna ogromna mačka? Ako jeste, ko nju miluje po leđima? Tada sam imao tri godine a čuda koja su dolazila bila su sve veća.”„Čudesni” mačak je bio prijatelj malog Nikole, bili su nerazdvojni. Možda je i na taj način priroda htjela da pokaže kako će otkrivati svoje najveće tajne onima koji budu voljeli i cijenili podjednako i ljude i životinje kao što je to činio Nikola Tesla.

Iz knjige „Tesla, mladost” Vladimira Pištala

Plava tečnostLudi naučnik zuri u bočicu plave tečnosti. „Ova supstanca ostaje tečna na −40°”, rekao je asistentu Borisu.Boris je upitao: „Celzijusovih ili Farenhajtovih?”Svojim tamnim očima, naučnik je pogledao Borisa i rekao: „Nije važno”.Zašto je naučnik to rekao?Sami zaključite ili, ako ste nestrpljivi, pogledajte rješenje na strani RJEŠENJA

Zaniml j iva f iz ika

Page 38: Zanimljiva matematika

36

Razgovarali smo sa učenikom naše škole Bećom Merulićem dan priјe polaska u Makedoniјu na Balkansku Matematičku Olimpiјadu. Ovaј mladi matematičar već јe uspјešno učestvovao na raznim takmičenjima:

: Zašto baš matematika?Bećo: Za matematiku sam se odlučio u šestom razredu osnovne škole, јer јe to bio јedini predmet iz koјeg јe bilo moguće takmičiti se tada. Na Regionalnom takmičenju bio sam prvi i plasirao se za Republičko, na koјem sam osvoјio drugo mјesto, pa јe usliјedilo Savezno takmičenje. Uspјeh godi. Riјešio sam da nastavim da se takmičim iz matematike.

: Da li јe smisao za matematiku rezultat rada ili talenat?Bećo: Talenat јe, naravno, potreban. I nema ga svako. Smatram da onaј ko nema talenta ne može da postigne uspјeh na takmičenju. Naravno, potrebno јe i mnogo rada.

: Kako na učenika u vezi s tim utiče nastavnik?Bećo: U osnovnoј školi mi јe predavao sјaјan nastavnik, koјi јe matematiku uviјek činio interesantanom i birao zanimljive zadatke. To јe, svakako, dopriniјelo da zavolim ovaј predmet.

: U osmom razredu si nakon Republičkog takmičenja otišao na Balkaniјadu, takmičenje na koјem su učestvovala 72 učenika iz 12 zemalja. Koliko ti znači bronzana medalja koјu si osvoјio tada u Bugarskoј?Bećo: Naravno da mi znači mnogo. Bilo јe to nezaboravno iskustvo. Medalja јe podstrek za dalje dokazivanje, za više rada. Onda ni uspјeh ne izostaјe.

: Tvoјe ovogodišnje učešće na takmičenju potvrđuјe da se nećeš zadovoljiti postignutim. Da li tvoјe ambiciјe prevazilaze balkansko takmičenje?

Bećo: Naravno da Balkaniјada niјe moј kraјnji cilj. To јe bilo veoma važno takmičenje, ali nadam se da neće biti i naјveće.

: Koјe su glavne teme razgovora kada se na јednom mјestu nađe toliko talentovanih, mladih matematičara?Bećo: Razgovarali smo o takmičenjima, zadacima, načinu vјežbanja. Sporazumiјevanje niјe bilo problem. Stekao sam nove priјatelje.

: Šta te јe motivisalo da kreneš na školsko takmičenje u Gimnaziјi?Bećo: Motiviše me želja da nastavim uspјeh iz osnovne škole.

: Da li јe osvoјeno prvo mјesto značilo više obaveza i nove, naporne pripreme za Republičko takmičenje?Bećo: Za Republičko takmičenje niјesam se dugo spremao. Bilo јe u vriјeme ozbiljnih obaveza iz drugih predmeta. Sedmicu pred takmičenje bio sam slobodan, što sam iskoristio da vјežbam. Pomogla mi јe, naravno, profesorica, koјa mi јe dala nekoliko zbirki zadataka.

: Prošle godine јe NVO „Dјetinjstvo” tražila naј diјete iz različitih oblasti. Ti si odabran za naј diјete nauke. Vјerovatno si ponosan.Bećo: Naravno da sam ponosan. Uspјeh svakom priјa i rad čini lakšim.

Kada smo Beća pitali šta će studirati, odgovorio nam јe da јoš uviјek razmišlja, ali će to vјerovatno biti - ekonomiјa. Za kraј smo ga zamolili da matematiku opiše јednom riјečјu. Najprije tišina. Bećo konstatuјe da јe to veoma teško. Ali uspјešno nalazi rјešenje zadatka i odgovara: razmišljanje. Složili smo se. Razmislite i vi o tome.

Od Beća smo se rastali uz želju da na Balkaniјadi osvoјi zlato. Znamo da će to, ako se ne desi ove, sigurno biti sljedeće godine. Biće to povod da ponovo razgovaramo s njim o novim uspјesima.

Milan Lakićević,Božidar Bukilić

III - L

U razgovoru sa Bećom

Intervju

Page 39: Zanimljiva matematika

37

Odabrani zadaci„S lakoćom uraditi ono što je drugima teško, eto talenta;

učiniti ono što je nemoguće talentu, eto genija!”Anri Frederik Amijel

Profesori sa PMF-a biraju za vas:Za one koji vole da rješavaju nestandardne zadatke iz matematike, rubrika „Odabrani zadaci” će svakim brojem donositi nove zadatke iz raznih oblasti matematike.Svi čitaoci mogu se takmičiti u rješavanju odabranih zadataka. Rješenja zadataka slati otkucana ili čitko ispisana na adresu ili e-mail redakcije časopisa, najkasnije do 01. maja 2009. godine. U sljedećem broju nagrađuju se tri takmičara koja su bila najuspješnija u rješavanju odabranih zadataka.

1. U prvi razred policijske srednje škole upisano je n učenika. Na prvom času gimnastike, učenici su postrojeni u jedan red i okrenuti licem ka nastavniku, kao na slici 1

slika 1. stanje prije komande „NALIJE-VO”

Na njegovu komandu „NALIJEVO”, dio učenika se okrene nalijevo a dio učenika se, zbog treme, okrene nadesno (npr. kao na slici 2).

slika 2 - jedno moguće stanje poslije komande „NALIJE-VO”

Shvativši grešku, u sljedećoj sekundi svi učenici koji gledaju jedan u drugog i susjedi su, okreću se za 180 stepeni (vidi sliku 3).

slika 3. stanje poslije prve sekunde

Poslije još jedne sekunde, opet se svi učenici koji gledaju jedan u drugog i susjedi su okreću za 180 stepeni, pa imamo stanje kao na slici 4.

slika 4. stanje poslije druge sekunde

Postupak se nastavlja u svakoj sekundi. Dokazati da će se okreti u jednom trenutku završiti, bez obzira koliki je broj n i kako su se okrenuli učenici na komadu „NALIJEVO”.

2. Tablica 5 × 5 puni se brojevima iz skupa -1, 0, 1, a zatim se izračunaju sume po vrstama, kolonama i na obje dijagonale. Dokazati da bez obzira na način popunjavanja tablice među dobijenim sumama postoje bar dvije jednake.

3. Postoji li prirodan broj n tako da je brojk n n n n= + + + + +1 2 3 2008...djeljiv sa 2008? Odgovor obrazložiti.

4. Odrediti sve prirodne brojeve koji su za 2008 veći od zbira kvadrata svojih cifara.

5. Dokazati da važi 2 3 2008 220083+ + + <...

6. Naći minimum funkcije:

f x x x x x( ) = − + + − +2 24 8 10 41

Za koje se vrijednosti dostiže taj minimum?7. Na pravoj su date:

a) tri tačke A, B i C; b) četiri tačke A, B, C i D.Odrediti na pravoj tačku T tako da zbir rastojanja od tačke T do datih tačaka bude minimalan.

8. Dat je pravougaonik ABCD. Dvije kružnice konstru-i sane su tako da jedna od njih sadrži tačke A i D i dodiruje stranicu BC, a druga sadrži tačke A i B i dodiruje stranicu CD. Dokazati da važi:8∙(R1+R2) ≥ 5∙(AB+AD)

gdje su R1 i R2 poluprečnici kružnica.

Adresa RedakcijeGimnazija „Slobodan Škerović”,Časopis za rubriku „Odabrani zadaci”,Vaka Đurovića 36, 81 000 Podgoricae-mail: [email protected]

Rješenja zadataka biće objavljena u sljedećem broju.

Odabrani zadaci

Page 40: Zanimljiva matematika

38

Zadaci

I razred

1. Neka je K = + ⋅ + +( ) ( )......( )13579 1 13579 1 13579 12 2 20 1 13579

Dokazati da važi 13579 1 13578213580 − =K

.

2. Poluprava iz tjemena A trougla ABC siječe stranicu BC u tački D, tako da je D između tačaka B i C, i siječe opisani

krug tog trougla u tački E. Dokazati da važi nejednakost: 1 1 4AD DE BC

+ ≥ .

3. Čudesni „4 - Kalkulator” ima četvorocifreni displej i četiri tastera. Kalkulator radi na sljedeći način:

- pritiskom na taster 1, broj na displeju se zamjenjuje sa 1.- pritiskom na taster 2, broj na displeju se dijeli sa 2.- pritiskom na taster 3, broj na displeju se smanji za 3.- pritiskom na taster 4, broj na displeju se pomnoži sa 4.

Na početku je na displeju 0. Svaka operacija koja za rezultat daje negativan broj, decimalni ili broj sa pet cifara, se ignoriše.Da li se na displeju može pojaviti broj 2008? Može li se pojaviti 2010? Obrazložiti odgovore!

4. Neka je F preslikavanje takvo da za sve realne brojeve x i y važi F x y F x F y⋅( ) = +( ) ( )

Ako je F 12008

2008

= , odrediti F 20082008( ).

II razred

1. Neka je Ѕ skup nekih kompleksnih brojeva, koji ima više od 5 elemenata. Ako za svako z S∈ važi | |z − ≤2 2,

tada postoje dva različita broja z i w iz S sa svojstvom | |z w− ≤ 2 Dokazati

2. Ako su a, b i c tri različita realna broja, onda su rješenja jednačine 1 1 1 0x a x b x c−

+−

+−

= dva različita realna broja Dokazati

3. Neka su E i F bilo koje tačke konture nekog 2008-tougla. Dokazati da je dužina duži EF manja ili jedanka dužini najveće dijagonale mnogougla.

4. U kružnici radiusa r upisani su jednakostranični trougao i kvadrat sa jednim zajedničkim tjemenom. Izračunati (u funkciji od r) površinu lika koji je zajednički trouglu i kvadratu. Da li je ta površina veća ili manja od polovine površine kruga?

Državno takmičenje 2008Matematika

Državno takmičenje

39

III razred

1. Neka je a1, a2, ... , an, ... beskonačan niz realnih brojeva takav da za svaki prirodan broj n važi an+2 – 2an+1+an=1

Ako su poznati članovi a1 i a2 toga niza, izraziti član an kao funkciju od a1, a2 i n (n je proizvoljan prirodan broj).

2. Dokazati da za proizvoljne pozitivne brojeve a, b, c i d važi najednakost

( )( ) ( )( ) ( )( ) .a b c d a c b d a d b c abcd+ + + + + + + + ≥ 64

3. Neka je P(x) polinom sa cjelobrojnim koeficijentima. Na grafiku funkcije y = P(x) u koordinatnom sistemu xOy uočene su dvije tačke A(x1, y1) i B(x2, y2), pri čemu su njihove koordinate x1 i x2 cijeli brojevi. Ako je poznato da je udaljenost između tačaka A i B cijeli broj, dokazati da je y1 = y2

4. U oštrouglom trouglu ABC povučene su visine AA1, BB1 i CC1 (tačke A1, B1 i C1 redom pripadaju stranicama BC, CA i AB). Ako su Ѕ(ABC) i Ѕ(A1B1C1) površine trouglova ABC i A1B1C1, a α, β i γ uglovi trougla ABC, dokazati da važi

S ABCS ABC( )( )

cos cos cos .1 1 1 2 2 21= − − −α β γ

IV razred

1. Osnova kvadra ABCDA1B1C1D1 je kvadrat. Odrediti najveću moguću veličinu ugla koji zaklapaju prava BD1 i ravan BDC1

2. Riješiti jednačinu x xx x3 2 2 1 2 4 03

−( ) ⋅ −( ) + −( ) ⋅ =sin sin u skupu realnih brojeva.

3. Neka su a, b, c i d realni brojevi takvi da je a ≤ b i c ≤ d. Cjelobrojna pravougaona rešetka je skup ( , ) | , , ,x y a x b c y d x Z y Z≤ ≤ ≤ ≤ ∈ ∈ , gdje Z označava skup cijelih brojeva. Data je cjelobrojna pravougaona

rešetka dimenzija 3×2008 (vidi sliku). Koliko postoji pravouglih trouglova čija su tjemena čvorovi rešetke?

. . . . .

. . . . .

. . . . .2008 kolona

4. Neka je N skup prorodnih brojeva i g: N→N funkcija takva da je g(k) najveći neparni djelilac prirodnog broja k

Dokazati da za svaki prirodan broj n važi g kk

nk

n ( ) .>=∑ 2

31

Takmičenja

Page 41: Zanimljiva matematika

39

III razred

1. Neka je a1, a2, ... , an, ... beskonačan niz realnih brojeva takav da za svaki prirodan broj n važi an+2 – 2an+1+an=1

Ako su poznati članovi a1 i a2 toga niza, izraziti član an kao funkciju od a1, a2 i n (n je proizvoljan prirodan broj).

2. Dokazati da za proizvoljne pozitivne brojeve a, b, c i d važi najednakost

( )( ) ( )( ) ( )( ) .a b c d a c b d a d b c abcd+ + + + + + + + ≥ 64

3. Neka je P(x) polinom sa cjelobrojnim koeficijentima. Na grafiku funkcije y = P(x) u koordinatnom sistemu xOy uočene su dvije tačke A(x1, y1) i B(x2, y2), pri čemu su njihove koordinate x1 i x2 cijeli brojevi. Ako je poznato da je udaljenost između tačaka A i B cijeli broj, dokazati da je y1 = y2

4. U oštrouglom trouglu ABC povučene su visine AA1, BB1 i CC1 (tačke A1, B1 i C1 redom pripadaju stranicama BC, CA i AB). Ako su Ѕ(ABC) i Ѕ(A1B1C1) površine trouglova ABC i A1B1C1, a α, β i γ uglovi trougla ABC, dokazati da važi

S ABCS ABC( )( )

cos cos cos .1 1 1 2 2 21= − − −α β γ

IV razred

1. Osnova kvadra ABCDA1B1C1D1 je kvadrat. Odrediti najveću moguću veličinu ugla koji zaklapaju prava BD1 i ravan BDC1

2. Riješiti jednačinu x xx x3 2 2 1 2 4 03

−( ) ⋅ −( ) + −( ) ⋅ =sin sin u skupu realnih brojeva.

3. Neka su a, b, c i d realni brojevi takvi da je a ≤ b i c ≤ d. Cjelobrojna pravougaona rešetka je skup ( , ) | , , ,x y a x b c y d x Z y Z≤ ≤ ≤ ≤ ∈ ∈ , gdje Z označava skup cijelih brojeva. Data je cjelobrojna pravougaona

rešetka dimenzija 3×2008 (vidi sliku). Koliko postoji pravouglih trouglova čija su tjemena čvorovi rešetke?

. . . . .

. . . . .

. . . . .2008 kolona

4. Neka je N skup prorodnih brojeva i g: N→N funkcija takva da je g(k) najveći neparni djelilac prirodnog broja k

Dokazati da za svaki prirodan broj n važi g kk

nk

n ( ) .>=∑ 2

31

Takmičenja

Page 42: Zanimljiva matematika

40

Rješenja

I razred

1. Važi:

13578 13578 13579 1 13579 1 13579 1 13572 2 20 1 2

⋅ = ⋅ +( ) ⋅ +( ) ⋅ +( ) ⋅ ⋅K ... 99 1

13579 1 13579 1 13579 1 13579

2

2 2 2 2

13579

0 0 1 2

+( ) == −( ) ⋅ +( ) ⋅ +( ) ⋅ +11 13579 1

13579 1 13579 1 13579

2

2 2 2

13579

1 1 2

( ) ⋅ ⋅ +( ) == −( ) ⋅ +( ) ⋅ +

...

11 13579 1

13579 1 13579 1

2

2 2

13579

13579 13579

( ) ⋅ ⋅ +( ) =

= −( ) ⋅ +( )

...

== −13579 1213580 .

2. Iskoristimo nejednakost između aritmetičke i geometrijske sredine:

1 1 2 1 1 2AD DE AD DE AD DE

+ ≥ ⋅ =⋅

(1)

Važi sljedeće:

∠ADC ≅ ∠BDE, kao unakrsni uglovi;

∠BED ≅ ∠DCA, kao periferijski uglovi nad istim lukom BA

Odatle, na osnovu drugog stava sličnosti trouglova slijedi

∆ ∆ADC BDE

ADBD

DCDE

AD DE BD DC

~

.

=

⋅ = ⋅

Sada iz (1) dobijamo

1 1 2 2AD DE AD DE BD DC

+ ≥⋅

=⋅

(2)

Iskoristimo nejednakost između aritmetičke i geometrijske sredine

BD DC BD DC BC

BD DC BC

⋅ ≤ + =

⋅≥

2 2

1 2

(3)

Pa iz (2) i (3) dobijamo 1 1 4AD DE BC

+ ≥

3. 2008 se može pojaviti kao rezultat na displeju. Na startu je 0. Na primjer pritiskom na 1 dobijamo 1. Zatim pritiskom šest puta na 4 dobijamo 4096, i najzad 696 puta biramo 3 i dobijamo 2008. 2010 se ne može pojaviti kao rezulatat rada „Kalkulatora”. Treba primijetiti da ako broj na displeju nije djeljiv sa 3, onda se nijednom operacijom ne može dobiti broj djeljiv sa 3. Naime, na početku je 0 na displeju. Jedina operacija koja sada daje rezultat je biranje tastera 1. 1 nije djeljiv sa 3, pa nakon toga nijedan broj dobijen kao rezultat rada „Kalkulatora” neće biti djeljiv sa 3, a 2010 je djeljiv sa 3.

Takmičenja

A

B D C

E

Page 43: Zanimljiva matematika

41

4.

F F F F12008

12008

2008 12008

220092008

2009

= ⋅

=

+ 0008

12008

12008

2008

12008

2008

20082008

( )

= ⋅

+ ( )

=

F F

F ++

+ ( )

=

+

+

F F

F F F

12008

2008

12008

12008

20082008

112008

2008

2009 12008

2008

20072008

2008

+ ( )

= ⋅

+ ( )

F

F F

F F2008 2008 1

200820082008 2( ) = − ⋅

= − .

II razred

1. Rješenja nejednačine |z−2| ≤ 2 čine zatvoreni krug. Neka je z=x+yi dobijamo (x−2)2+y2≤2 koja definiše zatvoreni krug sa centrom u tački (2, 0) i poluprečnika 2. Podijelimo sada ovaj krug pravima koje prolaze kroz tačku (2, 0) na 6 podudarnih zatvorenih kružnih isječaka, Ik, k =0, 1, 2, 3, 4, 5, pri čemu neka od tih pravih sadrži neku tačku recimo z0 iz skupa S. Tada je rastojanje dvaju tačaka sa proizvoljno odabranog isječka manje ili jednako od 2. Pretpostavimo da S ima više od 5 tačaka. Ako ima više od 6 onda na osnovu Dirihleovog principa makar dvije pripadaju nekom isječku. Pretpostavimo da S =zk, k = 0, 1, 2, 3, 4, 5 ima tačno 6 tačaka. Pretpostavimo, određenosti radi, da je zk∈Ik, k = 0, 1, 2, 3, 4, 5. Kako je broj z0 sa granice isječka I0 on istovremeno pripada isječku I5. Znači |z0 − z5| ≤ 2 što je trebalo dokazati.

2. Ako jednačinu pomnožimo najmanjim zajedničkim sadržaocem dobijamo: (x - b)(x - c) + (x - a)(x - c) + (x - a)(x - b) = 0, odnosno 3x2 - 2x(a + b + c) + (ab + bc + ac) = 0. Da bi rješenja ove jednačine bila realna i različita mora biti diskriminanta D > 0. To dokazujemo ovako:

D = 4(a + b + c)² – 4 ⋅ 3(ab + bc + ca),4(a + b +c)² – 12(ab + bc + ca) > 0 / : 22a² + 2b² + 2c² – 2ab – 2bc – 2ca > 0,a² - 2ab + b² + a² – 2ac + c² + b² – 2bc + c² > 0,(a – b)² + (a – c)² + (b – c)² > 0. Jasno je da posljednja nejednakost važi za sve različite realne brojeve a, b i c

3. Neka su Ak, k = 1,..., 2008 tjemena mnogougla. Ako je EF diagonala mnogougla onda tu nema šta dokazati. Pretpostavimo da je E tačka duži AkAk+1 i F tačka duži Al Al+1. Vidjeti sliku:Dokažimo da je EF ≤ maxEAk, EAk + 1. Neka je, određenosti radi, EAk + 1 ≤ EAk Tada je ugao ∠FAk E ≤ ∠FAk + 1E Kako je ∠AkFE spoljašnji ugao trougla ∆FAk+1E, slijedi da je ∠AkFE >∠FA k+1E. Zaključujemo da je ∠FAkE<∠AkFE, odnosno da u trouglu ∆Ak FE važi sljedeći odnos stranica: Ak E > FE Ako sada na trouglu ∆Ak Al Al+1 primjenimo prethodni postupak dobijamo da je duž AkE manja od diagonale Ak Al ili diagonale Ak Al+1. Odavde zaključujemo da je duž EF manja od jedne dijagonale mnogougla

Takmičenja

AkF

Ak+1

Al

Al+1

E

Page 44: Zanimljiva matematika

42

4. Tražena površina (vidi sliku) je P = PABCDE = 2(PAHE – PGHD). AH = 2r, dok je stranica jednakostraničnog trougla q = r 3 Iz ∆AFE je

AF = FE 3 (1) AF = AH – FH = AH – FE odnosno AF = 2r – FE (2)

Iz (1) i (2) imamo FE ⋅ 3= 2r - FE, tj. FE r r=+

= −( )23 1

3 1 .

Kako je AG a r= =2

3 32, to je GH r GD= =

2.

Uvrštavanje daje

P AH FE GH GD AH FE GH GD

r r r r r

= ⋅ ⋅ − ⋅

= ⋅ − ⋅ =

= ⋅ −( ) − ⋅ = −

22 2

2 3 12 2

2 32 994

Dakle P ≈ 1,21 r ² < π/2r ², te je tražena površina manja od polovine površine kruga.

III razred

1. Uslov an+2 – 2an+1+an =1 dat u zadatku možemo pisati kao

(an+2 – an+1) + (an+1 – an) = 1 (1)

Dakle, ako definišemo novi niz bn kao b a a n Nn n n= − ∈+1 , , jednakost (1) postaje b bn n+ − =1 1. Dakle, bn je

aritmetički niz sa razlikom 1, odakle slijedi bn= b1+ n – 1 = a2 – a1+ n – 1 za svaki prirodan broj n

Koristeći prethodnu jednakost, dobijamo

a a a a a a a ab b b

n n n n n

n n

= − + − + + − + == + + + +

− − −

− −

( ) ( ) ... ( )...

1 1 2 2 1 1

1 2 1 aa

n b n a

n b n n a n a a

1

1 1

1 1 2 1

12

2 2

1 1 22

1

=

= − + − + =

= − + − − + = − −

( )

( ) ( )( ) ( )( ) ++ − − + =

= − − − + − −

( )( )

( ) ( ) ( )( )

n n a

n a n a n n

1 22

1 2 1 22

1

2 1

Dakle, za svaki prirodan broj n važi a n a n a n nn = − − − + − −( ) ( ) ( )( )1 2 1 2

22 1

2. Na osnovu nejednakosti x y xy+ ≥2

, za x, y ≥ 0 (između aritmetičke i geometrijske sredine dva pozitivna broja) dobijamo

( )( )

( )( )

a b c d a b c d ab cd abcd

a c b d a c b d

+ + = + ⋅ + ≥ ⋅ =

+ + = + ⋅ +

22 2

2 2

22 2

4 4 4

≥≥ ⋅ =

+ + = + ⋅ + ≥ ⋅ =

2 2

22 2

2 2

4 4 4

4 4 4

ac bd abcd

a d b c a d b c ad bc abcd( )( )

Takmičenja

A

L

N

C

B

H

ME

D

GFO

Page 45: Zanimljiva matematika

43

Sabiranjem prethodne tri nejednakosti dobijamo traženu nejednakost.

3. Kako je po uslovu zadatka A(x1, P(x1)) i B(x2, P(x2)), gdje je y1= P(x1), y2= P(x2) i x1≠ x2, to udaljenost između tačaka A i B iznosi

| | ( ) ( ( ) ( )) | | ( ) ( )AB x x P x P x x x P x P xx x

= − + − = − + −−

1 2

21 2

21 2

1 2

1 2

12

(1)

Stavimo P x a x a x a x a x ann

nn

nn( ) ...= + + + + +−

−−

−1

12

21 0 , pri čemu su po pretpostavci an, an–1, ... , a1, a0 cijeli brojevi

i an ≠ 0. Otuda imamo

P x P x a x x a x x a x xnn n

nn n( ) ( ) ( ) ( ) ... ( )1 2 1 2 1 1

12

11 1 2− = − + − + + −−

− − (2)

Kako za svaki prirodan broj važi identitet ( ) ( )( ... )x x x x x x x x x xk k k k k k1 2 1 2 1

11

22 1 2

22

1− = − + + + +− − − − , zaključujemo da je

cijeli broj x xk k1 2− djeljiv sa x1 – x2

Koristeći tu činjenicu, na osnovu (2) slijedi da je m P x P xx x

= −−

( ) ( )1 2

1 2

cijeli broj, što uvršteno u (1) daje

| | | |AB x x m= − +1 221

Po uslovu zadatka |AB| je cijeli broj, pa stoga mora biti 1 2

1 2

+ =−

m ABx x| |

| | razlomak

Stoga stavimo 1 2+ =m pq

, gde su p i q relativno prosti prirodni brojevi.

Kvadriranjem prethodne jednakosti dobijamo ( )1 2 2 2+ =m q p

Iz prethodne jednakosti vidimo da je p² djeljivo sa q², pa zbog pretpostavke da su p i q relativno prosti prirodni brojevi, zaključujemo da mora biti q = 1 odnosno 1 + m2 = p2. Otuda je 1 = (p – m)(p + m), što je jedino moguće u slučaju p – m = p + m = 1, tj. p = 1 i m = 1

Otuda slijedi P x P xx x

m( ) ( ) ,1 2

1 2

0−−

= = odnosno P(x1) = P(x2), tj. y1 = y2, što je i trebalo dokazati.

4. Na osnovu slike imamo S ABC S ABC S B AC S C BA S ACB( ) ( ) ( ) ( ) ( )− = + +1 1 1 1 1 1 1 1 1 (1)

Nadalje važi:

S B AC AB AC

AB AC

AB

( ) | | | | sin

| | cos | | cos sin

|

1 1 1 1121212

= ⋅ =

= ⋅ =

=

α

α α α

|| | | sin cos

( ) cos .

⋅ =

= ⋅

AC

S ABC

α α

α

2

2

Ovim smo dobili S B AC S ABC( ) ( ) cos1 12= ⋅ α , a na analogan način dobijamo

S C BA S ABC( ) ( ) cos1 12= ⋅ β i S ACB S ABC( ) ( )cos1 1

2= γ

Uvrštavanjem prethodna tri izraza u (1) dobijamo S ABC S ABC S ABC( ) ( ) ( )(cos cos cos )− = + +1 1 1

2 2 2α β γ

Dijeljenjem gornje relacije sa S(ABC) dobijamo

1 1 1 1 2 2 2− = + +S ABCS ABC( )( )

cos cos cosα β γ , odakle slijedi tražena jednakost

Takmičenja

AB

A1

B1

C

C1

Page 46: Zanimljiva matematika

D A

BC

D1 A1

B1C1

QP

O

44

IV razred1. Neka je O tačka presjeka dijagonala kvadrata ABCD Trougao BDC1 je jednakokraki, pa je C1O njegova visina. Ako

je P presjek dijagonala kvadra, Q projekcija tačke P na ravan trougla BCD1. Tačka P je podjednako udaljena od tjemena B, D i C1, pa je Q centar opisanog kruga oko trougla BCD1. To znači da tačka Q pripada duži C1O. Traženi ugao je α =∠PBQ

Označimo dužinu ivice AB sa a i dužinu ivice AA1 sa b Tada je PB a b= +12

2 2 2 .Ako je O1 presjek dijagonala

kvadrata A1B1C1D1, tada jeCO a1 1

22

= , a primjenom Pitagorine teoreme CO a b1

2

222

=

+

Iz sličnosti trouglova POQ i C1OO1 dobijamo

PQCO

POCO1 1 1

= , pa je PQ

b a

a b

aba b

=⋅

+=

+2

22

22 22

22 2

.

Tada je sin .α = =+( ) ⋅ +( )

PQPB

ab

a b a b2 2 2 22 2

Kako je traženi ugao oštar, ugao α će biti najveći kada je sinα najveći.Koristeći transformacije dobijamo

sinα =+( ) ⋅ +( )

=

=+ +( )

=

=

+

ab

a b a b

ab

a b a b

ab

ba

2 2 2 2

4 4 2 2

2

2

2

2

2 2

2 2 5

1

2

+ 5

.

Primjenom nejednakosti artimetičke i geometrijske sredine dobijamo da je

ab

ba

ab

ba

2

2

2

2

2

2

2

22 2+ ≥ ⋅ = , pa je

sin .α =

+

+

≤⋅ +

=1

2 5

12 2 5

132

2

2

2ab

ba

Otuda je najveća vrijednost traženog ugla jednaka arcsin .13

2. Kako su funkcije f (t) = 2t i g (t) = t3 rastuće važe sljedeća tvrđenja:

- ako je sin x > 0, tada je 2sin x > 20 = 1, pa je 2sin x –1 > 0,- ako je sin x < 0, tada je 2sin x < 20 = 1, pa je 2sin x –1 < 0,- ako je x³–2 > 0, tada je 2x3– 4 > 0,- ako je x³–2 < 0 tada je 2x3– 4 < 0

Dakle oba sabirka su istog znaka, tj. istovremeno su oba pozitivna ili ova negativna. Jedina rješenja jednačine dobijaju se kada su oba sabirka jednaka nuli, tj. za x = 23 ili x k k Z= ∈π , .

3. Postavimo koordinatni sistem tako da se apscisa poklapa sa srednjom linjom mreže, a ordinata da bude normalna na

Takmičenja

C1

Q

P

O1

OC

b

22a

Page 47: Zanimljiva matematika

45

apscisu i da bude osa simetrije mreže. Neka je C tjeme pravog ugla, tada su na slici prikazane sve mogućnosti:

1 2 3 4 5 6 7 8

Trouglova grupe 1, čije su obje katete paralelne koordinatnim osama, ima 3n ⋅ 2 ⋅ (n –1), jer tjeme C može biti bilo koji čvor mreže (3n mogućnosti), dvije mogućnosti za tjeme na vertikali i n – 1 mogućnost za tjeme na horizontali.Trouglovi iz grupa 2, 3, 4, 5 i 6 su jednokraki sa katetama koji zaklapaju ugao od 45° sa koordinatnim osama.Njih ima: 2 ⋅ (n – 2) (grupa 2, množimo sa 2 jer postoji simetrija u odnosu na apscisu, tjeme C ne može biti u prvoj i u 2008. koloni), 2 ⋅ (n –2) + 2 ⋅ (n – 1) (grupa 3 i 4), 2 ⋅ (n – 4) (grupa 5) i 4 ⋅ (n – 3) (grupa 6, množimo sa 4 zbog simetrije u odnosu na apscisu i ordinatu).

Trouglovi iz grupa 7 i 8 imaju katetu dužine 5 . U grupi 7 ima ih 4 ⋅ (n – 3), a u grupi 8 ima ih 4 ⋅ (n – 5).

4. Za svaki prirodan broj k, važi k g kp k= ⋅2 ( ) ( ), pa je g kk

g kg kp k p k

k

n

k

n

k

n ( ) ( )( )

.( ) ( )=⋅

====∑∑∑ 2

12111

Među brojevima 1, 2, ..., n postoji n2

djeljivih sa 222, n

djeljivih sa 22

2,... nm

djeljivih sa 2m, gdje je m

prirodan broj takav da je 2m≤ n, 2m+1> n. Tada je brojeva za koje je p(k) = j jednak n nj j2 2 1

+ , pa se tražena

suma može napisati na sljedeći način:

g kk

n n

nk

n

j j jj

m

j j

( )=

+=

∑ ∑=

=

=

1

10

12 2 212 2

=

=

+ −

=−

=

−=

∑ ∑j

m

j jj

m

j jj

n

n0

11

0 11

12 2

212

12

mm

j

jj

m

j

n

n n

=

= −

=

212 21

.

Kako je za svaki negativan realan broj x ≥ [x], to važi:

g kk

n n n n n nk

n

jj

m

j jj

m

j jj

m( )= = = =∑ ∑ ∑ ∑= −

≥ − ⋅ = − =

1 1 1 1

12 2

12 2

14

nn n n n n n nm

m m− ⋅ ⋅−

−= − ⋅ ⋅ −

= +

⋅>1

4

1 14

1 14

13

1 14

23 3 4

23.

Takmičenja

Page 48: Zanimljiva matematika

46

FizikaZadaci

I razred

1. Mrav se udaljava od mravinjaka po pravoj liniji. Brzina mrava je proporcionalna kvadratnom korijenu njegovog rastojanja od mravinjaka. U početnom trenutku mrav se nalazio na rastojanju 1cm od mravinjaka i njegova brzina je bila 2cm/s a) Kolika će biti brzina mrava u trenutku kada se on nađe na rastojanju 4cm od mravinjaka?b) Nacrtati grafik zavisnosti recipročne vrijednosti brzine mrava od njegovog rastojanja od mravinjaka.c) Na osnovu tog grafika, odrediti približno u kom trenutku će rastojanje između mrava i mravinjaka biti 4cm.

2. Čovjek se nalazi u čamcu na jezeru i želi da odredi masu čamca M. Kako da to uradi, ako zna svoju masu m i ima samo dugačku pantljiku (traku) kojom može da izmjeri rastojanje od obale i dužinu čamca?

3. U Lamertovom eksperimentu (shema aparature kao na Sl. 1), određuje se brzina molekula (atoma) gasa. U molekul-skoj peći 2 dovodi se do željene temperature, npr. gas atoma žive ili pare nekog drugog istopljenog metala. Čestice gasa prolaze u snopu kroz niz pukotina (kolimator 4) u vakuumirani cilindar 1. U njemu se nalaze dva kružna diska, 5 i 6, sa uskim lučnim prorezima širine Δφ = 2º, koji su zakrenuti za ugao φ = 24° jedan u odnosu na drugi (kao na Sl. 2). Rastojanje između diskova je l = 40cm, a ugaona brzina kojom oni rotiraju oko ose cilindra je ω = 50 obrta/s. Čestice gasa koje prođu oba proreza, registruju se u detektoru 3. Odrediti maksimalnu i minimalnu brzinu čestica gasa

1

l

5

4

632

∆φ∆φ

φ

Sl 1 Sl 2 4. Ocijeniti srednju vrijednost gustine supstance Sunca. Poznat je period rotacije Zemlje oko Sunca T=1 god,

gravitaciona konstanta γ = 6,7∙10−11 Nm2/kg2 i ugao pod kojim se vidi prečnik Sunčevog kruga sa Zemljeα⊗=2RS/r=0,01rad, gdje je RS–poluprečnik Sunca, r–rastojanje između Zemlje i Sunca.

5. Satelit poluprečnika Rsat se okreće jednakom ugaonom brzinom oko svoje ose i oko masivne planete na rastojanju R Izračunati količnik momenta impulsa satelita L1–za rotaciju oko planete i L2–za rotaciju oko sopstvene ose. Poznat je moment inercije kugle I = 2mR2/5

II razred

1. U izolovanom cilindričnom sudu nalazi se gas mase m i molarne mase M Gas je od atmosfere odvojen klipom koji je za dno suda vezan oprugom konstante elastičnosti k (slika 1). Na temperaturi T1 klip se nalazi na visini h od dna suda. Ako je trenje između klipa i suda zanemarivo, odrediti temperaturu T2 na koju treba zagrijati gas da bi se klip podigao na visinu H

2. Dva mola idealnog gasa na temperaturi T1 = 300K izohorno se hlade sve dok se pritisak ne smanji dva puta. Zatim se gas izobarski širi dok mu temperatura ne postane jednaka početnoj vrijednosti. Odrediti ukupnu količinu toplote koju je gas apsorbovao u ovim procesima. Univerzalna gasna konstanta je R = 8,314.

Takmičenja

Slika 1

Page 49: Zanimljiva matematika

47

3. Na horizontalnoj podlozi leži špric prečnika klipa D = 3cm i prečnika otvora d = 5mm (slika 2). Ako se klip kreće konstantnom brzinom pod dejstvom konstantne sile F = 25N, odrediti domet mlaza vode iz šprica xd Zanemariti debljinu zida šprica, gustina vode i gravitaciono ubrzanje su poznati.

Fxd

Slika 2

4. Dvije paralelene metalne ploče, koje se nalaze na rastojanju d, povezane su provodnikom. Između njih se, paralelno njima, postavi treća metalna ploča na koju je nanijeto naelektrisanje q (slika 3). Odrediti koliko naelektrisanje protekne kroz provodnik ako se treća ploča pomjeri paralelno samoj sebi za x, ostajući pri tome između druge dvije ploče.

a

d

Slika 3

5. U kolu prikazanom na slici 4 otpor otpornika C iznosi R = 40Ω, a izvor ima elekromotornu silu E = 2.5V i zanemariv unutrašnji otpor. Za otpornike A i B ne važi Omov zakon U = IR, već je veza između struje i napona data jednačinom U = α2U2, gdje je α2 = 0.02A/V2 jednaka za oba otpornika. Odrediti:a) Struju koja teče kroz otpornik A ib) Vrijednost otpora R za koju će se na otpornicima B i C oslobađati jednake

snage

E

A

B

C

Slika 4

III razred

30°

m

M

R1

R2

B

Slika 1

1. Sistem prikazan na slici 1 sastoji se od gvozdene šipke mase m = 0,1kg i električne otpornosti R1 = 0,1Ω, koja klizi bez trenja po dvijema paralelnm provodnim šinama na međusobnom rastojanju l = 1m Šine su nagnute u odnosu na horizontalnu ravan pod uglom 30º, a sa gornje strane su povezane otpornikom R2 = 1Ω. Magnetno polje indukcije B = 3T normalno je na ravan u kojoj leže šine. Šipka je neistegljivim koncem povezana sistemom koturova sa tijelom mase M = 1kg. Nakon nekog vremena tijelo se kreće konstantnom brzinom. Kolika je ova brzina?

2. U elektromotorima uvijek ima kalemova sa velikim induktivitetom, koji takođe imaju termogeni otpor. Veliki induktivitet uzrokuje i veliku faznu razliku između jačine struje i napona, odnosno mali faktor snage. Da bi se povećao faktor snage, a time i aktivna snaga, paralelno se vezuje kondenzator (slika 2). Ako su poznati kružna učestanost priključenog ulaznog napona (ω), induktivitet (L) i termogeni otpor (R) kalema, koliki kapacitet treba da ima taj kondenzator da bi faktor snage bio maksimalan?

U C

R

L

Slika 23. Tijelo miruje na strmoj ravni nagibnog ugla 36º52' koja se nalazi na horizontalnoj podlozi. strma ravan počne

harmonijski da osciluje po horizontalnoj podlozi amplitudom 1mm. Koeficijent trenja između tijela i strme ravni je 0,8. Pri kojoj minimalnoj frekvenciji oscilovanja tijelo počinje da klizi?

4. Longitudinalni talas prostire se kroz vazduh brzinom v0 = 340m/s. U nekom trenutku elongacije i brzine uočenih čestica sredine iznose x1 = 7∙10–5m, v1 = 4∙10–2m/s i x2 = 10–4m, v2 = 2∙10–2m/s Odreditia) kružnu frekvenciju ω posmatranog mehaničkog talasa,b) amplitudu oscilovanja x0 čestica pogođenih talasom,c) rastojanje Δy između ravnotežnih položaja ovih čestica.Promjena amplitude oscilovanja sa udaljenošću od izvora se može zanemariti.

Takmičenja

Page 50: Zanimljiva matematika

48

5. Sistem prikazan na slici 3 sastoji se od sirene S mase m=1kg i tri opruge istih koeficijenata elastičnosti k = 100N/m. Sirena može harmonijski da osciluje po zakonu x = x0∙sin(ωt + φ), gdje je φ početna faza. U trenutku t = 0 sirena je udaljena x1 = 5mm od svog ravnotežnog položaja i udaljava se od njega brzinom intenziteta v1 = 7cm/s. Sirena tokom oscilovanja emituje zvuk nepoznate talasne dužine λ0 Prijemnik P koji miruje, registruje razliku izmežu maksimalne i minimalne talasne dužine zvuka koji do njega dopire. Smatrajuži da nema interferencije, odredite nepoznatu talasnu dužinu λ0 Brzina zvuka u vazduhu je u = 330m/s

Pk

k k

S

Slika 3

IV razred

1. Oblak od N = 104 pozitivnih piona kreće se pod dejstvom magnetnog polja po kružnici radijusa r = 20m brzinom v = 0,99c. Pioni se raspadaju, po zakonu radioaktivnog raspada, i srednji život u njihovom sistemu im iznosi τ = 26,1ns. Izračunajte: a) Koliko piona ostane u oblaku posle jednog punog kruga? b) Koliko bi piona ostalo nakon isteka istog vremena kad se oblak ne bi kretao? Dato je c = 3∙108 m/s

2. Zbog procesa na Suncu, najvjerovatnija talasna dužina u spektru njegovog zračenja pomjera se sa 500 nm na 400 nm, a poluprečnik Sunca povećava se za 5%. Koliko će puta više energije padati na Zemlju?

3. Naći maksimalnu kinetičku energiju elektrona izbačenih sa površine litijuma elektromagnetnim zraèenjem čije se električno polje mijenja po Zakonu:E=a (1+cos(ωt)) cos(ω0t), gdje jea konstanta,ω=6∙1014s–1 iω0=36∙1014s–1 Izlazni rad za litijum iznosi 2,39eV. Poznata je konstanta h=6,62∙10–34Js.

4. Foton talasne dužine λ i elektron kreću se duž istog pravca jedan ka drugom. Pri sudaru, foton se raseje pod uglom od 180˚. Kolikom brzinom treba da se kreće elektron prije sudara da se frekvencija fotona pri sudaru ne bi promijenila? Date su potrebne konstante: h, c i me

5. Atom u kome oko protona, ne kruži elektron, već kruži negativni µ mezon naziva se mezoatomom vodonika. Masa µ mezona 207 puta je veća od mase elektrona, a naelektrisanja su im jednaka. Odredite vrijednosti rastojanja izmedju protona i mezona i energije mezoatoma koristeći Borovu teoriju. Proton i negativni µ mezon kruže oko zajedničkog centra mase. Poznate su sljedeće konstante: h = 6,62∙10–34Js,e=1,6∙10–19C,me= 9,1∙10–31kg ik=9∙109Nm2/C2

Takmičenja

Page 51: Zanimljiva matematika

48

5. Sistem prikazan na slici 3 sastoji se od sirene S mase m=1kg i tri opruge istih koeficijenata elastičnosti k = 100N/m. Sirena može harmonijski da osciluje po zakonu x = x0∙sin(ωt + φ), gdje je φ početna faza. U trenutku t = 0 sirena je udaljena x1 = 5mm od svog ravnotežnog položaja i udaljava se od njega brzinom intenziteta v1 = 7cm/s. Sirena tokom oscilovanja emituje zvuk nepoznate talasne dužine λ0 Prijemnik P koji miruje, registruje razliku izmežu maksimalne i minimalne talasne dužine zvuka koji do njega dopire. Smatrajuži da nema interferencije, odredite nepoznatu talasnu dužinu λ0 Brzina zvuka u vazduhu je u = 330m/s

Pk

k k

S

Slika 3

IV razred

1. Oblak od N = 104 pozitivnih piona kreće se pod dejstvom magnetnog polja po kružnici radijusa r = 20m brzinom v = 0,99c. Pioni se raspadaju, po zakonu radioaktivnog raspada, i srednji život u njihovom sistemu im iznosi τ = 26,1ns. Izračunajte: a) Koliko piona ostane u oblaku posle jednog punog kruga? b) Koliko bi piona ostalo nakon isteka istog vremena kad se oblak ne bi kretao? Dato je c = 3∙108 m/s

2. Zbog procesa na Suncu, najvjerovatnija talasna dužina u spektru njegovog zračenja pomjera se sa 500 nm na 400 nm, a poluprečnik Sunca povećava se za 5%. Koliko će puta više energije padati na Zemlju?

3. Naći maksimalnu kinetičku energiju elektrona izbačenih sa površine litijuma elektromagnetnim zraèenjem čije se električno polje mijenja po Zakonu:E=a (1+cos(ωt)) cos(ω0t), gdje jea konstanta,ω=6∙1014s–1 iω0=36∙1014s–1 Izlazni rad za litijum iznosi 2,39eV. Poznata je konstanta h=6,62∙10–34Js.

4. Foton talasne dužine λ i elektron kreću se duž istog pravca jedan ka drugom. Pri sudaru, foton se raseje pod uglom od 180˚. Kolikom brzinom treba da se kreće elektron prije sudara da se frekvencija fotona pri sudaru ne bi promijenila? Date su potrebne konstante: h, c i me

5. Atom u kome oko protona, ne kruži elektron, već kruži negativni µ mezon naziva se mezoatomom vodonika. Masa µ mezona 207 puta je veća od mase elektrona, a naelektrisanja su im jednaka. Odredite vrijednosti rastojanja izmedju protona i mezona i energije mezoatoma koristeći Borovu teoriju. Proton i negativni µ mezon kruže oko zajedničkog centra mase. Poznate su sljedeće konstante: h = 6,62∙10–34Js,e=1,6∙10–19C,me= 9,1∙10–31kg ik=9∙109Nm2/C2

Takmičenja

49

Rješenja zadataka iz fizike

I razred

1. (a) Iz uslova zadatka je: υ = k x Odatle se dobija da je k x cm s= =υ1 1 2

Onda je υ2 2 2 4 4= = =k x cm s cm s

(b)

1,5

0,25

0,3

0,35

0,4

0,45

0,5

0,55

2 2,5 3 3,5 4 4,5

1 sv cm

1 1v k x

=

x[cm]

(c) υυ

= = ≅∆ ∆ ∆ ∆x t t x Ptrapeza, 1

P a b h t strapeza =+ ⋅ = + ⋅ = =2

1212

14

3 9 8 1 1( ) / , ,∆

2. Nalazeći se na jednom kraju čamca čovjek treba da ga postavi tako da on drugim krajem dodiruje obalu. Onda treba da pređe na drugi kraj čamca krećući se konstantnom brzinom υ. Čamac će se pritom udaljavati od obale konstantnom brzinom V.Iz zakona održanja impulsa je: m MVυ = (1)Ako je t - vrijeme trajanja kretanja, l - dužina čamca, a x - rastojanje za koje se čamac udalji od obale za vrijeme t, onda je υ ⋅ = −t l x (2) i V t x⋅ = (3).Iz (1),(2) i (3) slijedi da je m l x M x⋅ − = ⋅( ) , odakle se dobija M m l x= −( )1 , l,x – se mogu izmjeriti pantljikom.

3. Da bi čestice gasa mogle dospjeti u detektor, mora biti ispunjen uslov φ ω= ⋅ t (1).

Vrijeme potrebno da čestice pređu rastojanje između diskova je t l= υ (2).

Odatle se dobija da je brzina čestica υ ω φ= l (3), tj. υ = 300 m/s

Zbog toga što prorezi imaju konačnu lučnu širinu Δφ, onda će u detektor ući pored čestica brzine υ i sve čestice, čije su brzine u intervalu υ υ υmin = − ∆ i υ υ υmax = + ∆

Takmičenja

Page 52: Zanimljiva matematika

50

Najsporija čestica (υmin) je ona koja kroz prvi disk prođe uz početnu (lijevu) ivicu proreza, a kroz drugi disk prođe uz krajnju (desnu) ivicu proreza. Dok ona prođe rastojanje između diskova, oni se zakrenu za ugao φ φ+ ∆

Analogno prethodnom, najbrža čestica (υmax) je ona koja kroz prvi disk prođe uz krajnju (desnu) ivicu proreza, a kroz drugi disk uz prvu (lijevu) ivicu proreza. Dok ona prođe rastojanje između diskova, oni se zakrenu za ugao φ φ− ∆ .

Slijedi da je: υ υ υω

φ φω

φ φω φ

φ φmax min ( )− = =

−−

+=

−2 2

2 2∆∆ ∆

∆∆

l l l ,

tj ∆ ∆∆

∆∆

υωφ

φ φφ φ

υφ φφ φ

=−

= ⋅−

=l1 1

27 32 2( ) ( ), m/s

II način: Čestica koja prođe uz lijevi kraj proreza prvog diska može proći uz lijevikraj drugog diska za vrijeme za koje se diskovi zarotiraju za ugao φ ili uz desni kraj drugog diska, za vrijeme za koje se diskovi zarotiraju za ugao φ φ+ ∆

Razlika brzina čestica je:

∆ ∆∆

∆∆

∆∆

υωφ

ω φφ φ φ

ω φφ φ φ

υ φφ φ

= −+

=+

=+

l l l( ) ( )

Tako se dobija da je ∆υ = 27 3, m/s

Prema tome υ υ υmin ,= − =∆ 272 7 m/s i

υ υ υmax ,= + =∆ 327 3 m/s

4. Srednja gustina sustance Sunca je

ρπS

S S

S

MV

MR

= =4 33 /

(1)

II Njutnov zakon:

m m r m MrZ ZZ S= = ⋅ω γ22 (2)

Na osnovu (1,2) slijedi:

M rS =

3 2ωγ

(3)

Po definiciji je:

α⊗ =2RrS (4) i

ωπ= 2T

(5)

Postavljajući (3 i 5) u (1), dobija se:

ρωπγ

πγα

= ⋅ =⊗

( , ) ( )1 3 3

3

2 5

3 2

34

24rR TS

(6)

Tako da se dobija na kraju ρ ≅1 1, g/cm3

5. L m R1 = υ ,

R

RSAT L I2 = ω , ω υ= / R ,

LL

m RI R

mRI

1

2

2

= =υυ /

, tj

LL

mRmR

RRsat sat

1

2

2

2

2

22 552

= =/

.

Takmičenja

51

II razred

1. Na klip djeluju sila Zemljine teže m1g, sila pritiska gasa Fg = pS, sila pritiska atmosfere Fa = p0S i sila elastičnosti opruge Fe = kx, gdje su m1–masa klipa, p i p0–pritisci u gasu i atmosferi, x – deformacija opruge i S–površina klipa. Ako pretpostavimo da je opruga istegnuta u odnosu na normalno, nedeformisano stanje, poslednja sila ima smjer kao na slici 5.Ako indeksima 1 i 2 označimo veličine vezane za temperature T1 i T2, jednačina ravnoteže na ovim temperaturama, redom možemo napisati u obliku:

p S kx m g p Sp S kx m g p S0 1 1 1

0 2 1 2

+ + =+ + =

Vodeći računa o tome da je promjena dužine opruge jednaka promjeni visine klipa, odnosno da je x x H h2 1− = − , oduzimanjem prethodnih jednačina, lako se može dobiti: S p p k x x k H h( ) ( ) ( )2 1 2 1− = − = − (1)Jednačine stanja gasa na navedenim temperaturama mogu se napisati u obliku:

pV p Sh m

MRT

p V p SH mMRT

1 1 1 1

2 2 2 2

= =

= =

Iz ovih jednačina se može dobiti:

p p mRMS

TH

Th2 1

2 1− = −

(2)

Eliminacijom S iz jednačina (1) i (2) lako se moće odrediti tražena temperatura do koje treba zagrijati gas, koja iznosi:

T HhT MkH

mRH h2 1= + −( )

Fg

m1gFcFa

slika 5

2. p-V dijagram koji odgovara opisanom ciklusu prikazan je na slici 6.Uvedimo sledeće oznake za parametre stanja gasa: 1: p1, V1, T1

2: p2= p1/2, V2=V1, T2

3: p3=p2=p1/2, V3, T1

U izohoskom procesu 1→2 važi Šarlov zakon, pa je: p

TpT

pT

T T1

1

2

2

1

22

1

2 2= = ⇒ =

Količina toplote koju apsorbuje od okoline u ovom procesu iznosi: Q nC T T nC TV V1 2 2 1 1

12− = − = −( )

Znak minus pokazuje da u ovom procesu gas u stvari ne apsorbuje toplotu nego je predaje okolini. U izobarskom procesu 2→3 gas apsorbuje od okoline količinu toplote:

Q nC T T nC T T nC TP P P2 3 3 2 1 2 112− = − = − =( ) ( )

Ukupna količina toplote koju gas apsorbuje u ovim procesima jednaka je zbiru pomenutih toplota, odnosno: Q Q Q nC T nC T n C C TV P P V= + = − + = −− −1 2 2 3 1 1 1

12

12

12

( )

Ako se iskoristi Majerova relacija C C RP V− = , može se kazati da tražena količina toplote iznosi:

Q nRT= ≈ =12

2490 2 491 J kJ.

Takmičenja

p1

V1 V3

Q2-3

Q1-2

p

V

p1/2 2

1

3

T=const.

slika 6

Page 53: Zanimljiva matematika

51

II razred

1. Na klip djeluju sila Zemljine teže m1g, sila pritiska gasa Fg = pS, sila pritiska atmosfere Fa = p0S i sila elastičnosti opruge Fe = kx, gdje su m1–masa klipa, p i p0–pritisci u gasu i atmosferi, x – deformacija opruge i S–površina klipa. Ako pretpostavimo da je opruga istegnuta u odnosu na normalno, nedeformisano stanje, poslednja sila ima smjer kao na slici 5.Ako indeksima 1 i 2 označimo veličine vezane za temperature T1 i T2, jednačina ravnoteže na ovim temperaturama, redom možemo napisati u obliku:

p S kx m g p Sp S kx m g p S0 1 1 1

0 2 1 2

+ + =+ + =

Vodeći računa o tome da je promjena dužine opruge jednaka promjeni visine klipa, odnosno da je x x H h2 1− = − , oduzimanjem prethodnih jednačina, lako se može dobiti: S p p k x x k H h( ) ( ) ( )2 1 2 1− = − = − (1)Jednačine stanja gasa na navedenim temperaturama mogu se napisati u obliku:

pV p Sh m

MRT

p V p SH mMRT

1 1 1 1

2 2 2 2

= =

= =

Iz ovih jednačina se može dobiti:

p p mRMS

TH

Th2 1

2 1− = −

(2)

Eliminacijom S iz jednačina (1) i (2) lako se moće odrediti tražena temperatura do koje treba zagrijati gas, koja iznosi:

T HhT MkH

mRH h2 1= + −( )

Fg

m1gFcFa

slika 5

2. p-V dijagram koji odgovara opisanom ciklusu prikazan je na slici 6.Uvedimo sledeće oznake za parametre stanja gasa: 1: p1, V1, T1

2: p2= p1/2, V2=V1, T2

3: p3=p2=p1/2, V3, T1

U izohoskom procesu 1→2 važi Šarlov zakon, pa je: p

TpT

pT

T T1

1

2

2

1

22

1

2 2= = ⇒ =

Količina toplote koju apsorbuje od okoline u ovom procesu iznosi: Q nC T T nC TV V1 2 2 1 1

12− = − = −( )

Znak minus pokazuje da u ovom procesu gas u stvari ne apsorbuje toplotu nego je predaje okolini. U izobarskom procesu 2→3 gas apsorbuje od okoline količinu toplote:

Q nC T T nC T T nC TP P P2 3 3 2 1 2 112− = − = − =( ) ( )

Ukupna količina toplote koju gas apsorbuje u ovim procesima jednaka je zbiru pomenutih toplota, odnosno: Q Q Q nC T nC T n C C TV P P V= + = − + = −− −1 2 2 3 1 1 1

12

12

12

( )

Ako se iskoristi Majerova relacija C C RP V− = , može se kazati da tražena količina toplote iznosi:

Q nRT= ≈ =12

2490 2 491 J kJ.

Takmičenja

p1

V1 V3

Q2-3

Q1-2

p

V

p1/2 2

1

3

T=const.

slika 6

Page 54: Zanimljiva matematika

52

F

(1)(2) v2v1

xd

patpat

slika 7

3. Pritisak u vodi neposredno uz klip (presjek 1 strujne cijevi) jednak je zbiru atmosferskog pritiska i pritiska sile F, a na otvoru (presjek 2) jednak je atmosferskom pritisku (slika 7).Pošto je strujna cijev horizontalna, Bernulijeva jednačina primijenjena na ove presjeke može se pisati u obliku:

ρ ρv p FS

v pat at12

1

22

2 2+ + = +

Ako se na iste presjeke primijeni jednačina kontinuiteta i transformiše saglasno geometrijskim odnosima, može se

dobiti: v S vS

Ddv1

2 2

1

2

2 2= =

Posle uvrštavanja ove brzine u Bernulijevu jednačinu može se izraziti brzina mlaza vode na izlazu iz šprica, koja

iznosi: v D FD d2 4 42 2=

−ρ π( )

Čestice mlaza posle izlaska iz šprica kreću se po zakonima kretanja horizontalnog hica, čiji domet iznosi:

x v hgd = 22

I očigledno zavisi od visine h sa koje polazi. Najveći domet imaju djelovi mlaza koji izlaze kroz najviši dio otvora. Primijetimo da prečnik otvora šprica nije mnogo manji od prečnika šprica (D / d = 6 : 1), pa je maksimalan domet moguće odrediti ako se uzme h D d= +( ) / 2. Međutim,

D d4 4>> pa je:

vd

F2

2 2≈ρπ

a traženi domet iznosi:

xd

F D dgd ≈+ ≈ ≈2 2 0 503 50 3( ) . .

ρ πm cm

−q1

+q1

−q2?

?

+q2+ + + + + + + + + + + + + + + + +

− − − − − − − − − − − − − − − − −

− − − − − − − −

+ + + + + + + +

? ? ?

? ? ?

−q1 +q1

−q2 +q2

+q

slika 8

4. Označimo indukovana naelektrisanja na pločama sa –q1 i –q2 (slika 8).Prikazani sistem se može posmatrati kao sistem od dva kondenzatora. Očigledno je: q q q1 2+ = (1)

Treba primijetiti da se na spoljašnje površine spoljašnjih ploča raspoređuje količina naelektrisanja , da bi ploče kao cjelina ostale neutralne kao i prije unošenja treće ploče u prostor između njih. Ova naelektrisanja se raspoređuju tako da bi pomenute ploče bile na istom potencijalu. Raspored ovih naelektrisanja nam nije vazan, pa su na slici označena upitnicima.

Pošto su spoljašnje ploče spojene provodnikom, a nalze se na istom potencijalu, pa je potencijalna razlika između unutrašnje (zajedničke) i spoljašnjih ploča ovih kondenzatora jednaka. Iz jednakosti napona na kondenzatorima slijedi:

qC

qC

1

1

2

2

= , qSa

qS

d a

1

0

2

0ε ε=

odnosno:

q a q d a1 2= −( ) (2)

Iz jednačina (1) i (2) lako se mogu odrediti naelektrisanja na kondenzatorima koja iznose:

q q ad1 = i q q d a

d2 =−

Takmičenja

53

Kada se treća pomjeri za x, na primjer ka donjoj ploči, na isti način se mogu dobiti nova naelektrisanja na pločama kondenzatora, koja iznose:

q q a xd

'1 =+ i q q d a x

d'2 =

− −

Pri navedenom pomjeranju treće ploče, kroz provodnik protekne naelektrisanje:

∆q q q q q q xd

= − = − =1 1 2 2' '

Lako se može pokazati da bi pomjeranje za isto rastojanje treće ploče ka gornjoj ploči dovelo do istog rezutata.

5. Prema postavci zadatka, ako kroz otpornike A i B protiče struja I, na njegovim krajevima se javlja napon U I= / .α Ako pretpostavimo smjerove struja kao na slici,I Kirhofovo pravilo za čvorove može biti napisano u obliku:

I I IA B C= + (1)

A II Kirhofovo pravilo za odgovarajuće konture u obliku:

EAB: EI IA B= +α α

, (2)

BC: 0 = −RII

CB

α (3)

a) Iz poslednje dvije jednačine lako se može dobiti: I R IB C= 2 2 2α (4)

I E RIA C= −α 2 2( )

Uvrštavanjem ovih vrijednosti u jednačinu (1), može se odrediti struja kroz otpornik C, koja iznosi:

I EREC = +

αα

2 2

21,

pa tražena struja kroz otpornik A iznosi:

I E RERE

A mAA =++

= =α

αα

2 22

2

11 2

0 045 45.

b) Pošto su otpornici B i C vezani paralelno, naponi na njima su jednaki, pa se na njima oslobađaju jednake snage ukoliko kroz njih teku iste struje, odnosno, ako je I IB C= . Ako iskoristimo ovu jednakost u jednačini (4), lako se može dobiti da je u tom slučaju: I I

RB C= = 12 2α

a ako isto iskoristimo u jednačini (1), može se dobiti da je tada: I IA B= 2

Ako se ove vrijednosti uvrste u jednačinu (2), možemo dobiti traženu otpornost otpornika A, koja iznosi:

RE

= + =2 1 48 32α. Ω

III razred

1. Kada se pusti tijelo mase M, ono se spušta ubrzano. Kako je ovaj teg neistegljivim koncem povezan sa šipkom i šipka će se kretati ubrzano. Zbog kretanja šipke doći će do pojave indukovane struje, tako da će na provodnik djelovati i magnetno polje. Ubrzanje sistema će se smanjivati i kada ono bude jednako nuli tijelo M će se kretati stalnom brzinom. Na slici r1 označene su sile koje djeluju u sistemu. Sile F T FA/ / , ,

→ → →, djeluju na šipku. Kako se šipka

kreće stalnom brzinom vm, rezultanta ovih sila mora biti jednaka nuli, pa je IBl mg T= +sinα (1)

Na tijelo djeluju sile 2T i Mg. njihova rezulatanta je takođe jednaka nuli, pa je 2T = Mg, tj T Mg= / 2 (2)

Takmičenja

IA

ICIB

E

A

B

C

Slika 9

Page 55: Zanimljiva matematika

52

F

(1)(2) v2v1

xd

patpat

slika 7

3. Pritisak u vodi neposredno uz klip (presjek 1 strujne cijevi) jednak je zbiru atmosferskog pritiska i pritiska sile F, a na otvoru (presjek 2) jednak je atmosferskom pritisku (slika 7).Pošto je strujna cijev horizontalna, Bernulijeva jednačina primijenjena na ove presjeke može se pisati u obliku:

ρ ρv p FS

v pat at12

1

22

2 2+ + = +

Ako se na iste presjeke primijeni jednačina kontinuiteta i transformiše saglasno geometrijskim odnosima, može se

dobiti: v S vS

Ddv1

2 2

1

2

2 2= =

Posle uvrštavanja ove brzine u Bernulijevu jednačinu može se izraziti brzina mlaza vode na izlazu iz šprica, koja

iznosi: v D FD d2 4 42 2=

−ρ π( )

Čestice mlaza posle izlaska iz šprica kreću se po zakonima kretanja horizontalnog hica, čiji domet iznosi:

x v hgd = 22

I očigledno zavisi od visine h sa koje polazi. Najveći domet imaju djelovi mlaza koji izlaze kroz najviši dio otvora. Primijetimo da prečnik otvora šprica nije mnogo manji od prečnika šprica (D / d = 6 : 1), pa je maksimalan domet moguće odrediti ako se uzme h D d= +( ) / 2. Međutim,

D d4 4>> pa je:

vd

F2

2 2≈ρπ

a traženi domet iznosi:

xd

F D dgd ≈+ ≈ ≈2 2 0 503 50 3( ) . .

ρ πm cm

−q1

+q1

−q2?

?

+q2+ + + + + + + + + + + + + + + + +

− − − − − − − − − − − − − − − − −

− − − − − − − −

+ + + + + + + +

? ? ?

? ? ?

−q1 +q1

−q2 +q2

+q

slika 8

4. Označimo indukovana naelektrisanja na pločama sa –q1 i –q2 (slika 8).Prikazani sistem se može posmatrati kao sistem od dva kondenzatora. Očigledno je: q q q1 2+ = (1)

Treba primijetiti da se na spoljašnje površine spoljašnjih ploča raspoređuje količina naelektrisanja , da bi ploče kao cjelina ostale neutralne kao i prije unošenja treće ploče u prostor između njih. Ova naelektrisanja se raspoređuju tako da bi pomenute ploče bile na istom potencijalu. Raspored ovih naelektrisanja nam nije vazan, pa su na slici označena upitnicima.

Pošto su spoljašnje ploče spojene provodnikom, a nalze se na istom potencijalu, pa je potencijalna razlika između unutrašnje (zajedničke) i spoljašnjih ploča ovih kondenzatora jednaka. Iz jednakosti napona na kondenzatorima slijedi:

qC

qC

1

1

2

2

= , qSa

qS

d a

1

0

2

0ε ε=

odnosno:

q a q d a1 2= −( ) (2)

Iz jednačina (1) i (2) lako se mogu odrediti naelektrisanja na kondenzatorima koja iznose:

q q ad1 = i q q d a

d2 =−

Takmičenja

53

Kada se treća pomjeri za x, na primjer ka donjoj ploči, na isti način se mogu dobiti nova naelektrisanja na pločama kondenzatora, koja iznose:

q q a xd

'1 =+ i q q d a x

d'2 =

− −

Pri navedenom pomjeranju treće ploče, kroz provodnik protekne naelektrisanje:

∆q q q q q q xd

= − = − =1 1 2 2' '

Lako se može pokazati da bi pomjeranje za isto rastojanje treće ploče ka gornjoj ploči dovelo do istog rezutata.

5. Prema postavci zadatka, ako kroz otpornike A i B protiče struja I, na njegovim krajevima se javlja napon U I= / .α Ako pretpostavimo smjerove struja kao na slici,I Kirhofovo pravilo za čvorove može biti napisano u obliku:

I I IA B C= + (1)

A II Kirhofovo pravilo za odgovarajuće konture u obliku:

EAB: EI IA B= +α α

, (2)

BC: 0 = −RII

CB

α (3)

a) Iz poslednje dvije jednačine lako se može dobiti: I R IB C= 2 2 2α (4)

I E RIA C= −α 2 2( )

Uvrštavanjem ovih vrijednosti u jednačinu (1), može se odrediti struja kroz otpornik C, koja iznosi:

I EREC = +

αα

2 2

21,

pa tražena struja kroz otpornik A iznosi:

I E RERE

A mAA =++

= =α

αα

2 22

2

11 2

0 045 45.

b) Pošto su otpornici B i C vezani paralelno, naponi na njima su jednaki, pa se na njima oslobađaju jednake snage ukoliko kroz njih teku iste struje, odnosno, ako je I IB C= . Ako iskoristimo ovu jednakost u jednačini (4), lako se može dobiti da je u tom slučaju: I I

RB C= = 12 2α

a ako isto iskoristimo u jednačini (1), može se dobiti da je tada: I IA B= 2

Ako se ove vrijednosti uvrste u jednačinu (2), možemo dobiti traženu otpornost otpornika A, koja iznosi:

RE

= + =2 1 48 32α. Ω

III razred

1. Kada se pusti tijelo mase M, ono se spušta ubrzano. Kako je ovaj teg neistegljivim koncem povezan sa šipkom i šipka će se kretati ubrzano. Zbog kretanja šipke doći će do pojave indukovane struje, tako da će na provodnik djelovati i magnetno polje. Ubrzanje sistema će se smanjivati i kada ono bude jednako nuli tijelo M će se kretati stalnom brzinom. Na slici r1 označene su sile koje djeluju u sistemu. Sile F T FA/ / , ,

→ → →, djeluju na šipku. Kako se šipka

kreće stalnom brzinom vm, rezultanta ovih sila mora biti jednaka nuli, pa je IBl mg T= +sinα (1)

Na tijelo djeluju sile 2T i Mg. njihova rezulatanta je takođe jednaka nuli, pa je 2T = Mg, tj T Mg= / 2 (2)

Takmičenja

IA

ICIB

E

A

B

C

Slika 9

Page 56: Zanimljiva matematika

54

Struja koja se indukuje zbog kretanja provodnika je

30°

Mg

R2

B

FaFr

T

TT

T T

mg

Slika r1

IR R

BlvR R

i m=+

=+

ε

1 2 1 2

(3)

Zamjenom (2) i (3) u (1) i sređivanjem dobija se

v R RB l

g m Mm =

+ +

=1 2

2 2 20 66sin ,α

ms

Kako je brzina tijela vM = vm/2, za brzinu tijela se dobija

v R RB l

g m MM = + +

=1 2

2 22 20 33sin ,α

ms

2. Maksimalni faktor snage dobija se za slučaj kad je fazna razlika između struje i napona u kolu jednaka nuli. na slici r2 prikazan je odgovarajući vektorski dijagram. Sa slike se vidi da je:

I I1 2= sinφ ili U C UR L

ωω

φ=+2 2( )

sin

φ je fazni pomjeraj između struje kroz granu sa kalemom i napona na krajevima te grane, pa je

sin( ) ( ) ( )

φω

ωω

ω

ω

ω=

+=

+→ =

+ +U

U UL

L RU C U

R LL

R LL

L R2 2 2 2 2 2 2 2

Iz prethodnih izraza slijedi da je,

C LR L

=+2 2( )ω

FiFtr

F n

F1

a

mg

F in

α

Fi1

Slika r3

3. Kada strma ravan miruje, tijelo na njoj takođe miruje i intenzitet maksimalne sile statičkog trenja je veći od komponente sile zemljine teže koja bi pomjerala tijelo niz strmu ravan.Ako strma ravan osciluje, na tijelo djeluje inercijalna sila koja je periodična. Amplitudna vrijednost te sile je F ma mxi i0 0 0

2= = ωDa bi tijelo počelo da klizi, potrebno je da komponenta inercijalne sile koja je paralelna sa strmom ravni ispunjava uslov (slika r3), F F Fi t0|| ||≥ −gdje je F mg Ft i= −µ α α( cos sin )0 U graničnom slučaju je mx m g x mg0

20

2ω α µ α ω α αcos ( cos sin ) sin .= − −

Sređujući prethodnu jednaćinu i stavljajući ω πν= 2 min dobija se

νπ

µ α αα µ αmin

( cos sin )(cos sin )

,= −+

=12

2 80

gx

Hz

4. a) U trenutku t elongacije su date sa x x t ky1 0 1= −cos( )ω i x x t ky2 0 2= −cos( ),ω a odgovarajuće brzine v x t ky1 0 1= − −ω ωsin( ) i v x t ky2 0 2= − −ω ωsin( ). Kvadriranjem i oduzimanjem prethodnih jednačina dobija se

v v x x v vx x1

222 2

22

12 1

222

22

12 485− = − → = −

−=ω ω( ) rad/s

b) Kvadriranjem i sabiranjem izraza za x1 i v1, ili za x2 i v2, uz korišćenje osnovnog trigonometrijskog identiteta dobijamo

x x v x v0 12

12 2

22

22 2 41 08 10= + = + = ⋅ −/ / ,ω ω m

c) Faze oscilovanja uočenih tačaka su ωt ky x x− =1 1 0arccos( / ) i ωt ky x x− =2 2 0arccos( / )

Oduzimanjem prethodnih jednačina dobijamo ∆y y y k x x x x= − = −2 1 1 0 2 01 / (arccos( / ) arccos( / ))

Takmičenja

U

I

I1 I2C

R

L

φ I

I1

I2

U=UC

UR

UL

U

I

I1 I2C

R

L

φ I

I1

I2

U=UC

UR

UL

Slika r2

55

Pošto je k = 2π λ/ i v0 = λν , onda je k v=ω / 0 , pa dobijamo ∆y = 0 335, m5. Minimalana talasna dužina zvuka na prijemniku je

λ λ λω

minmax= − = −

0 00u v

uu xu

, a maksimalna talasna dužina iznosi

λ λ λω

maxmax= + = +

0 00u v

uu xu

. Odavde je λ λω002

= ux∆

Kako je ekvivalentni koeficijent elastičnosti 3k, slijedi da je ω 2 3= k m/ Iz x x1 0= sinϕ i v x1 0= ω φcos , slijedi tan /φ ω= x v1 1

Koristeći relaciju sin tantan

φφ

φ=

+1 2, za amplitudu oscilovanja dobijamo

x v x0 12

12 2= + ω ω/ , pa je λ λ

ω

λ0

12

12 2

12

122 2 3

1 5=+

=+

≈uv x

uv kx m

∆ ∆/

, m

IV razred

1. a) U laboratorijskom sistemu prođe vrijeme τ π= ⋅ ⋅ =2 423 2rv

ns, .

U sistemu piona tom vremenu odgovara vremenski interval t t vc

ns' , .= − =1 59 692

2

N N e NN

t

= ⇒ = =−

00

0 102 10 2'

, , %.τ Nakon jednog okreta ostaje N=1020 piona.

b) Kad bi u sistemu piona prošao isti period vremena kao u laboratorijskom sistemu, imali bismo NN

et

0

89 12 10= = ⋅− −τ , .

Piona dakle, nebi ni bilo.2. Na početku procesa snaga zračenja koja pada na Zemlju je:

Φ Φ Ω1 14

2

22

4= = ⋅ ⋅ ⋅S ST R

Rr

πσ π , a na kraju Φ Φ Ω2 2

42

22

4= = ⋅ ⋅ ⋅' 'S ST R

Rr

πσ π

Deobom se dobija: ΦΦ

1

2

2

1

4 2

=

TT

RRS

S

' .

Kako je: λ λ1 1 2 2⋅ = ⋅ =T T b , slijedi: ΦΦ

1

2

2

1

4 21 05 2 7=

λ, , .RR

S

S

3. Zakon po kome se mijenja električno polje datog zračenja može se napisati u obliku:

E a t a t a t= ⋅ + ⋅ ⋅ + + ⋅ ⋅ −cos( ) cos(( ) ) cos(( ) ).ω ω ω ω ω0 0 012

12

U tom zračenju su zastupljeni fotoni frekvencija:

νωπ10 14

25 73 10= = ⋅, Hz

νω ω

π20 14

26 68 10= + = ⋅, Hz

νω ω

π30 14

24 77 10= − = ⋅, Hz

Najmanja frekvencija zračenja koje može izazvati fotoefekat sa površine litijuma je ν 0145 77 10= = ⋅A

hHzi , .

Fotoefekat mogu izazvati samo fotoni frekvencije ν2. Tada je maksimalna kinetička energija fotoelektrona: T h A eVi= − =ν 0 37, .

Takmičenja

Page 57: Zanimljiva matematika

55

Pošto je k = 2π λ/ i v0 = λν , onda je k v=ω / 0 , pa dobijamo ∆y = 0 335, m5. Minimalana talasna dužina zvuka na prijemniku je

λ λ λω

minmax= − = −

0 00u v

uu xu

, a maksimalna talasna dužina iznosi

λ λ λω

maxmax= + = +

0 00u v

uu xu

. Odavde je λ λω002

= ux∆

Kako je ekvivalentni koeficijent elastičnosti 3k, slijedi da je ω 2 3= k m/ Iz x x1 0= sinϕ i v x1 0= ω φcos , slijedi tan /φ ω= x v1 1

Koristeći relaciju sin tantan

φφ

φ=

+1 2, za amplitudu oscilovanja dobijamo

x v x0 12

12 2= + ω ω/ , pa je λ λ

ω

λ0

12

12 2

12

122 2 3

1 5=+

=+

≈uv x

uv kx m

∆ ∆/

, m

IV razred

1. a) U laboratorijskom sistemu prođe vrijeme τ π= ⋅ ⋅ =2 423 2rv

ns, .

U sistemu piona tom vremenu odgovara vremenski interval t t vc

ns' , .= − =1 59 692

2

N N e NN

t

= ⇒ = =−

00

0 102 10 2'

, , %.τ Nakon jednog okreta ostaje N=1020 piona.

b) Kad bi u sistemu piona prošao isti period vremena kao u laboratorijskom sistemu, imali bismo NN

et

0

89 12 10= = ⋅− −τ , .

Piona dakle, nebi ni bilo.2. Na početku procesa snaga zračenja koja pada na Zemlju je:

Φ Φ Ω1 14

2

22

4= = ⋅ ⋅ ⋅S ST R

Rr

πσ π , a na kraju Φ Φ Ω2 2

42

22

4= = ⋅ ⋅ ⋅' 'S ST R

Rr

πσ π

Deobom se dobija: ΦΦ

1

2

2

1

4 2

=

TT

RRS

S

' .

Kako je: λ λ1 1 2 2⋅ = ⋅ =T T b , slijedi: ΦΦ

1

2

2

1

4 21 05 2 7=

λ, , .RR

S

S

3. Zakon po kome se mijenja električno polje datog zračenja može se napisati u obliku:

E a t a t a t= ⋅ + ⋅ ⋅ + + ⋅ ⋅ −cos( ) cos(( ) ) cos(( ) ).ω ω ω ω ω0 0 012

12

U tom zračenju su zastupljeni fotoni frekvencija:

νωπ10 14

25 73 10= = ⋅, Hz

νω ω

π20 14

26 68 10= + = ⋅, Hz

νω ω

π30 14

24 77 10= − = ⋅, Hz

Najmanja frekvencija zračenja koje može izazvati fotoefekat sa površine litijuma je ν 0145 77 10= = ⋅A

hHzi , .

Fotoefekat mogu izazvati samo fotoni frekvencije ν2. Tada je maksimalna kinetička energija fotoelektrona: T h A eVi= − =ν 0 37, .

Takmičenja

Page 58: Zanimljiva matematika

56

4. Po zakonu održanja energije h T h Te eν ν+ = +' ' , a kako je, po uslovu zadatka ν ν= ' , slijedi T Te e' = , tj. Intenzitet

brzine se ne mijenja, ali se po zakonu održanja impulsa, mijenja njen smjer:hc

p p hc

p hce e e

ν ν ν− = − = −' ' Slijedi p hc

h mvvc

he = =

−=ν

λ ν;1

2

2

Odatle se nalazi v c

Ehc

=

+

1 02λ

, gdje je E0 energija mirovanja elektrona.

5. U mezoatomu čestice kruže oko zajedničkog centra mase uslijed privlačne elektrostatičke interakcije. Rastojanja čestica od centra mase su r1 i r2, pa važi relacija: m r m r1 1 2 2= , a rastojanje između čestica je: d r r= +1 2 . Slijedi:

r m dm m

r m dm m1

2

1 22

1

1 2

=+

=+

;

Jednačine kruženja čestica oko centra mase su:

m v

rk ed

m vr

k ed

112

1

2

2

222

2

2

2

⋅ =

⋅ = odnosno

mm

m m v k ed

mm

m m v k ed

1

21 2 1

22

2

11 2 2

22

⋅ + ⋅ = ⋅

⋅ + ⋅ = ⋅

( )

( ) (1)

Po Borovom postulatu kvantovanja momenta impulsa važi jednačina:

L m v r m v r n h n= ⋅ ⋅ + ⋅ ⋅ = ⋅⋅

=1 1 1 2 2 2 21 2 3

π, , , ....

Odatle slijedi:

v v hm r1 21 12

+ =⋅ ⋅ ⋅π

,

a kako je:

vr

vr

1

1

2

2

= , to je: v rr

n hm r1

2

1 1 1

12

⋅ +

= ⋅

⋅ ⋅ ⋅π; slijedi

v n hm r r

n hm d1

1 1 2 12 2= ⋅

⋅ ⋅ ⋅ += ⋅

⋅ ⋅ ⋅π π( );

slično tome: v n hm d2

22= ⋅

⋅ ⋅ ⋅π; (2)

Iz jednačina (1) i (2) dobija se: d n m m hm m k e

n m n= + ⋅⋅ ⋅ ⋅ ⋅ ⋅

= ⋅ ⋅ =−2 1 22

21 2

22 13

42 84 10 1 2 3( ) , : , , , ...

π

Ukupna energija mezoatoma vodonika je:

E m v m v k ed

nk eh

nke

= ⋅ ⋅ + ⋅ ⋅ − ⋅

= − ⋅ ⋅ ⋅ ⋅

= − ⋅

12

12

1 2

1 2 54

1 12

2 22

2

2

2 2 4

2

2

π µ

, VV n, , , ,...=1 2 3

Takmičenja

Page 59: Zanimljiva matematika

56

4. Po zakonu održanja energije h T h Te eν ν+ = +' ' , a kako je, po uslovu zadatka ν ν= ' , slijedi T Te e' = , tj. Intenzitet

brzine se ne mijenja, ali se po zakonu održanja impulsa, mijenja njen smjer:hc

p p hc

p hce e e

ν ν ν− = − = −' ' Slijedi p hc

h mvvc

he = =

−=ν

λ ν;1

2

2

Odatle se nalazi v c

Ehc

=

+

1 02λ

, gdje je E0 energija mirovanja elektrona.

5. U mezoatomu čestice kruže oko zajedničkog centra mase uslijed privlačne elektrostatičke interakcije. Rastojanja čestica od centra mase su r1 i r2, pa važi relacija: m r m r1 1 2 2= , a rastojanje između čestica je: d r r= +1 2 . Slijedi:

r m dm m

r m dm m1

2

1 22

1

1 2

=+

=+

;

Jednačine kruženja čestica oko centra mase su:

m v

rk ed

m vr

k ed

112

1

2

2

222

2

2

2

⋅ =

⋅ = odnosno

mm

m m v k ed

mm

m m v k ed

1

21 2 1

22

2

11 2 2

22

⋅ + ⋅ = ⋅

⋅ + ⋅ = ⋅

( )

( ) (1)

Po Borovom postulatu kvantovanja momenta impulsa važi jednačina:

L m v r m v r n h n= ⋅ ⋅ + ⋅ ⋅ = ⋅⋅

=1 1 1 2 2 2 21 2 3

π, , , ....

Odatle slijedi:

v v hm r1 21 12

+ =⋅ ⋅ ⋅π

,

a kako je:

vr

vr

1

1

2

2

= , to je: v rr

n hm r1

2

1 1 1

12

⋅ +

= ⋅

⋅ ⋅ ⋅π; slijedi

v n hm r r

n hm d1

1 1 2 12 2= ⋅

⋅ ⋅ ⋅ += ⋅

⋅ ⋅ ⋅π π( );

slično tome: v n hm d2

22= ⋅

⋅ ⋅ ⋅π; (2)

Iz jednačina (1) i (2) dobija se: d n m m hm m k e

n m n= + ⋅⋅ ⋅ ⋅ ⋅ ⋅

= ⋅ ⋅ =−2 1 22

21 2

22 13

42 84 10 1 2 3( ) , : , , , ...

π

Ukupna energija mezoatoma vodonika je:

E m v m v k ed

nk eh

nke

= ⋅ ⋅ + ⋅ ⋅ − ⋅

= − ⋅ ⋅ ⋅ ⋅

= − ⋅

12

12

1 2

1 2 54

1 12

2 22

2

2

2 2 4

2

2

π µ

, VV n, , , ,...=1 2 3

Takmičenja

57

InformatikaZadaci

IV razred

1. Jedan prozor se sastoji od ivice koju prikazujemo simbolima '-' (minus), '|' (vertikalna crta) i '+' (plus), od unutrašnjosti prozora koja je popunjena znakovima '.' (tačka), a na sredini gornje ivice prozora nalazi se naziv prozora između dvije vertikalne crte. Naziv će biti centriran ili malo lijevo ako ga je nemoguće točno centrirati. Rastojanje između prvog slova naziva i lijeve ivice prozora biće jednaka ili za tačno jedan manje od rastojanja posljednjeg slova naziva od desne ivice prozora. Drugim riječima, naziv će biti centriran koliko je to moguće. Naziv prozora sastoji se od malih slova engleske abecede. Svaki prozor je dovoljno širok da cio naziv stane na gornju ivicu prozora zajedno sa susjednim znakovima '-|' slijeva tj. '|-' zdesna. Širina prozora biće barem za 6 kolona veća od dužine naziva. Visina svakog prozora biće najmanje tri reda, a neće postojati dva prozora s istim nazivom. Dat je izgled ekrana s više prozora, tako da se nijedna dva prozora ne prekrivaju (ni djelomično ni potpuno). Prozore treba složiti u takozvani "cascade mode" na sljedeći način: • visina i širina svih prozora mora ostati nepromijenjena • gornja lijeva ivica prvog prozora treba biti u gornjem lijevom uglu ekrana • svaki sljedeći prozor treba prekrivati prethodni i biti pomaknut točno za jedan red dolje i za jednu kolonu udesno • prozori trebaju biti poredani abecedno po nazivu – prozor čiji je naziv prvi po abecedi ide u prvi red, drugi po abecedi u drugi red itd

Ulazni podaci U prvom nalaze se prirodni brojevi M i N, 100,10≤≤NM broj redova i broj kolona ekrana. U svakom od sljedećih M redova nalazi se po N karaktera koji predstavljaju izgled ekrana. Naziv svakog prozora sastojaće se od najmanje jednog a najviše 10 malih slova engleske abecede. Brojevi M i N biće dovoljno veliki da svi prozori poslije slaganja stanu na ekran.

Izlazni podaci U svaki od M redova potrebno je štampati po N karaketra tj. traženi izgled ekrana nakon slaganja prozora.

Primjeri podataka za testiranje

ulaz12 12.............+-|ana|-+...|.......|...|.......|...|.......|...+-------+....+-|bozo|-+..|........|..|........|..|........|..|........|..+--------+

izlaz+-|ana|-+...|+-|bozo|-+.||........|.||........|.+|........|..|........|..+--------+.............................................................

ulaz15 22...............+-|a|-+.+-|winamp|--+.|.....|.|...........|.|.....|.|...........|.|.....|.|...........|.|.....|.|...........|.|.....|.+-----------+.|.....|...............|.....|+----|kit|----+|.....||.............||.....||.............||.....||.............||.....||.............|+-----+|.............|.......+-------------+.......

izlaz+-|a|-+...............|+----|kit|----+......||+-|winamp|--+|......|||...........||......|||...........||......|||...........||......|||...........||......|++-----------++......|.....|...............|.....|...............|.....|...............|.....|...............+-----+...........................................................

ulaz14 15+---|a|---+....|.........|....|.........|....+---------+..................................+--|ab|---+....|.........|....|.........|....+---------+....+--|abc|--+....|.........|....|.........|....+---------+....

izlaz+---|a|---+....|+--|ab|---+...||+--|abc|--+..+||.........|...+|.........|....+---------+..........................................................................................................................

Takmičenja

Page 60: Zanimljiva matematika

58

2. U vatrogasnu jedinicu stiglo je saopštenje o mogućem požaru u jednom sektoru guste šume oblika kvadrata. Da bi pronašli mjesto požara, dežurni je pozvao vazdušnu službu osmatranja i poslao N aviona da pretraže dati sektor I ustanove tačno mjesto požara. Nijedan od aviona nije prijavio požar. Poznato je da se avion kreće pravolinijski i pilot vidi pojas šume širine 50km lijevo i desno od linije na površini zemlje nad kojom se kreće avion (vidi sliku), gdje su tačke koje su udaljene tačno 50km od linije vidljive iz aviona.Po povratku u bazu, svaki od pilota je prijavio tačku (xb,yb) u kojoj je ušao u sektor šume i tačku (xe,ye) u kojoj je napustio sektor. Između te dvije tačke avion se kretao pravolinijski.Treba napsiati program koji na osnovu izvještaja pilota određuje da li je stvarno čitav kvadrat šume bio pregledan. Ako nije, treba naći koordinatu bar jedne tačke u kvadratu ili njegovim stranicama koja nije obuhvaćena pregledom iz aviona.

Ulazni podaciUlazna datoteka ima N+2 reda. U prvom redu upisana je dužina L stranice kvadrata u kilometrima, (0 < L ≤ 1000). U drugom redu ulaza je prirodan broj N – broj aviona koji pretražuju šumu, 1≤ N ≤100. U svakom od sljedećih N redova upisana su po četiri realna broja – koordinate xb, yb, xe, ye, razdvojene jednom bjelinom. Koordinate su zadate u kilometrima. Donji lijevi ugao kvadrata ima koordinate (0, 0) a gornji desni ugao kvadrata ima koordinate (L. L).

Izlazni podaci Izlazna da sati u kilometrima, sa tačnošću od 1 metra.

Primjeri test podataka ulaz 245 1 26.1 0 193.568 245

izlaz155.123 100

3. Na kružnoj autobuskoj liniji dužine S kreće se N autobusa koji su označeni brojevima 1, 2, ..., N, po redosljedu polazaka. Autobus broj 1 kreće se iza autobusa broj N. Autobusi se kreću istom brzinom V0 i sa jednakim vremenskim intervalima između autobusa. Dispečer u podne istovremeno uklanja sa linije K autobusa i šalje vozače tih autobusa na ručak. Da bi se vratili jednaki intervali kretanja između autobusa potrebno je neko vrijeme T i, možda, promjena brzine kretanja nekih autobusa koji su ostali na liniji. Za vrijeme tog perioda, autobusi se moraju kretati konstantnim brzinama iz intervala [Vmin, Vmax], pri čemu brzine zadaje dispečer. Promjena brzine izvodi se trenutno. Poslije isteka vremena T, autobusi ponovo nastavljaju da se kreću brzinom V0 Vaš zadatak je da napišete program za automatskog dispečera, koji izračunava minimalno vrijeme Tmin za koje intervali između autobusa postaju jednaki i brzinu kretanja svakog od autobusa.

Ulazni podaci U prvom redu ulazne datoteke nalaze se prirodni brojevi N, K, S, Vmin, Vmax i V0, gdje je K<N≤10000, S≤10000, Vmin<Vmax≤10000, Vmin≤V0≤Vmax). U drugom redu su u rastućem poretku dati K brojeva autobusa koji se uklanjaju sa linije u podne. Svi podaci u razdvojeni jednom bjelinom.

Izlazni podaci U prvom redu izlazne datoteke treba upisati vrijeme Tmin. U svakom od sljedećih N-K redova upisati po dva broja razdvojena bjelinom: broj autobusa i brzina kretanja autobusa u toku inetrvala Tmin

Primjeri test podataka ulaz ulaz4 1 60 21 70 60 3 4 2 40 30 80 50 2 4

izlaz izlaz0.2041 1 45.5 2 21 4 70 0 1 50 3 50

Takmičenja

59

Rješenja zadataka iz informatike

IV razred

1. Opis algoritma: Program se odvija u sljedecim koracima: 1. Ucitamo sve ulazne podatke u matricu. 2. Nadjemo sve prozore, odredimo im sirinu, visinu i naslov. Usput brisemo sve prozore iz matrice (dovoljno je brisati samo ivice prozora, jer je unutrasnjost prazna). Sad je matrica prazna (sadrzi samo tacke).3. Sortiramo sve prozore po naslovu (bilo koji algoritam sortiranja je dobar) 4. Crtamo sve prozore u matricu, tako sto crtamo i unutrasnjost proz, jer na taj nacin brisemo dejlove prozora koji su ispod 5. Stampamo sadrzaj matrice.Slozenost rjesenja je O(N3)

program kaskade; const MAXN = 100; MAXM = 100; MAXP = 100; type Tnaslov = string[10]; type Tprozor = record naslov : Tnaslov; sir, vis : longint; end;

var a : array[1..MAXN+MAXN, 1..MAXM+MAXM] of char; p : array[1..MAXP] of Tprozor; temp : Tprozor; n, m, np, i, j, k : longint;

begin (* ucitavanje... *) readln(n, m); for i:=1 to n do begin for j:=1 to m do read(a[i, j]); readln; end; (* ...ucitavanje *) (* trazenje prozora... *) np := 0; for i:=1 to n do for j:=1 to m do if a[i, j]='+' then begin np:=np+1; (* trazimo sirinu... *) for k:=j+1 to m do if a[i, k]='+' then begin p[np].sir := k-j+1; break; end; (* trazimo visinu... *) for k:=i+1 to n do if a[k, j]='+' then begin p[np].vis := k-i+1; break; end;

(* spremamo naslov... *) p[np].naslov := ''; k:=j+1; while a[i, k]<>'|' do k:=k+1; k:=k+1; while a[i, k]<>'|' do

Takmičenja

Page 61: Zanimljiva matematika

58

2. U vatrogasnu jedinicu stiglo je saopštenje o mogućem požaru u jednom sektoru guste šume oblika kvadrata. Da bi pronašli mjesto požara, dežurni je pozvao vazdušnu službu osmatranja i poslao N aviona da pretraže dati sektor I ustanove tačno mjesto požara. Nijedan od aviona nije prijavio požar. Poznato je da se avion kreće pravolinijski i pilot vidi pojas šume širine 50km lijevo i desno od linije na površini zemlje nad kojom se kreće avion (vidi sliku), gdje su tačke koje su udaljene tačno 50km od linije vidljive iz aviona.Po povratku u bazu, svaki od pilota je prijavio tačku (xb,yb) u kojoj je ušao u sektor šume i tačku (xe,ye) u kojoj je napustio sektor. Između te dvije tačke avion se kretao pravolinijski.Treba napsiati program koji na osnovu izvještaja pilota određuje da li je stvarno čitav kvadrat šume bio pregledan. Ako nije, treba naći koordinatu bar jedne tačke u kvadratu ili njegovim stranicama koja nije obuhvaćena pregledom iz aviona.

Ulazni podaciUlazna datoteka ima N+2 reda. U prvom redu upisana je dužina L stranice kvadrata u kilometrima, (0 < L ≤ 1000). U drugom redu ulaza je prirodan broj N – broj aviona koji pretražuju šumu, 1≤ N ≤100. U svakom od sljedećih N redova upisana su po četiri realna broja – koordinate xb, yb, xe, ye, razdvojene jednom bjelinom. Koordinate su zadate u kilometrima. Donji lijevi ugao kvadrata ima koordinate (0, 0) a gornji desni ugao kvadrata ima koordinate (L. L).

Izlazni podaci Izlazna da sati u kilometrima, sa tačnošću od 1 metra.

Primjeri test podataka ulaz 245 1 26.1 0 193.568 245

izlaz155.123 100

3. Na kružnoj autobuskoj liniji dužine S kreće se N autobusa koji su označeni brojevima 1, 2, ..., N, po redosljedu polazaka. Autobus broj 1 kreće se iza autobusa broj N. Autobusi se kreću istom brzinom V0 i sa jednakim vremenskim intervalima između autobusa. Dispečer u podne istovremeno uklanja sa linije K autobusa i šalje vozače tih autobusa na ručak. Da bi se vratili jednaki intervali kretanja između autobusa potrebno je neko vrijeme T i, možda, promjena brzine kretanja nekih autobusa koji su ostali na liniji. Za vrijeme tog perioda, autobusi se moraju kretati konstantnim brzinama iz intervala [Vmin, Vmax], pri čemu brzine zadaje dispečer. Promjena brzine izvodi se trenutno. Poslije isteka vremena T, autobusi ponovo nastavljaju da se kreću brzinom V0 Vaš zadatak je da napišete program za automatskog dispečera, koji izračunava minimalno vrijeme Tmin za koje intervali između autobusa postaju jednaki i brzinu kretanja svakog od autobusa.

Ulazni podaci U prvom redu ulazne datoteke nalaze se prirodni brojevi N, K, S, Vmin, Vmax i V0, gdje je K<N≤10000, S≤10000, Vmin<Vmax≤10000, Vmin≤V0≤Vmax). U drugom redu su u rastućem poretku dati K brojeva autobusa koji se uklanjaju sa linije u podne. Svi podaci u razdvojeni jednom bjelinom.

Izlazni podaci U prvom redu izlazne datoteke treba upisati vrijeme Tmin. U svakom od sljedećih N-K redova upisati po dva broja razdvojena bjelinom: broj autobusa i brzina kretanja autobusa u toku inetrvala Tmin

Primjeri test podataka ulaz ulaz4 1 60 21 70 60 3 4 2 40 30 80 50 2 4

izlaz izlaz0.2041 1 45.5 2 21 4 70 0 1 50 3 50

Takmičenja

59

Rješenja zadataka iz informatike

IV razred

1. Opis algoritma: Program se odvija u sljedecim koracima: 1. Ucitamo sve ulazne podatke u matricu. 2. Nadjemo sve prozore, odredimo im sirinu, visinu i naslov. Usput brisemo sve prozore iz matrice (dovoljno je brisati samo ivice prozora, jer je unutrasnjost prazna). Sad je matrica prazna (sadrzi samo tacke).3. Sortiramo sve prozore po naslovu (bilo koji algoritam sortiranja je dobar) 4. Crtamo sve prozore u matricu, tako sto crtamo i unutrasnjost proz, jer na taj nacin brisemo dejlove prozora koji su ispod 5. Stampamo sadrzaj matrice.Slozenost rjesenja je O(N3)

program kaskade; const MAXN = 100; MAXM = 100; MAXP = 100; type Tnaslov = string[10]; type Tprozor = record naslov : Tnaslov; sir, vis : longint; end;

var a : array[1..MAXN+MAXN, 1..MAXM+MAXM] of char; p : array[1..MAXP] of Tprozor; temp : Tprozor; n, m, np, i, j, k : longint;

begin (* ucitavanje... *) readln(n, m); for i:=1 to n do begin for j:=1 to m do read(a[i, j]); readln; end; (* ...ucitavanje *) (* trazenje prozora... *) np := 0; for i:=1 to n do for j:=1 to m do if a[i, j]='+' then begin np:=np+1; (* trazimo sirinu... *) for k:=j+1 to m do if a[i, k]='+' then begin p[np].sir := k-j+1; break; end; (* trazimo visinu... *) for k:=i+1 to n do if a[k, j]='+' then begin p[np].vis := k-i+1; break; end;

(* spremamo naslov... *) p[np].naslov := ''; k:=j+1; while a[i, k]<>'|' do k:=k+1; k:=k+1; while a[i, k]<>'|' do

Takmičenja

Page 62: Zanimljiva matematika

60

begin p[np].naslov := p[np].naslov + a[i, k]; k:=k+1; end; (* jos preostaje prebrisati prozor... *) for k:=1 to p[np].sir do begin a[i, j+k-1] := '.'; a[i+p[np].vis-1, j+k-1] := '.'; end; for k:=1 to p[np].vis do begin a[i+k-1, j] := '.'; a[i+k-1, j+p[np].sir-1] := '.'; end; (* ...prozor je prebrisan *) end; (* ...trazenje prozora *) (* sortiranje prozora... *) for i:=1 to np do for j:=i+1 to np do if p[i].naslov>p[j].naslov then begin temp := p[i]; p[i] := p[j]; p[j] := temp; end; (* ...sortiranje prozora *)

(* crtanje prozora... *) for k:=1 to np do begin (* vodoravne crtice... *) for i:=1 to p[k].sir-2 do begin a[k, k+i]:='-'; a[k+p[k].vis-1, k+i]:='-'; end; (* naslov... *) j:=(p[k].sir-length(p[k].naslov)) div 2; a[k, k+j-1]:='|'; a[k, k+j+length(p[k].naslov)]:='|'; for i:=1 to length(p[k].naslov) do a[k, k+j-1+i]:=p[k].naslov[i];

(* vertikalne crtice... *) for i:=1 to p[k].vis-2 do begin a[k+i, k]:='|'; a[k+i, k+p[k].sir-1]:='|'; end;

(* kutovi... *) a[k, k]:='+'; a[k, k+p[k].sir-1]:='+'; a[k+p[k].vis-1, k]:='+'; a[k+p[k].vis-1, k+p[k].sir-1]:='+';

(* sredina... *) for i:=1 to p[k].vis-2 do for j:=1 to p[k].sir-2 do a[k+i, k+j]:='.'; end; (* ...crtanje prozora *) (* stampanje podataka... *) for i:=1 to n do begin for j:=1 to m do write(a[i, j]); writeln; end; (* ...stampanje podataka *)end.

Takmičenja

Page 63: Zanimljiva matematika

61

2. Prvo rješenje: Posmatrajmo sljedeći skup duži: 4 stranice kvadrata i 2N duži koji su presjek pojasa koji su avioni pregledali sa stranicama kvadrata. Odredimo sve presjeke tih duži, po parovima. Ako postoji tačka koja nije pregledana, ona mora biti u okolini neke od presječnih tačaka posmatranih duži. Razmotrimo 4 ugla koja se dobijaju kao presjek dva pojasa koja su pregledala dva aviona i neka je A tjeme takvog ugla (vidi sliku). Ako je jedna od duži stranica kvarata, treba posmatrati samo uglove koji su u unutrašnjsoti kvadrata. Odredimo presjeke simetrala svakog od uglova sa najbližom duži koja ne prolazi kroz tačku A i označimo te tačke sa B1, B2, B3 i B4. Dovoljno je provjeriti da li središta duži AB1, AB2, AB3 i AB4 pripadaju nekom od pojaseva. Drugo rješenje: Provjerimo da li sva 4 tjemena kvadrata leže u nekom od pojaseva ili postoji jedno od tjemena koje nije pregledano nijednim od aviona. U oba slučaja, zadatak je riješen. Ako zadatak nije riješen, podijelimo svaku stranu na polovine i dobijemo 4 manja kvadrata. Analiziramo tjemena svakog od dobijenih manjih kvadrata. Sve kvadrate čija su sva 4 tjemena pokrivena jednim pojasom eliminišemo iza razmatranja. Ovim postupkom naćićemo rješenje ili će dužina stranice kvadrata postati toliko mala da će računar smatrati da je dužine 0. U drugom slučaju, kvadrat prelazi u tačku i smatraćemo da je pregledan nekim od aviona.

3. Autobusi idu redom jedan iza drugog: broj 2 iza broja 1, broj broj 3 iza broja 2, ..., broj 1 iza broja N. Autobus koji za vrijeme Tmin prelazi najduži mogući put mora se kretati najvećom brzinom Vmax. Ako odredimo najduži put i znajući Vmax, može se odrediti Tmin. Znajući vrijeme i pređena rastojanja drugih autobusa, lako je odrediti njihove brzine. Prvo odredimo rastojanja između autobusa poslije uklanjanja K autobusa sa linije. Zajedno sa tim dijelom koda pamtimo i brojeve preostalih autobusa. J:= N - K + 1; for I:= N downto 1 do if M[I] = 0 then inc(MDist[J]) else begin dec(J); MDist[J]:= 1; MNum[J]:= I; end; MDist[1]:= MDist[1] + MDist[N - K + 1];

Rastojanje između preostalih autobusa računamo kao broj intervala između autobusa prije uklanjanja K autobusa sa linije, pa koristimo niz cijelih brijeva a ne niz realnih brojeva. Polazimo od toga da je brzina prvog autobusa minimalna, i računamo puteve koji su prešli ostali autobusi, pamteći najduži i najkraći put. MaxWeg:= 0; MinWeg:= 0; D:= 0; Flag:= false; T:= 0; VVV:= V0; for I:= 2 to N - K do begin Work:= MDist[I] * FInt - LInt + D; if abs(Work) < 5.E-10 then Work:= 0; if Work <> MaxWeg then Flag:= true; if Work > MaxWeg then MaxWeg:= Work; if Work < MinWeg then MinWeg:= Work; D:= Work;end;

Sada određujemo vrijeme prvog autobusa if Flag then begin T:= (MaxWeg - MinWeg)/(VMax - VMin); VVV:= -MinWeg/T + VMin; end; writeln(Out, T:1:4); writeln(Out, MNum[1], ' ', VVV:1:4);

I na kraju:D:= 0; for I:= 2 to N - K do begin Work:= MDist[I] * FInt - LInt + D; if Flag then VVV:= (Work - MinWeg)/T + VMin; writeln(Out, MNum[I], ' ', VVV:1:4); D:=Work; end;

Takmičenja

B4

B2

B1

B3

A

Page 64: Zanimljiva matematika

62

U srednje škole Crne Gore nedavno je uveden novi nastavni program po kome se izučava svijet moderne informatike. U sklopu opšte gimnazije informatika je uvedena kao obavezan izborni predmet za prvu godinu i omogućena je kao izborni za drugu, treću i četvrtu godinu. Dakle, u prvoj godini uče se dva najosnovnija programa iz Microsoft office paketa, Word i Power Point. Ova dva osnovna programa iskorišćena su za upoznavanje učenika sa interfejsom, kreiranjem datoteka i slično. Problem se javlja kada se u drugoj godini prelazi na naprednije tehnologije. Microsoft office sve više profesionalaca smatra neozbiljnim kada se radi o Web dizajnu (Ekspression Web) i radu sa bazama podataka (Access). Uprkos ovome, ova dva programa ulaze u predmetne programe namijenjene drugoj i trećoj godini. Idemo li naprijed ili nazad?Za učenike druge godine predviđeno je da izučavaju Web prezentacije, tj. dvije osnovne tehnologije koje pokreću statične Web stranice: CSS i XHTML kroz upotrebu već pomenutog Expression Web-a.CSS (Cascading Style Sheets) namijenjen je izgradnji stila Web stranice, tačnije, koristi se za podjelu Web sajta po tabelama i dodjeljivanje karakteristika (pozadinska boja

ili slika, format teksta itd.) tj. odvaja izgled stranice od njenog sadržaja. XHTML je posljednja, standardizovana verzija HTML-a (HyperText Markup Language). On je sama osnova svjetskog Web-a. Prevod naziva govori o njegovoj funkciji - koristi se za formatiranje hiperteksta.Ove dvije tehnologije čine samu osnovu Web-a, ali svakako ne otvaraju njihovom poznavaocu sve mogućnosti. Moderni Web sajtovi obuhvataju tehnologije za izradu animacija, dinamike, interaktivnosti sa korisnikom, baza podataka. Tako su u stalnoj upotrebi Flash animacije, PHP Web aplikacije, SQL baze podataka...Jedan Web developer trebalo bi da nastavi izučavanje kroz upoznavanje sa već pomenutim PHP-om (Hypertext PreProcessor). Ovaj jezik koristi se za izradu dinamike Web sajta i interaktivnosti sa korisnikom. Njegova

funkcija je da automatski generiše hipertekst na osnovu podataka koje mu unese korisnik. Da bi se iskoristile sve mogućnosti ovog jezika, često je potrebno poznavati i SQL baze podataka kao i njihov interpretator MySQL. Njegovo korišćenje omogućava PHP-u da sačuva i ponovo koristi podatke koje korisnik unese. Ovako funkcionišu blogovi i forumi Pored poglavlja o programiranju (Web development), razvijeno je i ono o dizajniranju Web stranica. Današnji Web dizajneri koriste alate kao sto su Photoshop, za obradu fotografija, ili Flash, za izradu animacije, koje često i same imaju mogućnost interaktivnosti sa korisnikom.Prema ovome za učenike koji izaberu da se bave informatikom bilo bi korisno uvesti programe i tehnologije koji su upotrebljiviji i moderniji u svijetu informatike. Evo jednog predloga:Od kako su se prošle godine ujedinili Adobe i Macromedia, novi paket Creative Suite 3 sadrži sve što jednom dizajaneru, Web i grafičkom, animatoru, video ili audio producentu može zatrebati a cijena nije ništa veća od Microsoft-ovih paketa.Naravno, bilo bi previše tražiti od učenika srednje škole

poznavanje svake od ovih oblasti.Osnovnim paketom moglo bi se omogućiti:na prvoj godini - obrada foto-gra fija u Photoshop-u, crtajnje vektorske grafike u Illustrator-u na drugoj godini - Flash animacije na trećoj godini - Dreamweaver i postavljanje na Internet.Na taj način bi učenicima bilo neuporedivo interesantnije i bili

bi bliži svjetskim tokovima informacionih tehnologija. Adobe-ov paket preporučuje njegova upotrebljivost, softverska kompatibilnost i rasprostranjenost među korisnicima Ono što se zasad može naučiti na časovima Web prezentacija je sama osnova, ali je potrebno dosta nadgradnje znanja da bi se nivo mogao dovesti do profesionalno upotrebljivog. Sa druge strane, ako ste zainteresovani, možete obratiti pažnju na već pomenute tehnologije preko drugih kanala. U sljedećem broju časopisa biće više govora o nekoj od njih...

Matija Tapušković II-2

Kako učiti informatiku

Nešto Vam nije jasno? Nikada niste čuli za to? Onda ste osjetili poentu ! ! !

ili

U koraku sa...

Page 65: Zanimljiva matematika

63

Rješenja

Zanimljiva matematika

Ljepljivi smajliji:

10 poena M I I I

13 poena M I I B B

17 poena M M M B B B

Jedno rješenje koje pokazuje koliko je smajlija svakog igrača palo u svaki red. Pronađite i druga.

Krivudavi zatvor: Brz način kako odrediti da li se mrav nalazi unutar ili izvan krivudavog zatvora je da se prebroji koliko puta zamišljena linija, koja kreće od mrava ka spoljašnosti, siječe krivudavu liniju zatvora. Ako linija siječe krivu paran broj puta, mrav se nalazi izvan zatvora; ako je u pitanju neparan broj puta, onda je mrav unutar krivudavog zatvora.

Elegantne elipse: Na slici je prikazano jedno rješenje. Koliko bi teže bilo da smo dodali više elipsi ili više linija koje dijele. Kako bi kompjter riješio ovaj problem?

Tri kvadrata: Evo jednog rješenja

1 2 8 9 6 4

10 11 12 5 7 3

Možete li pronaći druga ?

Tajanstveni niz: U pitanju je broj 7. Rješenje je povezano sa brojem segmenata na standardnom ekranu digitrona koji prikazuje cifre počev od 0.

Nađite uljeza

Uljez je broj 54

Svi ostali su za jedan manji od kvadratnog korijena nekog broja (4, 9, 16, 25, 36, 49, 64)

Iz jednog poteza

Krenuti iz tjemena pravog ugla.

Datum

Ako parne datume, kada je bio utorak označimo sa x, x + 14 i x + 28, onda je x = 2, pa su dani 2, 16 i 30. Poslednji petak je bio 26. dan u mjesecu.

Matematičko-logičke zagonetke i problemi

1. Euro nije izgubljen; kelnerova 2 eura ne treba dodavati iznosu od 12 eura već oduzeti od ovog iznosa da bismo dobili 10 eura, koliko iznosi račun. Situacija je veoma jasna: kod blagajnika je ostalo 10 eura (iznos računa), kod kelnera 2 eura i po 1 euro kod gostiju, što iznosi tačno 15 eura.

2. Podjela nije izvršena pravilno, niti je to moguće učiniti.

Naime, kako je 12

13

19

1718

3436

+ + = = , dakle, manje od je-

dnog cijelog, pri podjeli automobila javio bi se ostatak.Zbog toga je podjela prema testamentu nemoguća uprkos snalažljivosti matematičara.Pozajmljivanje jednog automobila ima za posljedicu da je svako od braće dobio više jer je:

18 352

6 353

2 359

> > >, , .

3. Označimo sa I istinito a sa L lažno tvrđenje. Moguće su četiti kombinacije: II, IL, LI i LL. Prvu kombinaciju treba odbaciti jer je rečeno da je bar jedna izreka lažna. Druga i treća ko mbi-nacija takođe ne zadovoljavaju uslove jer u oba slučaja, ako jedno dijete laže, drugo dijete ne može govoriti istinu. Prema tome, oba djeteta lažu. Dakle, dječak ima riđu, a djevojčica crnu kosu

4. Uključite prvi prekidač i ostavite ga da radi 5 minuta (koliko je potrebno da se dobro zagrije sijalica). Ugasite ga i uključite drugi prekidač. Uđete u sobu. Ako sijalica svijetli, onda drugi prekidač pali sijalicu. Ako ne svijetli, pipnete sijalicu. Ako je sijalica topla, onda prvi prekidač pali sijalicu, ako nije onda je to treći prekidač.

5. Lako je provjeriti da nijedan trocifren broj koji se može napisati pomoću cifara 3, 1 i 6 (njih ukupno šest), nije djeljiv sa 7 Rješenje je vrlo prosto i duhovito (sjetite se, radi se o gimnastičarima). Potrebno je da gimnastičar sa brojem 6 dubi na glavi (tako se dobija broj 9), a da pored njega stanu (ili još bolje, kleknu) gimnastičari sa brojevima 3 i 1. Dobijeni broj 931 djeljiv je s 7.

SofizmiDijeljenje nulom! S obzirom na to da je A = B, to je A – B = 0, pa ne možemo dijeliti sa A – B

Sudoku Lakša Teža

8 3 1 9 7 2 5 6 46 5 2 4 3 8 9 1 79 4 7 5 6 1 2 8 34 7 9 2 1 6 3 5 83 1 8 7 4 5 6 2 95 2 6 8 9 3 7 4 12 8 4 3 5 9 1 7 67 6 3 1 2 4 8 9 51 9 5 6 8 7 4 3 2

8 3 9 1 5 2 7 4 66 1 4 7 3 8 9 5 25 2 7 6 4 9 3 1 83 8 6 4 9 5 2 7 11 4 5 2 7 3 8 6 97 9 2 8 1 6 5 3 49 7 1 3 2 4 6 8 52 6 3 5 8 1 4 9 74 5 8 9 6 7 1 2 3

Rješenja

Page 66: Zanimljiva matematika

64

Šibice

1.

2. Okrenuti jednačinu naopako.3.

4.

Plava tečnost

– 40 stepeni Celzijusove skale jednako je – 40 stepeni Farenhajtove skale. Konverzija iz °C u °F se dobija kada se vrijednost pomnoži sa 9/5 i doda 32. Konverzija iz °F u °C obrnuto, dobija se kada se oduzme 32, zatim razultat pomnoži sa 5/9:(– 40 °C) ∙ 9/5 = – 72 + 32 = = – 40 °F,slično – 40 °F – 32 = (–72) ∙ 5/9 = – 40 °C.

Zanimljiva fizika - pitanja i odgovori 10 + 10

1. Na prvi pogled, pozitivan odgovor bi protivrječio Kulonovom zakonu. Ali, prisjetimo se da naelektrisana tijela privlače nenaelektrisana (na primjer, komadiće papira privlači naelektrisani češalj), a to takođe formalno ne odgovara Kulonovom zakonu. Ovakvo privlačenje objašnjava se preraspodjelom naelektrisanja u tijelu (sl. 1).

Slika 1 Slika 2

Ovaj efekat može dovesti do toga da se privlače i istoimeno naelektrisana tijela: „bliža” strana jednog tijela može izmijeniti znak naelektrisanja (sl. 2). To je moguće kada se tijela nalaze dovoljno blizu jedno drugom i kada je naelektrisanje jednog od tijela mnogo veće od naelektrisanja drugog. U tom slučaju tijelo s manjim naelektrisanjem, opisanom preraspodjelom može dovesti do privlačenja naelektrisanja suprotnog znaka.

2. Označimo sa A dužinu daske, a sa M rastojanje koje pređe osa cilindra do trenutka kada drugi kraj daske dođe do njega. Pošto je brzina daske u odnosu na podlogu dvaput veća od brzine ose cilindra, onda je A + M = 2M, tj. M = A Prema tome, čovjek treba da pređe 20 metara.Zadatak se može elegantnije riješiti u sistemu referencije „vezanom” za dasku. U tom sistemu osa cilindra se kreće prema čovjeku brzinom istog intenziteta kao i u sistemu „vezanom” za podlogu i za određeno vrijeme preći će 10

metara, tj. stići će do drugog kraja daske. Za isto to vrijeme čovjek, čija je brzina dvaput veća, preći će 20 metara.

3. Crvenu, jer se frekvencija svjetlosti ne mijenja.

4. U ovom slučaju nije eksplicitno navedeno da se otpor vazduha može zanemariti, pa je kinetička energija lopte na kraju leta manja nego na početku. Takođe, na bilo kojoj visini, brzina lopte je manja pri spuštanju nego pri podizanju. Odavde je jasno da je vrijeme penjanja kraće od vremena spuštanja.

5. Kondukter i putnik koji je čitavo vrijeme sjedio prešli su jednake puteve u odnosu na zemlju.

6. Lik koji se dobija poslije odbijanja zraka od oba ogledala, poklapa se sa samim izvorom.

7. Svrha nepokretnog kotura je samo da promijeni smjer sile. Da bismo podigli teret težine Q, potrebno je da vučemo silom F1= 100N prema dolje (slika A).Pokretni kotur djelovanje težine tereta prepolovi na obje strane užeta (slika B). Slijedi, pokretni kotur smanji silu

potrebnu za dizanje tereta na polovinu: F Q2 2= = 50 N.

1F

2F

QQ

Slika A Slika B

S druge strane, da bi se teret podigao za visinu H, kraj užeta treba vući za dvostruko veću dužinu (2H).

8. Sunce nam je najbliže u zimskom solsticijumu (21. decembra), a najudaljenije u ljetnjem solsticijumu (21. juna).Više temperature ljeti uslovljene su time što u ljetnjem solsticijumu sunčevi zraci padaju pod najvećim uglom, a u zimskom pod najmanjim uglom prema našem horizontu.

9. Uđite u lift, stanite na vagu i posmatrajte težinu koju vaga pokazuje kada liftom krenete vertikalno naniže s odgovarajućim ubrzanjem. Naravno, ovdje nije riječ o smanjenju mase tijela, nego o težini kao sili kojom pritiskamo vagu.

10. Polovina energije E prešla je u unutrašnju energiju. Ne može se zanemariti sila trenja, bez obzira na njen intenzitet. Pri spuštanju tijela u sistemu dolazi do oscilacija: tijelo po inerciji „prolazi” ravnotežni položaj, prolazi naniže, zatim se vraća naviše i, da nema trenja, ovo oscilovanje se nikad ne bi završilo. (Smanjenjem sile trenja ne bismo mogli da utičemo na gubitak energije, već bismo samo usporili proces dolaska ovog sistema do ravnotežnog položaja.)

Rješenja

Page 67: Zanimljiva matematika

Nakon osam godina ponovo sa Vama!

γ Šta je to GAMA?

GAMA je časopis za kreativne mlade matematičare, fizičare i programere, ali i za one koji su dosad mislili da to nijesu.

γ KomejenamijenjenčasopisGAMA?

Svim učenicima gimnazija i ostalih srednjih škola u Crnoj Gori, ali će u njemu ponešto moći da pročitaju i mlađi, a mnoge stvari će, svakako, zanimati i starije!

γ Ko je zaslužan za drugi broj časopisaGAMA?

U prvom redu, kreativni učenici Gimnazije „Slobodan Škerović“ iz Podgorice, a zatim grupa profesora entuzijasta iz iste škole kao i profesori sa Prirodno-matematičkog fakulteta, iz Zavoda za školstvo i Ispitnog centra.

γ ŠtasuambiciječasopisaGAMA?

Prije svega da ga prihvatite kao najbolji časopis za popularisanje matematike, fizike i informatike. U tom cilju trudićemo se da sa čitaocima, kako učenicima tako i profesorima i nastavnicima, uspostavimo što prisniju saradnju, čime ćemo iz broja u broj biti još bolji. To zaista nije teško jer matematika i fizika su tako lijepe, korisne i neizbježne, toliko prepune divnih ideja i neočekivanih rješenja „nerješivih“ problema da je dovoljno biti samo aktivni posmatrač, učesnik i objektivni reporter, naravno uz veliku informatičku podršku.

Dakle, PIŠITE NAM! KRITIKUJTE NAS ILI POHVALITE! PITAJTE NAS! PREDLAŽITENAM!

REDAKCIJA

Napomena: Zbog finansijskih problema, koji nažalost prate jedan ovakav časopis, nijesmo mogli da štampamo časopis u prošloj školskoj godini. Zato skrećemo pažnju čitaocima da su odjeljenja učenika, autora članaka, važeća za školsku 2007/08. godinu.

Ne znati nije sramota.

Sramota je ne htjeti znati.

Sokrat

Page 68: Zanimljiva matematika